You are on page 1of 124

BLACKWELL'S

V I W RHZTSHAN, MD
Llnivcrsirv of (?dIifornia, Sa11 Fnncisco. (:lass of 199 1 Ccrirs Ecli~o!: Lliag~~nsticr hdiologist

VISHAL PALL, MBBS


Governmertt Medical Collrgr, Chandi~qrlh, India, Class of 1956 S ~ r ~Editor, es 1 1 .nf Trx;t<,C:~lvestcln,Rrrsidrnt in Inter11:ll hltdicinc k Pwvrnrivr Merlicirle

TAO LE, MD
Llniversirv of California. San Fr~~~ciqco, Claqs ot lW(i

HOANG NGUYEN, MD, MBA


Nr>r~tllweslr~.n Univrl-sty3Cla,ua of 20Ol

W A L SONL,
L T ( : L i \ SchooI n t Mrtliri~ic, Class of 19!)9

Blackwell Science

CONTRIBUTORS
Robert Nwon LTrlivrrcizy OF Texas Mrdiral Branch, Class of 2003
Tisha Wang ITnivri-sity of Texas Medical Brat~ch, Class nf ?OO% Kristen Lem Mygdal, MD University of Eia~isa.; School of Mcrlicine, Res~clentin R a r l i o l n ~

Fadi Abr~ Shahin. M D Vnircrsity of Uarrrascus. Syt-ia, Clmq of' 1999


Jose M. fierro, MD La Sallr Cnir-rrsity. Mexico Cin

O 2002 Ijv Dlack~~c11 Science. Tnr.

Editorial Offices: Con~n~ercc ['lace, 350 Main Slreel, Maldcn. Marsachuscus 132148, l'S:l nsnev Mexi, Clxforrl OX2 OEI., E n g l ; ~ ~ ~ r l 25John S11.cc1, Lonclnn WCI N 2BS. frlglancl 23.4inslic Place, E d i n h i l r d ~ EHJ 64).Scorlancl 54 C'nivprsih Siieel, G~rFtvn,l ' l c t n l i a 30.33, AuatraIia
Other Editorial Offices: Hlarkrv~ll t+'i.;sen.;cIr:tt~+Vcr!afi GmbH,

4r qnisiuons: Laura -il~\i>uug Dezfclupmcr~t: Amv \'utrhr.ock Plndr~rtinn: Lrlrna I lincl a n d Sh.1wri G I I S ~ P ~ ~ P T M a n u i : ~ c t nq: ~ ~ rI ~ .lsa FIanaqi~~ Markctinq M a n a g e r h t h l e r n Mulcaliv Cover design hv Lcslie Hairnes 111terinrdesikqi by Sliarm Cirqlwrgrr Tvprset b\ Tech l3o(,k< Pnt~tctl and hound b? Cnpital Cirv P r w u

K~~rfi~rstr.rtdan~rn 57. l(1707 Rerlin, Crnnanzr Rlackwell Srier~ce KK. MG Kodenmarhrl Building. 7-1 0 );nrIenrn;tchn Nihornh.~rl~~. Chrlrrku. Tokvo 101.Jnpan Sriencr Euqo is a rrarle mark ot 1rlw;i State Un~vcnim P~PFF. .ZBIdlkwcl! SCLCIICC Tht. I3laclt1~cll UlacLw~11 Scic~tcc L~rl.. tcxistrrrtl >tt~ h L'nited c C:nmpaiiy, 2121 S . Stale :\venue. :\mrr, l(nw liin~dnrn Tr:lrlc Mnrkr Regirtry 500 lC8JOO. USA
Wsb-ibutors:

Blackwell's Urlderground Clinical Vignettes: Pathaphysiology I,3e ISBN 0-R3%0455 1-5

TIIP : l rnprirnl Rlackwell Publishing


C/O

nlr)r:

PO. Box 2(1 50 tVin1rr Sport Lane will is tor^. \rf 054950tl?o (Tclephonc orders: R0&2 I &252.5"2: tau nrdrrs- tl0Wti?-7ti%) A zirfmlm Bl;~ckwell Science Pry, [-Id. 54 ITniv~r~i@ SLTYCL Carlrtln, t'irtoria 3053 ~Telepliur~c o l d c r s n3Y547-WW0:

fax orders: tI.%934%3.7016) U l ~ t ~ TTJ id~ ,4 mrn'ms un(1 i.tu~tmlfrr B1:rckwcll Science. I .td c/n Mamtnn h r ~ Srviccs, k Lt~l. P.0. 'ti!) Ahingdnn Oxon OX14 +YN Ertgla~ld (Telrphrlne orrlrw: 44-01 235465iOk tax orden: 4 M 1233465555
All rights rcscrr~cd.No part o f this hrlc~k11iav be repmrluc~d in any fnmm ur 1,y any elcctrnnir o r mechanicdl nlcalls. incllldirl~ infnrmntirln stomgC ant! 1 . ~irvdl 1 ~ ~ y ~ r ~ rwiYilhu111 n5. pcnlltwon it1 wrltnng frnm lhe pul~lisller: cxcrpt by n reviewer whu may q i ~ o hriel ~ c ]m;lssa~es in a rewrw.

Library of C u n p s s Cataloging-in-Puhlimtian Data Rttushan. Vika~, BIackwiwrll's r ~ n d e r g r o ~ ~clinical t>tl \ipetles. Parhoph'i.uiolug / aurhrx, Vikav Dllushan. - Snl eri. p. :cqn. - tUndergrr>undclirlic:~lvigmettes) Rex ed. of: Pathr~pt~ysiolcrg~ / Vikas Eh~rsl~ar). 2r1d rrl. clYY!F. lSRN Wf324.1551-5 Iphli.) I . Pl~ysioloffy, Patholr+$cal -Case studies. 2. Miwirians - Liccnses - I'niterl Statcs Exami nalioirs - Study p~irles. [DNLM: I. C:linical Mcrlicine - C:ase l<eporl. 2, Clinical Medicin? - Prnhlems ;mtE Ext:rci~es.1liR 18.2 R575hh 2 0 W ] I. Title: Unrle~-gm~i~ntl clit~ical rigne1tc.s. Pa~l~r>pIiysiolr)~y. 11. P a ~ t ~ o p l ~ ~ i n l r ~ g y . 111. 'l'illc. S~rien. KB 1 13 .B459 SO02 ti I fi.OT'OTtic;!l

n :

Notice T S r c nurhnl-; of this vnl~lrnt1i;rvt. t.tkc11cnrr TIIAI thr ir~fi,~-mntitnn rontainrtl hcl,cin i s accumre a.nd cornpa~ihlc with t l ~ e skundartls gc~iet;~lEy a c c r p t ~ r: l i~ ~ h time r r j i pullicadrm. Neve~.~l~rlcss. it is rliffic!rlt lo cnsure i11a1 at1 LIIC i~irornlation given is t.r~tirc.[raccur;itc ii:rr all circum~rant.es. TIIFpi~hlisI>cr and ar~tlinn dn nr)t guarantw the rrm1rnr.s (11- tllik Ijnuk and cliscbi~n arlv IiahiliR loss. or r l i ~ n l a ~ incnrred c ac a cnnseqner~c-e, dirrctlv o r ir~direcrlv, r r t [Flc Lrsc nrtd applirarinn n f nnv oF lhr crbrllcnts elf ihic vnlumr.

CONTENTS
Acknowledgments Preface t o the 3rd Edition How to Use This Book Abbreviations
x

xiii
wi xvii

Aortic Dissection Aortic Insufficiency Aortic Stenosis Atherosclerosis Atrial Fibrillation Atrial Myxoma CAD-Myocardial Infarction Cardiac Tamponade Cardiac Transplant

Congestive Heart Failure Constrictive Pericarditis Co r Pulmanale Dilated Cardiomyopathy Eisenmenger's Complex Hi gh-Altitude Sickness Hypertrophic Obstructive Cardbmyopathy
Hypothermia Malignant Hypertension Marantic Endocarditis Mitral Insufficiency Mitral Stenosis Myocarditis Peripheral Arterial Embolism Shock-Hypovokmic Sinus Bradycardia Thrornboangiitis Obliterans (I3 uerger's Disease) Thrombophlebitis-Superficial Wolff-Parkinson-White Syndrome Actinic Keratosis Atopic Dermatitis Basal Cell Carcinoma Contact Dermatitis Dermatitis Herpetifomis Dysplastic Nevus Syndrome Erythema Multiforme Furuncle

Dermatology

Gastroenterology

Kaposi's Sarcoma Kawasaki's Syndrome Lichen Planus Malignant Melanoma Mycosis Fungoides Osler-Weber-Rendu Syndrome Pernphigus Pitynasis Rosea Psoriasis Pgoderma Gangrenosum Seborrheic Dermatitis Vitiligo Gushing's Disease De Quenmin's Thyroiditis Diabetes Mellitus Type I (Juvenile Onset) Diabetes Mellitus Type I1 (Adult Onset) Hashimoto's Thyroiditis Hyperthyroidism (Solitary Nodule) Bypopituitarism Hypothyroidism-Congenital Hypothyroidism-Primary Pinealorna Frolactinoma Sporadic Multinodular Goitel Thyroid Carcinoma Acute Angle-Closure Glaucoma Benign Positional Vextigo Cribriform Rate Fracture Deafness-Sensorineural Labynnthitis Obstructive Sleep Apnea Open-Angle Glaucoma Presbycusis Presbyopia Pseudotumor Cerebri Retinitis Pigmentosa Reiinoblastoma Uveitis Achalasia Ascending Cholangitis Barrett's Esophagus Budd-Chiari Syndrome Candida Esophagitis Celiac Disease

' : :p

rn
y,:,;:

1., : y-;
t 7 :,,'!
" <

, "
A

',<?
, ,

i
? :

, ,
; ,

",: , ., ,

I:: r , ,
P T V , V

L'

, ?

Chronic Atrophic Gastritis Chronic Pancreatitis Colonic Polyps Crohn's Disease


Div~rticuLitis

E a
r73

r' 1

r;+ I
f'"
I : !
K

Diverticulosis Esophageal Spasm Esophageal Variceal Bleeding Gastroesophageal Reflux Disease (GERD)
Bernochramatosis

r''
F';
- 7

Hepatic Cirrhosis Hepatic Encephalopathy Hepatitis-Alcoholic Hepatocellular Carcinoma Hepatorenal Syndrome Metastatic Carcinoma-Liver
Pancreatitis-Acute Peutz-Jeghers Synd~orne Plummer-Vinson Syndrome Primary Bibary Cirrhosis Toxic Megacolon Ulcerative Colitis

-7
-7

r.
i'
f-

1
p-

-1

1
k-

'1
#'

-4

Zollinger-Ellison Syndrome

-1

*ThrougI~otl t I h c prncliiclic~n or t h i s hnok, we harre had the supporr or rna~iy fi-irr~rls atlrl cc~llragt~es. Special thanks l o o u r support team including Ann Gupta. Andrea Fellows, Anastasia Anderson. S r i ~ h tGupta, i Mona Pall,,Jonathan Kirsch nnd Cl~irag Xinin. Ft-rr prior contrih~ltions ~ c thank c Giar~~ Le ri N~averl. I i r u ~ Mathur. i ,4lcx Griannl, Sonia Sanlos and Elizahet lr Sandcrs. I1at.r etjoycd working with a WO~~CI-CIZL~S intel-nadonal p u b lishing ernup a ( Rlackwetl Science. incl~~rlinp; IdauraDcYo~mfi. X~ny Knttbrock, Lisa Flanagan. Shawn Girsbergcr. Lorna Hilid and Gordun Tihhitts. For llrlp with bccuring inlager for the c.n tirc SCI-ics wr- alsu thank Lce Martin. Kristophel-Jones. Tina Panir/i and Peter Andei.son a t the tlniversity of libhama. the Armed Forces Institute nf Patl~ologyand many of o u r kllow Black~vell Scicnce authni-s.
wt=

For s11l11nittingconnncnts, correctiot~q, editing, prcjofreading, and ,~ssistarir~ act-osq LIII of' the vignette titles in all cclitir~ns. wr coIlectively thank:
K1.i~ 4rlen. Henry E. i2rv;ln. I . y ~ t ~ ~ISarnlol-, atl Nanl,?Eie Eart~.neva. Dean Bartholurrlew. Dehashish Rehera. Surmit Bhatia. Sanjay Binclr-a, Dave Rrinlori, -Ji~liannc Rrurin. hlusanclcr Bsownie. T a r r ~ a tChllal .~ Ian. Davirl Canes. R l y n C;isey. Aaron Carlghey. H p i t ~ lChen. -~ ,Jonathan Cherlg, hnr)lrl Cheung, Xmt,Irl Chin. Simion Cl-~iose;~, Yoon Cho, Samuel Cliuiig, Gr.rtcl1err C n l i a n t , Vlarlimir Col-ic, Chi-irtophcr Cnsgrove, KonaFd Co~.van, Threkin R. Cunniugll:li-ri, A. Seal1 Ilalley. Ralna L)a~-irl;lmucli, %nit Das, Rvan r2rn1anclo Dave, Jol~rt Daiid, Emmanucl clc la Cmz. Robrrt DrMello. N d v n ~ e i Ilhillntl, Sllnrtnila 13issanaike. Davicl Donson, ,4dolCEtchegarity, A e a Eusehio, Pri~cilla A. Frase. David Frenz. Thistin Gaurr~er. Yc>hanncsGchrcu~zial~hcr. . h i 1 Gclii. To~ty C*r~~-ge. 1L.M. Gocancu. Panrl G o p l . ,Vex Grirnm, Rajeev G~lpta, :\hmarl Hallrn, S t ~ e Hall, Dacirl Has~ell>achec Tamm I-Ici~nurt, Michelle Hig1c.v. Dan licit. EricJack?on,Tim Jac k>rm,St~~lrlav Ja~anrnat~, Pci-Ni Jone, /-\arclrar~ Josl~i, K?jni K.Jt~tla. E'aivaz Ehpacli, Seth Tiar-p, rlaron S. K e s s ~ l h r i m Sana , Khan. .Indrew Pin-wci KC), Francis k n g , Pa111 K o n i t z k ~ Warren S. Fj;lckc~v,Rrtijatnin H.S. Ian. i\nn 1,aCnw-e. Connie Lee, S c o ~ t Lee. Guillcrino Lehma1111, I<e\in Lcung. Paul Lcwtt. f i r r u n Lr.\,in~on, Eric Lcv. K e i ~ Lin,
Tini Acl;irnu~ich, Carc~lvrl Alexarlder;

Pace1 Loka1~ov.J.M a ~ k Matldos, h a m Mardian, Satnir Mehta, Gil Melm~rl, Jne Messi tla. Roherl Mrrsca. Illichael Murphv, Viveli Nari tl karni , Sim Naravnan , Gal-vell N g ~ ~ y e i1,inli i , Ng-t~veli. Deauna Noh1ei-a. Chig Nnclul-i't. r ~ n r g e Noumi, Darin T. Oklldn, ilrlarn I.. Palance, Palal Pamphru';, Jinlaa Park, Snnnr~ lxarrl. Ricarclo Pir.11-obon,Riva L. Rahl. AavI~ivaRanrleria, Racluul Rrdtlv. R e a t n u Reig, Mariloil~Rcyus. Jcrerrlv Ricllmon, - h i I<oe,Rick Rullei; R?jilr ROT: Diego lli11i7,:\nihony Russell, Sar!j;~ySahgal, Vr.tr~imal;l Sarlcat; John Schilling, Isabell Schmitt, lhl-cn Schithmncl~er. Sonal Shah, Facli Abu Shahin. M a c Shcikli.LUi. Erlie Shc~r, Justin Smith, John Stulak. Lillian S u , Julie S u n r l a ~ ~ rRita n . Suri. Seth S w c c ~ ~ c Antonio r. Tllmero, Mcri t a I>II. Mark Tanaka. Eric T~vlnl; .Jew T'Ii~>rnpwrl, Iiidi Trehan, Rivlnnnd T~rlllel; Bkdo Llcl-ter~na, Eric 'Llvg-uancr,. Richa Viisma, - J o l ~ n \Vages, Alan I$';lng, Eunicc M'ang. Andy Wciss. Amv 1l'illiam.i. Brian \':u~g, Hany Zaky. Ashr;tC Zaman ancl Da\+idZipf.
rtip entire I:ndrnrnr,l~t?d C1171irftt Viptptrr Stchp 1 s e ~ i e sTVC . rollectivelv thank thc staff at Rlack~rll Scicncc in Osfr,rcl, Bnston, and Berlin a< well as:

For gen~rrruqlv ron~r-ih~lrinq inxqec to

Rcl g D. ~ \ d u n n r ~Clinirnl d SIaI1.I nnd Plrv~irnl Dtnposix.

RIackwcll Scielicc Lld.. 1Y!1!). Figlrrcs 7.10. 7.12, 7.13, 7.2, 7.3. i.7. 7.H.7.9. 8.1. 8.2. 8.4. R . 5 , !).2. 10.2, 11.3, I I..?. 1.'.(5.
b

C~~suhicri 12. I-Iunncss~ TPl, Greeilhalg1-1RM.Rrlwlev D.4, Gracr- PI\. Clinir-nf , S Z E I ; ~ P IQ ? . s ~ ~ Mvad: T . Blackivcll Scicncc Ltd. 11196. Fijiurcs 13.19. 18.29, 18.33.

* C;illrr;pieSM,k~rnford F i ,Vlrrlirnl h iirrnl~iolqq n?zd I n f ~ d i o n nf

I ; l ( ~ n t vOst~ev . Meark I<lackt+lell Scienrr Lrd, 2000. F i ~ l r 20, e ~ 23.

* C;i~~st>erg L.I,PI.~JT/, i W r r on :Y1*1rwtu~~. T'" liditrnn. Osney Mead:


Black~vcbll Scie~~c Ctil. e IQ!)I-). F i g ~ u - e 1'.~ '..3,

18.3. 1H.Sb.

Elliott T. Hastir~g.;M, Dc5urlhrrgrr L ' . L~rt11rr ~ Y I Io~ nP Af~t!i('ttl T iMirvol~?olc!p, 3"' I<di/iorr. O s n ~ v Mearl: Dlack~vellScirnr-e Ltd, 1997. Figures 2, .5, '7. 8.9. 1 1 . 12. 14. 15. 16. 17, 1 9 , 2 0 , 2 5 , 26. 27, 29. 30, 34. 3.5,52.

* Mehba ,4B, Huffbrand AV. H ~ m a t o I o g nt n rJlnnr~. 0srlr-y


Mead: Blackwcll Scitnce LtcI, 2000. Figures 22.1. 22.2, 22.3.

Please lei us knnw if yollr name 113s l ~ c e missed ~i or tnisspclled and we will be happv to nrakc t h c up&-ltcin the next udition.

PREFACE TO THE 3R0 E'DFTIOPI


We were verv plca~cd with the n~erwIwlming!vpositive shrr!ent feedhack for the 2 n d rdiiion of our Ihrlwpnunrf Cllziml T r i p r r ~ l t ~ s series. Well over 100,000 c o p i ~ s OF t h ~ITCl' " hooks aru irt print ;~nd hiwe been uwil hy st~~detits all over. the r\.orld.
Over the last rrvo vex5 wc h a w ;accurnulatud alld incorporatecl over a thousand "updates" and irr~provenients srlgge5tecl by ynn. our rradets. includi~ig:

manv atlclition~ of specific lsoards and n~ardstc.stal)lc conten1

rlcletions of recl~~nclant ancl overlapping cascs rpol-del-ingand r e o r p n i z n t i n n n C all cases in hot11 scries

a new rnilqter iriclex hy casr name in each Atlas

d i a ~ ~ o sand i s treatxrlent updates

* arlcliticln nf 5-20 new cases in

e w r y book

and the addition of clinical rxarn pllc>~o~.r,ipli.i williin L T C 1 ' A $1 rrlomj

Aticl mnst impnrtant of all. the thircl editicrn sets now inclr~dc hvo hrancl 1-tcw COLOR AlTA.5 supplcn~er~ts. orlr fnr each Cliniral Vignette wries.
The I ' C T 1 3 n ~ i r . % ~ P I E C ~C o l w ~ l t l (St+ ~ z ~ I) i~lcludes oxw- 2.50 cc~lor plates, tlividetl into gross p;ttliology, inicrnscnpic pxtltol0g-y ( tjist ologv). hett~arnlorn,and microhiolop f sn-icars). The ITL'lf-Cli?rit.al SrEmrr (=llfor ,2tIrrr (.qt~/> 2) ha5 over 125 color platcs. including patient imagrs, dcrmatcllop,?., and fundrlrcc>py.

Each atlas image is descripliuely captioned a n d linked rn its cnrresponding Srep 1 case. S t r p 2 case. anrl/nl- Step 4 Minicase.

How Atlas Links Work:


Step 1 Book Codes are: A =Anatomy Step 2 Book Codes are: ER = Elnri,gr.ntv Medicitlr IMl = ltitcrr~al M~rlic~nc, Vol. r IM2 = In~rrnaIM?dirine. X7t)l.11 BG = Diochcillistrv NEI 1 = Nrlirnlop 3i 1 = \Ii<-rc>l>ioIc>p. Vo1. I OR = (1R/C;YY M2 = \,ficrubioIop, X,?)l.TI PED = Prdirltri( 'I PI = P < l t l l o ~ > t ~ \ ~ s VOI. iolt~ I ~~. SliR = S111.g~ry P? = Ritl~opI~vsiolop-R l'c'l. I1 PbT = Psvch~arrr 1'3 = l':~tRnl~l~r~inlop-, \:ol. I l t MC Mini(:.rct1'1 f = l ' l l ~ l~ l l < t r < j l t > q ~

KS = 13~liav11)~rrl Sciatncc

Indicates Type of Image: H

\/ A
Ease Number

M-P3-032A

EB-03 5A, ER-035B

= Hrrna~nlnpr = M~rroh~oln~y

Indicates UCVl or UEV2 Series

PC: = Grrjrh Patholo? PA1 = Mirrovnpic P ; l r h o l n ~

[r the Case numl~er(1J.32. 0.15, erc.) is 11c-11 followed by a letter, then thcrc is oilIv oilc image. O t h c i - ~ i s A, e B. C, D indicate t1p to . I images.
Bold Faced Links: In nsrler tn give vnu acrew tn the largest nrirnhrr of irnagcs possible, we 1la1.c-chosen to cross link thr Slrp T a n d 2 ueries.

TF thr link i s hold-faced this inclicatcs that the link is direct (i.e, Step 1 Case with the b s i r Scienre Step 1 Atlas link).

* If he !ink is not bnlcl-Faced t h i s indica~es that the link i q it~dirrcc (Step 1 case with Clinical Science Step 2 Atlas link or \.ice vcrsa) .

Mrr 11;lve also implemented a fcw stnlct~lral changes upon your request:

* Each current anrl future cdi tion of our popular Fi~slAid fm


Ihr C,lSA1fT3Scfl) I (Apple~nn K. Lange/McGi;l~v-!-Hill) and Fir%( Ard for / h p I :SI~II.IS, $ t ~ f2 ) (Appleton Rc Lange/MrGraw-Hill) hook will be linked to the corresponding LTCX C B S C .

We elirnix~atcd COT 4 First hid links a s thcy frequently I?ecnm~ nut of date, as t h r 1 2 i ~ Aid ~ r hooks are reviwri yearly.

T h r Color Atlas is also spcciallv clrsi~med f4)r qt1iz;lingciip~ions arc rlc'icriplivr arwl do not g i v ~ army the cx3e name
clirectly.

\$-e l i o ~ e 1I1e 11pr1ate~I C'I:il serirs will remain a

u n i q u e ancl

well-

in tegr,~tedsr u d v (no1 I hat provirles cn tnp:lct clinical correIa tions 10 ha&- science in lbrmation. Thcv arc d ~ s i ~ ~ ct or he l eaFv and liln (compxrativclv) tr) scad. and hclpti~l I'or hnt11 licensing pxatm anct the wards.
i$'e i i n i l e votn- cckrr~ctions a n d ~~iggestions for the fotirth edition ctf these honk?.For thc first submission of each lactual c o y I-cctiou nr new vigi~cttc that is selected tor- iticlusinn i n the Cnurth edition, you ullI rccrivr a prrqclnal ackno~vleclgmcnt in the I-ci-ired hnnk. 1Fvou \ t ~ I > t r ~ i tover 20 high-quality ccmcc~iuns, xdrlirions or new vigllettes wc will also consider inviting you to become a " C o n ~ b ~ r t o r on " the hook of your chaicc. I f v n l ~ are

inlcrc5ted in Iwcoaning a pulc~~ti;ll "C:oi-~ irih~~~n or l -"Author" " CIII a ruturr YC1' book. or worLirlg wir h o ~ i ream r in rleveloping ;~tlcliliclilal hnnks, please a1sr-1 P-mail LIF volir C V / r e s ~ r ~ ~ l r .

I m l ' c prcfcr hat you sriF>mitcorrection% or nlggestinns via clcctronic mail l r ) UCVteam~ahoo.com. 1'1~aseinclude "LTiidergu~~i~d Vignettes" iw the stlhject of your messagc. If volt do not lzavc acccsq to r-mail. usc the following rnailir~g acldress: Rbckwcll I'l~l~lisf~ir~g, .%~tri: LJIT I ~ , c ~ 3.50 ~Main ~ oStreet. ~-F, Malrlen. M h 02148. USA.

HOW TO USE THIS BOOK


This series waq otigit~ally cleveloped to address the increasing number nf clinical v ~ ~ I I P ~ ql~estiotls F P on medical examinations, including the USMLE Step I and Step 2. It is also desigtled to supplement and curnplement the popular F h t Aid jot. .!iw C ! S I U St+ 1 (Appleton & Lange/McGraw Mill) and First Aid jor I ~ Cr,SISIJX P St$2 (Appleton 8c LangeJMcGraw Hill).
Each LrCV 1 book uses a series of*apprnxitnatrlv 100 "supraprototypical" cases as a way to condense testable facts and associations. Tllc clinic31 vignettes in this series are designed tr, illcorporate a rnaily testable facts as possible into a cohesive ancl memorable clinical piclure. The vimetles srpi-esenl composites drawn from general and specialty textbooks, rpference kooks, thousands of USMLE style qucstiuns and the personal experience of the authors and rcritrvcrs.
Although each case tends to present all the signs,symptotns, and diagnostic findings for a particular illness, patients generally will not present with such a "complete" picture either clinically or on a medical examination. Cases are not meant to simulate a potential real patient or an exam vignette. All the boldfaced "bwmords" are for learning purposes and are not necessarily expected to Ile found in any one patient with the disease. Definitions OC selected important let2ns are placed within the vig-nettes in (SMALL WS) in parentheses. Other parenthetical remarks nften refer llle pathophysiology or mechanism of di~ease. The formal shotild also kelp str~dents learn to present cases succinr tly during oral "bullet"presen tar ions nil cli~lical rr~tation%. The c s e s are meant to strvr as a condensed revirw, not as a priman- reference. The information provided in this hook has been prepared wid1 a great deal of thought and careful research. This hook should not, liuwtvcr, be considcrcd as your sole snurre of information. Gorrectior~s, st~ggestions and sd>mi~sions of new cases are encouraged and will bc acknowledged and incotpol-sled when appropriate in future editions.

5-,45A
:WGr

,\n!ru
ACE
ACTH

,4DH ,WP
A1

AIDS lZLL ALT AML IZNA ARDS ASD

As0
.+UT
4 1 7

I3E
RP

BUN
CAD

CALL4 CBC CHI! CK CIS, CML


CMV CNS

coPn
CPK CSF CT L T l : %

5-aminosalicyliu acid arterial blood gases adria1nycin/bleo1n~cin/~4ncristine/dacarl~a7.ine ar~giotensin-converting enzyme adrenocorticotropic hormone antidiurutic hormone alpha f e u 1 pror~in aortic insufficiency acquired irnrnunodcficir-ncv syrtdt-om? acutc lymphocytic leukemia alaninu tnnsarr~inase acute myeIogeno~is leukemia antinuclear antibodv adult rcspiratvry distress syndrome atrial septa1 defect anti-streptofvsin 0 asparrate transaminase artcriovenous barium enema blood pressure blood urea nitrogen coronary artev cIisease common acute Iymphohlastic leukemia antigen complete hIood count congestive heart fail~lt-e cl-eatine kinase chronic 1ynphoq.tic leukemia chronic myelogenous leukemia cytomepfovints ccntral nervous system chronic obstructive pulmonary discasc cl-eatine phosphokinasc c~rebrospinal fluid computed iomog~aphy cere hrnmscuIar accidcnl
chest x-ray

CXR
DIG DIP DKh DM DTR q RVT

clissemii~aced intravascular coagulation

distal inrerphalangeal diabetic ketoacidosis diabetes mrllitus deep tendon reflexes deep venous tl~rornbosis

E I3C'
EEX; Echo

EF KGEl EMG EKCP ESR FEY


FNrZ

rtecrr-omvographv
e~~rloscopic re~rograrlec I i o l a i ~ ~ o p n c r c ; i ~ o ~ i ~ ~ ~ h ~ c.r).thmqtc srdirncntation rate

line nt-edlp aqpii-arion rreponemal antihndv absurp~ion ETA-ABS fllloresr~nt


glorncn~lar filtration rarr g m c t PI lionnnne qa5t r-ointestinal gran~~locvtr macropIiage ri>Eonvstimulating fartor

F 1 : c GFR

GM
GI

GM-(:SF
GU =A\'

IIcC. I-IEEKT' Hn'


HJA

genitourinary hepi~ti tis A virtis hl~rnan chnrionic g o u a d o ~ ( ~ p h i n head. FVFS. cars. IIOSC. and throat
1111rnanleukotrvtc anrigen histury of prcsent i Ilnesr hcan ratr hnm;in r;lI~ic-ui m m u n e globulin hereditary sphei-ocytosis idrn~ificationand chief complaint iizs~tlit~-dg~encl~n I rliahetes nlellitus immunoglobulin ~tctor insulin-li ke p t ) w l l ~ F

1-1PI HR HRlG HS

In/cc
JI3DM

k
IGF
114

.FT
KLT I 3 LDH LES LlT\ L I'
tAV

laclate dehwlroq-vase lower csophagc;ll sphincttr lives f~tnclion r~sts

LvI 1
Lvtev

hICI-IC

MClr hIEN

elecu-oh~es m e a l corp~ircz~lar hemnglohin concentration meart corpllsculal- v o i ~ ~ l n r t1111l tiple endriciine neoplaria

non no clonal gammopar hy n C ~illdetcrmined si~nificance majclr histocon~patihility complex

MR KHL h'IDDM NPO N SAI 11 PA PIP PBS PE


PFTs

I'M1 PMN
PT

PTTA PTH

m
PUD R13C RPR RR

RS
RV Rk' H

SBFT liMH SLE S'I'D TFT.; 1PA TS H TIBC


TIPS

magnetic resonanre (iniagingj non-Hodgkin's lymphoma non-irisutin-dep~tlrlentdiahctes nlelli~us nil p e w 5 (nothing by rnozith) uonsteroidal anti-in flarn~na~o? dnrg pus trroanlerior prnxirn;il interphalangeal peript~evalblond slnear pl-lysical cxam puImonarl; fi~rlc tioit tests point nf maxi ma1 intellsjrv polymorphonuclear leukocy t r protl~~tlornbin timu pcrcutaneuus rr-ansl~~m inal arlpjap1a.i~ pnrxthyroid hormonc partial thromhoplastin time ppptir: ulcer diseasc recl bloocl cell rapid plaq~na reagin respiratory rate Reed-Stcrnbctg (cell) rig11r ventricular right vcntrictrlar tivpertl-nphv small t > o ~ ~ follow-througli el syndrome or inappropriale secret ion of . D H systemic I ~ r p r crythamatosu~ ~s sexually tmnsmir ted diseasc ~liyroid r~mction tests thyroid-stin~r~lating hormone ioral ir.ui~-binrling capacity tranqjuplar intrahepatic porto~sternic shunt

TPO TSH TTP UA UGl


Z T S

thrombotic tl~rornhncyr opetlic purpurd

urinalysis upper GI ultrasound

VDRL VS VT N C
MTM? XR

Venereal Di3ease Research Laborittory vital signs ventricular rachycardia white hloud cell C$'olTf-Pat-fin~on-U?litc (syndrome)
x-ray

ID/CC

A 4%year+Icl male with a history of hypertension is brought by ambulance to the emergen? room because of the dcveloptnent o f sudden sharp, tearing, intractable Ieft chest pain with radiation to the back.
When he first arrives, lle shows a declining level or conscious nrss. h e c o m ~ s pde and short of breath (DYSPNEA), has decreased w i n e output ( ~ L I G U R I A )and , is unable to move his lefi arm and

HPI

Icg; st]hseqclen tfy he Taints


PE

(SWCOPK).

VS: marked hypotension (BP 90/50) in left arm, with significailtly h~ghtrr reading it) right arm (BP 170/PO). PE: pallor; cyanosis; diaphoresis: irldistinct 1lea1.tsounrls; aertic regurgitation murmur (high-pi tchcd, blowing, diastolic decrescendo murmur) ; inspiratory crackles at lung bases hilatel-allv (clne 20 pulmonarv cduma); anuria (due to decreased r ~ n a perfuston); l leftsided herniplegia.
ECG: 1 1 ~ e\4dt=ncc ) of myocardial infarct. CT/MR: spiraling intima1 flap with true and false lumen (DOLIBLF-~ARRE~. .M,IZTA). Aigio, aorrographv: con fir ma tot.^. CXR: mediastinal widening {due to I1crno1-rhage) .
Longitudinal separation of imnica media nf aortic wall,

B <

Labs

Imaging

Gross Pathology

Treatment

ICU monitoring for shock: ailtihyptrte~lsiveagetlts to decruasu vascular shear forces (avoid arteriolar dilators sltcl~ as hydralaziile): surgiraI correction.
,\ortic clissectiorl is a life-threatening conditior~ requiring immediate r t-eatment. Prcdisposing factors include hypertension and cnnnectivc tissue diseases (cystic mcdiaI d e p e n tation ~ as in Marfan's syndr-ome): conlplicatiorls include rupture and extension. Sudden death rnay occur with pericardial tamponade o r extension of dissection into coronary arteries.

Discussion

Atlas Lhks

t ' B P " l ' I PG-PI-001, PM-PI-001

r)"l A O R T I C DISSECTION

ID/CC

waq diagnosed with Marfan's syndrome more than 20 years age recently deveInprd severe shortness of breath.

X 31-vcar-olcl white male who

HPI

He rlenies smoking 01drinking and claims to have laad no major illncsscs in thc past.
'ITS: pulse bounding, large i n voIume, and collaping {~v,u-FR-

PE

COKKICAV'S ITILSE) . producing wide pulse pressure wirli mpid rise atlcl fall. PE: 40 Tt, higll-pitchecl, hlowing diastolic decrescendo murmur heard best at left sternal border with paticnt leaning forward and in expiration: diastolic nlurrniir heard when femoral artery conlpresserI with stetli~scopc
~ C ~ M M EOK K

Labs

ECG: tcft r r n trictilar Ilypcrtrophy (LVW)

Imaging Gross Pathology

GXR: lefr ventricular dilatation. Echo: L\W; Dopplei. confirmatorv.


Car~secl hy rlerect oFaortic valves or roots that lead\ to reg~l-gilarion of I,lood from aorta into left ventricle.

Treatment

Sl~rgical prosthetic valve replacement Tor svmptomatic patients or fur asvmptomatic patien LS with LL' cl\sf~~rlction. For symptomatic patients will1 normal 1.1' funcrion, diuretics or afterloadrrctticing drug\ tnav be beneficial. Antibiotic proplrvlrtxis against inIecti\,c-en~locardi~is hefor-c undergoing surgical or dcntal proccrlurc-s.
Conlmon causes or aortic i n ~ ~ i f f i c i c ~ include ~ c y congen j tnl bicur pirl cntvc, infec ti1.c endoc,~rdi ~ i u an . tl hvperrension: less crlinmon cause? inrlurle rf~etlmaric heart disease and aortic root discases (u.~..M a r b n ' s smdromc, ankvlosing spondyIi is, Reiter's ynrll-orne. tertiary yphilis).

Discussion

ID/CC

A 24year4ld Inan complain5 of easy fatigability, dyspnea on milcl exertion, and angina.

HP1

H e d s n admits to having occasionaI spclls of lightheadedness and fainting while playing hasketball.
Crescenddecrescendo systolic ejection murmur to right of sternum and radiating to neck: sort S2 wilh paradoxical splitting (d~~ to r ,pulnlotrary valve c l o s ~ ~prrceding re aortic wive closure); weak ancl delayed ("parnrs ct lardus'') rnrotid pulses.
ECG: left runtriculax. hypertrophy.

PE

Labs

Imaging

CXR: caIcificaLirlnson valvc leaflets and enIargcd cardiac shadow (due to largo lefi ventricle) . Echo: pr-esence of bicuspid aortic l-aIve. Conge~enilal bicuspid valve tvi th calcificatio~~ .
Uallnon valrrlilnpla~~; sur$cal prosthetic replacement or charlging o l normal pulnior~ary valve to aor~ic site and i n s e ~ - ~ i o of n pulmonary proslliesiq (ROSSmocenu~l;.) : antibiotic prophylaxis with penicillin prior to surgical or dental proced t~res.
C a u ~ of e ~aortic slenoqis incrude congenital bicuspid aortic valve (more coIrlm oir in males), progressive degenerative calcification o f normal valves (inoru ccl~nrnnnin elderly males) , and rhelumatic heart disease (the mitral valve i q a190 involvc-d in 95% uf inditiciduals with rheumatic disease of the aortic valve).
PG-PI-003

Gross Pathology
Treatment

Discussion

Atlas Link

h 59-year-old white male complains uf pain in the calf muscles


cluring exercise ( c ~ u n 1 ~ 4 m o along N ) with coldness and nutnbnrss i r ~ both legs: his symptoms have heen occurrirlg for a year and are relieved by rest.

'The patient has also been impotent and h a s been experiencing a hrlominal pain (due to mescn tcric ischemia) about half m hour after eating ( P C ~ T R I ~ N U WPL , u ~ ' ) .He smokes two packs uf cigare ties a day.
VS: hypertenqion (BP 150/100). PE: diminished peripheral pulses bilaterally; Toss of hair over dorsurn of feet and hands; clerroasecl relnperaturc in hands and feet; carotid and femoral

arterial bruits; atrophv of calf muscles.

Labs

Elevated LDL md decreased HDL: elevated total set-unn cholesterol.

Imaging

Angir~: multiple large atheromatous plaques in aortailiac distribution.XR, plain: irregular arterial vascular calcificatinns. LJS,Doppler: high-velocity postsrenotic flow jet.

Grass Pathology

Early: fatiy streak in suberldorhelillm; latc: fibrofatty plaque formation with clysnrophic calrification (atheroma) rz,ith narrowing uf Illmen of vessel wall.
Elrly: foam cells with intimal proliferation of'smooth muscle cells: late: smooth musclc cells synthesise collagen and form fibrous cap with necrotic lipid core and fibrous p'taquc.

Micro Pathology

Treatment

EXCI-C~SL'; smoking cessation; Iuw-closr aspirin; can pol of hypertensior~; cholestcrol-lrvwering drugs ( e . g . ,Ir~viistatin); angioplasty; coronary sten ting: coronary artery bypaqs grafting
(TAG).

Discussion

Atherosclrrosis is Il-ie main causr of coronarv artery disease and the leading cause of mortality in the United States. Plaques are commonly found in t h e abdominal aorta, coronary arteries, popliteal artrries, descencling thomcic aorta, internal carotid arteries. and cil-cle of Willis artrries. Thus. thrv are responsible for aor~ic aneurysms, CAD, peripheral vascular disease, intestinal angina, renov;lscular hypertension. and cr.rebrovasclilar disease.

Atlas Links

P
,
IS

ATHEROSCLEROSIS

IDJCC

A 47-y~ar-cllrlman complains of occasional palpitations and shortness of breath.

HPI

He also savs that lie occadonally experiences mild dizziness and chest rliscon~fort.
1%: irregularly irregular puisr. PE: loss of a waves in jugular pul~t,; ~arialllr-i:~t~~~~iv SI with orrasio~laI SS.
ECG: varidde ventrici~lar raw (PO lo 200); can he > 200 wit11 wide QRS i T associated with accessory pathway: no discernible P waves seen. hrurmalCK-MB.
C;Xk tlol-mal. Echo: enlarger1 I r f t arr-it~m.

PE

VPTIOUS

Labs

Imaging
Treatment

Beta-blockers; calcium channel blockers; digitalis (to decrrasc conduciio~l at ,4Lr tlihde in order to prevent vci-t~ric~zkir arrhvt 11mias): chemical cardiovcrsiorl with class U, TC. or III antiarrhythmic5 to carlvert hack to sirl~ls rhytlltn if pazien t remains svrnptotnatic; electrical cardioversion (if associaterl uith venti-ic~llar tarhyrartlia): patients sho111dalso he anticoagulated with warfarin to prevent en~tmlic disease.
Atrial fibrillation, the innst common chronic arrhythrriia. is nssociattd with a high risk of embolic disease. Causes inctitde drugs, tnitral valve diqease, Ilr,pertetlsive anrl iqrlwrnic Iieart clisc:tst.. dilated cardiomvopathy alcoholism. hyperthyroidism. pericarcli tis. pulmo~rary embolism, esercise, atrial septa1 defect, chronic lung disea~c, and cardiac surgery. Tt rnav ;hso he irliopathic.

Discussion

ATRIAL FIBRILLATION

A 50-year-old woman complains of recr~rl-cnt. kansient losses of conscio~~snex~ (sm~:orf:) ancl dizziness.
For the past fcw months she has had contin~rous mild to moderatc fever. fatipe, sweating, and joint pains (ART~IR.ILC;IAS) and has cxpcrien~rcl unexplainable breathlessness at rest ((epistwlic pl~l~nnn:~rv eclcma) that is relieved in a supine position and exacerbated by standing. She also cornplainc of significant weight loss over the past year.

VS: mild fever, PF: pallor and clubbing; on auscultatiorz. S1 delayed and decreaqed in intensity anrl cl~aractcristi c low-pitched sound ( I umor plop) during early diastole, followetl bv a rumbIe; auscultatory findings vary wiEh body position.
Labs

CRC: normorhromic. n o r r n o c y ~ c ailernia. FJevatecl ESR, increased I@; hlnocl culture? sterile. ECG: sinus rhythm.
Echo (2D): characleristir echo-producing mass in left atrium. MR. cardiac: globular maw in lrfi a1 ritlin.
Single g l o h ~ ~ lIel't a r atrial mass about 6 cm in diameter. peduncular~d with fihrovascular stalk arising fi-om interatrial septum in vicinity of fossa ovalis (favored site nl' atrial orisin).

Imaging

Gross Pathology

Micro Pathology

Stellate, multipotential rnesenchymal myxoma cells mixed with enrlotheIid cells; malure and immature smooth muscle cells and niacroph:tges, all in an acid mucopolysaccharide matrix.

Treatment

Surgical excision using cardiopulmonary- b y a s s i s cura rive.

Discussion

?'he most common type of primary cardiac tumors, myxomas may he IocatecF in any of the four chambers or, ral-eEy in the valves. 'I'hey arr predominantly atrial with a 4 1 left-to-right ratio anrl ill-u usually single. Their- signs and symptoms are closely rclntcd to their 1oc;lrion and In the patient's positinr~. Allhnugh mvxomas ar-e benign, the!: r a n emholizc, res~lltir~g in tnetastatic tlisease. Alrhnugh most myxorniiu are sporadic, some arc Fdmilial ~ 5 ) i t I autwornal-dominant l transmission; thw s, ecllouardiographic screening of filst-degree relatives is appropriale.

Atlas Link

ID/CC

A 7S-pcar-old whitc male is llrought into the emergency room with nausea, dypnea, ant1 a crushing s~~bsternal chest pain that radiates to I l i v left arm and jaw: the pain has lasted For al>our 30 minutcs and is n o t rc1ievc.d with rest.

HPI

One sublingual i~itroglycerinltahlct did not relieve his pin. His lliqtor) reveal5 a sedentary lifestyle, moderate hyercholes trrolemia. and obesity. The patie~r t is a h a diabetic and smokes.
\IS: I~ypotension.PE: diaphoresis.

>

r)

PE
Labs

ECG: ST elevation with pcaking of T wiivcs; suhserl~i~n t develop m r n r of inverted T waves and permanent Q waves. Later; S T * anrl T waves nor~nalixe. Elevated CK-MB: elevated troponin T and I. CRC: I e ~ t k o n ~ o s i s .
Ecllo: decreased wall motion
(HYP~KWFSIS).

Imaging
Gross Pathotogy

1 2 hour-9: n o myocarrtial clamage; 24 hours: pallor cIuc tu coagulation nccl-mis or r ~ d d i ~ moltIinq; h 3 to 5 days: dcinarcatcd vetli~~ region v w-i tlr hvp.pc.rr.mic hor dcr; "eel 3 wceks: ?oft, grlat itlol~s; 1 to 0 months: white %carancl firm, lhin wall.

Micro Pathology

12 to I X hnur';: tli~clcarpyknori9, coagulation necrosis, and eosiiinpf~ilia: P rn 3 days: in tense neutrnphilic infiltrate, loss nf nuclei and cross-striations: 1 M Y - C ~ :disappc-arance oFPMNs. onsct c~f fibrohEastit rrpair; 3 wreku: granulation ~ i s v t e with p r t ) g r ~ ~ fi ~ hrr)si~. it'~
Oxvgcn, bcd rest. aspirin. pain relicf will1 morphine. nitrates, bcta-klockerq: plaque s~abilizalion with I~eparin, anti-Gp IIa-ITTh rnonoclon;il atltihodi es; thromkol~uis with I I'A iif cardiac rarhe~erizatinnis not immerTiately ;lvailahlc: cardiac cathrtrsization with angiopl;isty or s~trgicalreperfusion with a b y a s s ~ a f depending i nn 13;ltlircor diseaqe; ACE inhibitors (limit postinfarct re~nocleIing) atid cholesterol-lo~r~ring dr~tgs.
171e mo5t common cause of invocardial infarction i.i alhernsclerusi5 (coror~a~-!j artery c!ise;-lsr): i t i s Iew commonly c a u ~ e rIlr r ~ coro1i;~r-y vasocpavn (Prin7rnetal's angitla). Sequrlar incliic-le arrhyrhinins. congestive hcart failure, pulmonary cdernn. shock, p~~Irnonarv cmhoIi<m, papillarv muscle rupture, ventricular aricul-ysin.ventric~tla~. wall rupturr, ta~nponaclc, i111dautoinlnlunu fil~rirtous prricarditis (DRESSLER'S SWDROM~,).

Treatment

Discussion

Adas Links

I I I 7 -'D 1 PG-PI-007, PM-PI-007

"

CAD-MYOCARDIAL

INFARCTION

ID/CC

A 50-war-old male who MQS admilled lo the CCU 3 days ago fnl Inwing an MI presents with hypotension.
Thu patient was tkromholyzed post-MT and was recovering well. H e also ron-lplained of a nlild fever l ~ u n t o chilIs or rigors.

HPI

PE

VS: tachycardia; weak. thready pulse; tach\pnea: hypotension. PE: parlor; cool. moist skin: mild cyarlosis nf lips anrl digits: > I O-mmHg fall in arterial pr-essln-ewit11 inspiration (P~;LSUS
P,+RADO?CUS) ; heart sounds

muffled and JVP elevated; lu~lgs clear

bilare~.alle
Labs

Elevated cardiac enzymes (CK-kIB, tl-oponitl) as a result of recent acute Mr.

Imaging

Fcho: diasrolir compresion of the righr ventricle; pericardial


eff~~sioi~.

Cross Pathology
Micro Pathology

RIipt itre of ihe left ven tl-iculal-wall with humopericardium.


Isc hemir coap~lative necrosis of the affecled myocardium, co~~sisting of mu1tiple erythrocytes and dear!, anucleated
tnvoryrer.

Treatment

Emergency pesicardiocen tesis; tt-ear ~Iiock by infusing fluid and isoprn~erei~ctl; surgical ]-?pair of cardiac rupturr subsequent !o st~hilizatinn.
Cardiac rupture most typically develops 3 l o 10 davs after the initial onset of the infarction seco11da1.y to rupture of necrotic cardiac musclc: thcrc is usually little warning tlefore the sudden collapse, which is associaied with acute cardiac tampmade and electromechanical dissociation. Papillary rnuscle rt lpture rnav also occur following an acllte MI, resulring in ~ l i t r art'qirgital

Discussion

Ij

'

CARDIAC TAMPONADE

IDJCC

A 60-year-old male presents


evaIuation.

tn a clinic

for a heart transplant

HPI

Tlic patient was diiapiosecl lag! v e x with claw I11 {mnrkerl limitation nT activity; comfnrtable o111y at rest) congestive heart faifure ~econdary to idiopathic dilated cardiomyopathy. He is ctn'ren~ly being treated w i ~ h cligoxin, r~lrospmirle (diuretic), lisi nopl-il (ACE inhibitor), and wal-hriir (anticoa~llant) hut torltirlues Lr, he s~mplonlatic.
VS: normill. PE: ~ 1 ~ ~ a t e d JS3/S4 T; gi~llop Iwat-d (311 a u s c ~ ~ l ~ a lion: significant pitting lower extl-emirv edema.

9 m
0

r 0

PE

rn

<

Labs

CRC:/Lytes: Normal. TFTs, I,FTF, total protein, alhurnin, uric acid. and PQho~lr pmtein/creatininc normal: PSA normal: IgG and IgM antibody titers a p i n ~ CVV. t HHS HI\:, V%V, hepatitis B ancl C, and rouoplaqn~orisr~egalive; PT/PTT/Ih'R normal.
Echo: EF 15%with moderatc mitral t-dvr rrgurgitation. CXR: cmdiorne_@y. ECG: ncca~ir~nal premature ventrictrlar contractions (PVCs). TZiallium scan: exercise-ind~~ced global cardiac ischemia.

Imaging

Treatment

If npprrwd as a viable transpla~ltcaridida~e. the pnrient mllst wait lor a ruihhle donor (tnatclled according to btldv size, weight, ARO blood p z ~ p i n gand , levels of panel reactive antibody. o r PRA). r-lboul 24% of patients waiting fnr a cardiac tran5plant (CT)rlie b e f i ~ r a e donor cat1 he fcninrl. Moct carrlii~rtransplant?

;we orthotopic.
Discussion Gat-rliac tranqplairtation accollnts Tor 14% or nrgan transpla~~t prnrerltlrel; and can dran~atically improve ciirdiac f i i ~ ~ c t i o in r~ inrli\;rduxlswith end-stage cardi:tc di~eiisc. Patirntq murt Iiaw New York Heart Association (NYIU) class I I I or IV congestive heart failure, having hilrd rnmimlrm rnedical thet~pv arld otl-ier tlltrapcutit inlerventio~lr sc~clias I3CT/\ for CAD. C ~ ~ I - T P ~ I ~ I V , ischemic heart disease account5 I'or appl-oximately:55% OF cnllses secjitit-ing CT and idiopathic cadiomyopathy for roughly 4W%.

"

C A R D I A C TRANSPLANT

A ti5-yrar-old whitc malc complain5 of requiring three paows in bed in order to breathe comfortably ( O R T H O I ~ K E , ~ and ) hajing rn open 1 h e wiiidow to gasp for air at night ( P . ~ R C S ? E J ~ \n~' A 0C I .r t t l R ~ 1 . V ~
~I~PVT.A).

Hc has alsu nutt-d increaqing shorhess of breath while walking at: well as swellins of his ankles and legs. JJP had a myocardial inrarction 2 y a l - s ago and has a histosv of chronic hypertension.
VS: ~arhvcarriia: tac hypnea; weak, thready pulse. PE: cen cral wanosis; distention o f neck veins ( ~ I L ~n P e l e v a t e d - m ) ; third heart so~md: grade 11I/\rl rrescendr, aortic s?lstolic murmur: crepitant r a l ~ over s 110th lower lobes; lower lung field.; dull to percussion hilat erallv; tender hrpatornegaly; 4+ pitting edema in both lower extremities: coId extrernitieq.

Labs

~BGF hvpoxernia: : low cardiac output a< measured 1 3 Eck ~ equation a i d Swan-Gal17 ratheter (2.4 L/min): trarisudale in pleural

flrlid; i n r r e a s ~ d DUN. ECG: left vc=ntricularhyertl-nphy. Imaging

CXR: enlarged cardiac sillln~tette: bilateral pleur-a1 eff~iqions and diffilw increased lung markings {KERI.EY I 3 L I N F , ~ )suggesting pulrnnnrrry edema. Echo: ejection fraction o f 40%.

Gross Pathology

C;lrdiomegaly r l 1 1 ~ to hot11 dilatation and hypertr.ophy; pulmonary edema with increase in weight and reddish-purple cnlnr: nutmeg liver (clue to chronic pasdve congestion).
Hepati;l,a~ion of Itlngs with alveolar capil1ar.y cc)nge:~ion and alveolar macrophagrs with hemnsiderin ( "IILLK~ F,:.ULUR.E <:ELLS") ; ceri trilobt~lalliver congestion.
Diuretics: low-sodium diet, digoxin; ACE inhibitors: nitratus: ail ri;lrrIivthmic~.
Congcstiw heart failure (C:HF) is heart failurc due to a deficit in myocardial strength rtr to an increase in workload. CHF is a ccjmman cnrnplicatiorl of ischemic and h y p ~ r ~ e i l ~heart ive clisease in olcler populations.

Micro Pathology

Treatment

Discussion

Atlas Links

C O N G E S T I V E HEART F A I L U R E

ID/CC

A SC-war-old fcm:~lc Asian immigrant complains of weakness. shortness of breath on exertion, a11d w e W i of both feet.
Slle itlso complains of propessive abdominal distention and Catip~e. She was i.rrate~l for-p~llmonary tuberdosis a few yeat-q

HP1

5
PE

13:mild llvpotension: reduced pulse pressnre. PE: peripheral


yanosic ancl cold cxtrenriries; paklnr; neck veins di5tencIecl;JW inmaws during inspiration ( K l i ~ ~ h 7 4 t ~ Sir l~ ,V) '% ; pedal edema; modera~e hepaiornegalv, splenornegalv. a t ~ d ascites: reduced-intensityapical impulse, distant heart soundq. and early third heart sound (pericmdial knock): un pulst~% paradoxl~s: ilo rn~ri-rn in- or T rth B~ea!-rl.

g
0

m
4

Labs

ECC:: low-voltage QRS cornplexcs with fl;ttlening of T wa1.e (nonqpecific).LFrs tnilrlly ahnot-nlal (due to tlepatic co11g.e.rtion) : asci tic f l ~ ~ i transudative rl (low protein. high sngar) . LA: protcinuria. n o casts.

Imaging

CXR: fibrosis (old Ilealerl tr~hrrculocis) : heart sharlow shows signs of pericardid calcification. Echo: pericardial thickening. CT: pericardial calcification and thickening.
Thick, dcnsr. fibrous obliteration of pericardial space with calcification encasing t h c heart and limiting diastolic frlling. Complete pericitrdial resection is the only dcfinitivr trratmen t; in\tiu~te antituberculous therapy when a p p r n p r i a t ~ diurt-tics; : 5odi11m restriction; digiiaIis for asmciatc.rl atrial fibrillatian (in one-third nf patietl~q). T h e etic>logvor constr-ictive pericarditis lies in the Cnrznation or sml- t i s ~ u e that cncascs the hcixrt and intcl-feres with ventricul;~t. filling. Tuberculosis is the most cominon causc worldwide. Mosr cases n o w sccn in t h e United States arc idiopathic. h11t cases r-ewli i t ~ j fr-nm i pxposllre ro radiation, t n u r n a , cardiac srirgety, rheumatoid arthritis. ur uremia 1 3 2 1 ~ become ~ more cornmot).

Gross Pathology

Treatment

Discussion

CONSTRICTIVE P E R I C A R D I T I S

ID/CC A 6O-year-old white male who has heen treated for COPD comes
to the enlergeilcy room with sevcre dyspnea at rest.

HPI

Over t h e past few months. tho paticnt has noted an incmaqed productive cough ancl exertional dypnea. H e arlmitq to I~eing a heavy smoker and failed to quit smoking even after the appearance of symptoms and the diagnosis of COPD.

PE

ElevatedJVP with Eal-KP a and v waves; loud P2; cvar~usjs; cluhhi~lg of fingers: hilatcral whce7irlg; rxpir-atary rhenchi; prolor~gedexpiria inn: use or accesqory rnusclus of'respiration; left para"ertia1 heave: ankle and s a d edema; tender hepatomegaly.
EGG: ri~ht-axis deviation a n d peaked P waves ( P rr TI.MOK.;~I.E) PlTs: COPD pallern.
CXK: right venrricnlar and pulmonary artery enlargernenl; hyperinflation.

Labs

Imaging

Gross Pathology
Treatment Discussion

Kightventricnlar h ? ~ ~ r t s o p h v .
Oxygen; salt and watcr rcstrictio~~: treatnleni c)f COPD.

Cnr pulmonaEc is right heart f d u r e due to a pulmonary cause. most comrno111yCOPD. Other causes x u epulmnnarv fibrosis, ~~ieumocorlioses, t-ccurrenr pulmonary enlbolism, primary pr~lmonary hypertension. ohcsjty with sleep apnea, cystic fibrosis,

COR PULMONALE

I D/CC

A 29-year-olrf female who recently gave birth to a hcalthy infant rlevelops dyspnea and swelling o f her Feet toward the rnd of the day.

HPI
AE

She is til~rsing her &week-old child.


LrS: RP miltlly elevated. PE:JVP raised with prominent a and v waves; tender, mild hepatosp3enornegaly; cardiac apex hearing and displaced o t ~ ~ s i d midclavicular e line; pansystolic apical murmur (due to m i d i n ~ ~ c i e n c and y ) systolic murmur increasing 114th inspiration heard in tricuspid area (due to tricuspid insufficiency) : lollrl puImonary component of S2; SS and S4 gallop; fine inspirator). l>asal crcpitan t ralcs at both lung basts; pedal edema.

Labs

ECG: premature ventricular con tractions.

Imaging

CXR: interstitial pulmonary edema (due to severe p~~linanary


venous hypertension) ; global d o m e g a l y . Echo/Nuc:

cardiomegaly wit11 diminished ventricular contractility (systolic dysfunction). Stress test: decreased ejection fraction with stress (e,jection fraction no t-mallv increases with stress).
Gross Pathology
Micro Pathology

Glol>al dilatation of all chambers.


Extensive fibrosis without active inflammation on endocardial

l>iopsy.
Treatment

Cardiac failure [reat ed with salt restriction, diuretics. vasoclila tors, ACE inhibitors, ancl digoxin: chronic anticoagulation: nutritional supp1ernentation; considcr cardiac transplant if medical llierapy rails.
Dilated cardiornyopathy usually develops in the peripartum period (t3 months). Other rtiologie~ incIudt. alcoholism ( d ~to l thiamine ~ deficier~cy or direct toxicity), hypoth!?-oidism, Friedreich's alaxia, preiriniir,myocarditis (usuaIly due to coxsackie B) , tnyotnnic dvszrophy, chronic hypocalcemia or hypophosphaternia, sarcoidosis, and drug toxici~iec; (e.~., adriarnycin, q c l ophosphamide, tricyclic antidepressants. lithium. and cobalt).
I ' I,-- 1311 PG-PI-013

Discussion

Atlas Link

DILATED CARDIOMYOPATHY

IDJCC

-4 23-war-old wornan is seen with complaints of excessive breathlessness, palpitations, fatigue, blood-streaked sputum (MILE- HFM<)I~TYSIS), and swelling of the feet (ET>F.MA).
She was diagnosed ~ vth i ven~wlar septal defect (I'SD) at birth, t111t her patents had refused qurgery.

HPI

PE

VS: HR. BI' normal; mild tacbvpnea; n o fever. PE: central cyanosis; c l u h h i n g ; . v no~+mal: left parasternal hrave; P2 palpable: sing!e second heart sound, predominantly loud P', (due tn pulmonary hypertension); pansvstolic mnrmur along left sternal edge; qjcction ytolic murmur in pulmonary area; mid4iastolic murmur (GK,~I-IA~I STELLL M CTRMCI K UI: PCI LMOY.~RY R E C L ~ R C I T ~ O N ) i n pulmonary area that increased with inspiration (CARVAI .I.O'S
SECTN, IYIIICMLNG RIGHT-SILIED MI'RMUR).

Labs

ARC;$: h ~ e r c a p n i ahpoxia, , and partIv cornpensa~edrespiratory acirlo~is. CBC: polycythemia. ECG: right ventricular hypertrophy with right-axis deviation. Cardiac cathercrizar ion reveab right-tc-left shut-~t, pulmonary arterial hypertension. and pulrnunar? regurgitation.

Imaging

Echo (with DoppIer): VSD with right-to-leftsystolic shunt; right \c~~tric~i enlar~ernen lar t and hypertrophy. CXR: plzlmonary oligemia ("peripheral pruning") and greatly enlarged hilat p ~ ~ l r n o n aartcry rv shadows;carcliomegaly.

Treatment

Heart-lung transplantation; surgical correction of' a VSD is irleallv performed before irreversible ptilrnonarv vascular changes vet in.
The term "F,isenn~~nger's syndronw" applieq to tllose defects in which pulmonary vaqcular disezqe causes right-tm-leftshunt of blood; Eisenrnenger's complex is right-to-lcft shunt due l oa lurxc- VSD. The riqk o f ' infective endocardiris is high; therefore, a n timicrobial pmp'tiplaxis is mandatory. Pregnancy is contrainclicated owing to a high maternal mortality rate.

Discussion

yc* EISENMENCER'S COMPLEX

ID/CC

,I20-vear-old college s t l d e n ~ i q Iwought hack From a summer


r a m p in the mountains after clcvcloping severe shortness of

breath ( ~ W I ~ N F ~ J crlngI1 I). with blood-tinged sputum ( I ~ E M O I ~ ~ ; I S ) , ancl wheezing.

HPI

The group had ascended to a height of 8,000 feet and I~ad engaged in strenuous physical activities. Tht patien 1 albsequently developed dyspnea and cough that !vrorsenedd11rit ~ the g night, leacling to marked respiratory distress and a shock-like state.

'

m
I--

2
0 0

PE

13:wchvcardia: t a c h ~ r l e ahypotensic>n. : PE: central cyanosis; pair anti coFcI eatremi lies; marker1 respiratory distress; widespread aales and rhonchi ovcr both ltulg ficlrls.
CBC: elevated hernat~crit and hemoglobin: ~nilrlly increased U%C. ABGs: markedlv decreased arterial Po, (hypoxia): low Pco?.Increaqed pH (respiratory alkalosis) . EGG: s i i ~ r tachycal~s dia with acute puhonary hypertension.

<

Labs

Imaging

CXR (PA view) : noncardiogenic pulmonary edema and prominent matrl p~ilmonar-v artery.
Extensive pn11nr)nat-v edema: pror~in-richexnclate with alveolar hen-tat.rhnges and alveofar hyaline membranes.
Prompt descent, hyperbaric oxygen inhalation, sublingvd nifedipine (after checking blood pressure),ancl placcrnrrl t in partalde hyperbaric chamber while being tranupot-ted; hospitid management consists of continuous high-flow oxy-gen,dexamethasone For CNS symptoms, and acetazoldde.
High-altitucle pulrnorras~ederna i s pt-imnrily a clisosder of the pt~lmn~inry circulatio~l induced by sustained alveolar hypoxia. Thp initiating event is an al~nnrmal deg-ee of hypoxia-induced pulmonary arteriolar (precapillary) constriction ( hvpoxia causes dilatation of sy.;lr.rtlic Mood veweIs) ~ l i a t elevates pulmonary arterial pressure. Thc i~tlbalancr of increased blood Row and pressure allows fluid to leave the pulmonary v~qr~dature, resulting in edema.

Micro Pathology

Treatment

Discussion

HIGH-ALTITUDE S I C K N E S S

ID/C6

A 21-ycarsld white male presents with anginal chesr pain, + p e a on exertion, and an episodr of syncope while playing

basketball.

HPI

Thc patirnt has ntj history of hllte spells. squatting for relief, o r rhe~tmatic I'ever in chilcll~ond.

PE \%: pulse hisferious (norr~r.~. I ~ I c ~ ~ K FP .~ E) .: , P normal; cardiac


ap?x forceful with strong premrsto!ic impulse ( 1 3 0 1 ' .~4~ PlW I M P L I S ~ ): systolic tllt-ill palpable orcr left stcrn;ll hord er: 54; ejection y t o l i c murmur over left 111 ird ~ I tercn~tal H ~ p x t'ildiate itlg ro has? anrl axilla; Inurmur. increased by exercise and dtaring forced expiration against a closed slottis (\':u~Kv.\ >LWELYER) I-n~tdeaeaqcd by squatting.

Labs

ECG: left-axis deviatiotl due to left ventricular hyperfrophy; Qwave ~xa~gemterl i n i n k rior and latrral precordial leacls (due t o septa1 hypertrnphy).
CXR, PA: often normal. Echo: asymmetrical septal hpertmphy and systolic anterior motion of rnitral valve; Doppler rnalrshow mitral regurgitation, Angin, cardiac: marked thickening of left ventridar septal waU; small ventric~ilar cal-ity with impairer1 ventricttlar filling (diasialic dl;rfimction) and narrow outflow tract ( "HC)CI KLLISF" IWI~I.A\K,LY(: :E) .
Enlarged heart with increased weight and aspmmetricaI septal hyperirophy.

Imaging

Gross Pathology

Micra Pathology
Treatment

Mvnryte disarray wid1 increaser! norepinephrine content.


Negative inotl-opic agents (c.g..beta-blockers) to decrease stiff: nus? of lcfi ventricle atlcl prevent FataI atrhyrhrninq: avoidance of competitive sports;amiodaronc ( ~ n a v hc u ~ f 1 1 in 1 p~+evention of' lothal cardiac arrhythmias) ; sur-~cal mvomectorny of intesvrntrictllilr s e p t u ~ n in parienw with ot~tflow ohsrruction.

Discussion

Also known as idiopathic hypertrophic subaortic stenosis (IHSS). An autosomal-dominant pattern of disease is noted in 50% of casts: ventricular outflow tract ohsaltction by phy produccs symptoms. The presen tirrg symptom in athletes rniql~r be sudden death seconrlal-y to lethal cardiac wrliythmias.

~~~~~~~o-

Atlas Link

mPG-PI-016

HYPERTROPHIC OBSTRUCTIVE CARDIOMYOPATHY

IDJCC

A 28-year-old wo~nan found on a park bench apparently dead is I>ro~~g;ht to the ER in the early hour5 of the morning. No rliscerni hle pulse was palpated. but a faint, infrequent respiratory effort was noted; CPR was hegut1 a n d c o n t i n ~ ~ e r l during I ~ e transport r to the hospital. The temperature ovwight was near-freezingwith contirl~tous rain.
VS: arterial pulse not palpated: trylotensinn: reduced respiratory rat?; severe hypothermia (< 28C). PE: no respiratory effort;Fixed and dilated pupils; blotch! areas of ervthrma on ski11;bullae over h ~ ~ t t o c k chest s ; exam shorvs diffuse rales bilaterallv: absent ho~vc-1 sounds; absent deep terldot~reflexes.

HPI

nXJ

PE

S 52
b G) <

Labs

CBC: iilcrcased hc-~natocrit. FIypogIycumia: increased BUN and creatinine. Lytes: rlecreased bical-honate; hyperkalcmia, ARGs: suvere rneraholic aciclosis. ECG: evidei~ce of marked bradycardia hirhd (Osborn) waves ( ~ ~ p w awave5 r d immediately f o l l o l n g the S wave). CXR:
atelectasis.

Imaging

Treatment

Intubalion and ventilation as neccssarv; cardiac massage for arrest; warm the patient rhrn~lgh use of a combination of heatcd hlankeis, heat packs, warm gastric lavage, wasm-~r;lrcr immersion, and high-flow oxygen; monilof- cardiac rhythm for arrhytl~mias.

Discussion

Hypothertnia is defined as core lemperatlire helow Sj0C;severe accidental h y p o t h e h a (below 30C, or 86F) is associated with marked depression in cerebral blood flow and cerebral oxygen requirement, reduced cardiac output, and decreased arterial pressure. Victims cat) appear to be lifeless as a result of nlarlced depression 01' hrain fcmctio1-i. Peripheral pulses may be difficlll l to detect tlerause of Aradycarilia and vasoconstriction. Cornplica~ions or W S F C I ~ ~ hypothermia C may irlclude ventricular fibrillation, pancrratitis, renal hil~tre, and coagulopa thy.

*
y e

HYPOTHERMIA

X 42-veal.-o!d l~lackmale presen is with chest pain, headache,

altered mental stattzs, and confusion.

He i u kiiclwti

to

havr labile essential hypertension. E T c has n o

history oCfcvcr.

VS: severe diastolic hypertension (RP 230/ 1:i0). PE: disoriented and confused;bilateral papilledema: 110 rocal nel~rologic clefici ts; r-e~r~aitr rlel- o f rsaln 1101-nlal.
Labs

CHI:: mirroangioparhic Ilcmolytic anemia. UA: hematuria and proteinuria. Increased BUN and serum rreatinine. ECG: Ieft ventricular hypertrophy. CT/US, ahcloniun: hilaterdl small and scarred kidneys.
Kidncv suri'acc appears "flea-bitten" (dtw
;I!-tc-rio1c.s and domert11;11ral~il1a1-ies).
to

Imaging Gross Pathology

rupture or cnrtical

Micro Pathology

Rcnal hir,ps~(not i-outinelv i~ldicated) shnwr hyperplastic arteriolosd~rosis ( " O K I ~ Ysw~\rrr;r;") nr i 11tel-lnh111ar artcl-ics with fibrinoid nemsis and thro~nhi in arterioles and small arteries; necrotizing domerulitis wit11 neulropl~il i n f l [ration alqn wen.
i\.' sodium nitmpmsside 01- IV hctal-blockers in cor!jut)clinn with ?+hour cardiac inonitoring in acuic. phase; s u l w e q ~ ~ e n managrz m e n t with nrnl antihypei-t~iiciveq anrl rtnphasis on d c t patient cornpl iarlre.

Treatment

Discussion

End+r.gan d a m y e cacrsed Iw malignartt hvpevtension incIutIes 11emol.rhagicand lacnnar strokes, encephalopathy, filndal hemorrl~ages, papilledcmi~, rnvocardii~l i s c h r ~ ~ ~ i a / i n C ~ r c rleft jan, ver~tr-ic~~larIi)r~ert~-nphy. C C I I I ~ ~ S ~llenrt ~ T C fdililr~rr, actile renal IhiEurc. nephru~clerosis. aortic dissection, and necrotizing vasculitis.

Atlas Links

&

MALIGNANT HYPERTENSION

lD/CC

Thc case of a 50-year-old man who died or 11Ieeding complications is discllssed at an autopsy meeting owing to peculiar vegetations seen on his mitral valve.

HPI

He underwent surgery for adenocarcinorna of the stomach. ShortIy I~efore his death he was diagnosed as having disseminated intnvascular coagulation (DIG); he subsequendy dicd of bleeding cornplicatinns.
Small (I- to 5-mm ) friable, sterile vegetations loosely adherent to A k a 1 valve leaflets along limes of dosure.

Gross Pathology

Micro Pathology

Vegetations round to be sterile fibrin and platelet thrombi loosely attachcd without evidence of inflammation (bland) or x-alve damage. Noilhacterial 1hrornhoti.c endocardi tis characteristicallvoccurs in settings of prolonged debilitating diseases such as cancer (particularly tirceral adenocarcinomas) , DIC, renal Failure, chronic sepsis, or other hypercoagulable states. Thc vegetations may produce emboli and suhsrql~en t inrarctions in the heart, kidneys, brain, mesentery. or extremities.

Discussion

MARANTIC ENDOCARDITIS

ID/CC

A 37-year-old white male complains of increasing fatigue and ghnrtness of breath during minimal physical exertion.

HPI

We denies having h a d anv ches! pain or having anv pt-evious


history of similar W p r o m s . A carefill historv reveals rheumatic fever at age 7.

PE

\'S: jerkv pulse (RWID UPSTROKE). PE: high-pitched pansystolic murmur at apex with radiation to axiwilla; 53.

Labs

EGG: left-axis deviation; left atrial ancl lefi ventricular


Ilypertrnptly.

Imaging

CXR/Echo: cnlargernent of left atrium and w n h c l e . Doppler:


cnnl:t-maror?.

Treatment

OrxI arteriolar vasodiIators (e.g.# ACE inhibitors, hydrala7ine) to improve for~mrd cardiac outpm; surgical repair or prostl~ctic ruplacemmt; antihioric prophylaxis with penicillin prior to surgical or dental praccrli~r~s.

Discussion

Co~nmon causes of m itral insufficiency include mikal valve prolapse, ischemic papillal-v muscle dysf~~nction. infrctive undocardi tis, hvpertrophir cardiornvnpatli y, vet11rirular enlargement, n ~ rat i i a n n d u s calcification, anrl dilatecl cardiomyopathy; rhe~~rnatic hcart discise i s nn longer rhe leading carisc.

MITRAL INSUFFICIENCY

ID/CC
HPI

A 94-y~ar-old white female in her 27th week of pregnancy is i~dmit~ecl to the hospital with dyspnea and orthopnea. Thc paticnt denies any prior c,udiovascular disease. hut a carrfiil history reveals that she ~uffered from sb.eptococd pharpngitis and rheumatic heart disease as a child.
Malar flush; e1evated.P (due to vennus congcstionj; loft parasternal heave: loud 81 ; opening snap;rumbling, low-pitched mid-diastolic murmur at apex hem-cl best in Left lateral position.
T:C:G: left atrial hypertrophy and/or atrial fibrillation.

PE

z E
0

7 -

2 <

Labs
Imaging

CXR double silhoz~ette due to enlarged left atrittn~; Kerlcy B lines {due to intcrjtitial edema). Echo: Ieaflet thickening with fusion of the comrnissures.
Thickened and scarred mitral valve.

Gross Pathology Treatment

Treat atrial fihrillatioi+~: anticoagulation, cornrni~sr~rnt~my, prosthetic valve replacement; antibintic prophylaxis prior to surTjcal or dental procedures; diuretics to relicvc pulmonary congestion.
The rr~ost common cause of rnilral stenosiq is rheumatic heart rliwaw. The main cl~anges to the vatve include leaflrt thickening, fusion of the comrnissures, and shortening, thickening, ancl f't~sio of the cotdae tendineae.

Discussion

MITRAL STENOSIS

A 1 1 lX-ye:~r.olrlwllitr male ronlplains of gsaclli:~llypmgr~s\ing shortness of breath and ankle swelling. His hynptcllns startetl f ~ ~ l l n w ia i~ URI. g I E e also ct)mplains of excessive fatigue and frequent chest pain. HP ha5 no histurv vf joirl~ pain, .;kin 1-;t.;li. or.ir~volittirnl.~ movements (vs. r h u ~ ~ r n a l i t r e ~ r r iurd ) is nrit her. Iivl~eriptir;ive t)o~. rliabctir.
\IS: tach!carclia; hipntcnslon: n o frve~, PE: elevated JW; pitting pcclal cdema; finr inspirator!. ralrs a i both lung h a w : inild cender hepalotnegaly: spler~c>meqaly; right-sided SS; murmurs of

mi tral regiirgitation.
Labs
titers not I-itiscd. CBC:: Iynpl~ocytosis. Elctrlred KSR,ECG: rust-cle-gree AV block with nonspecific ST-T changes. Coxsackievin~s isolalrcI on ph;~rvngeaIwafhings: i~lcrea~ccl titers o f srrllni ;a11 tiho(lie4 to cousackievi1.11~: clevated d i a c enzymes.

Imaging

CXR: c;lrdiomcg;~ly rmd pl~lmorlary rdrma. Echo: s11g-gestiveof dilaltrcl cardionlvop;tthy w i ~ h 'low qjecl inn rract inn.
Flabby, dilated hcart with foci of mvocardial prlrchial hetnorrIlagc.u.
Enclnmyorarrlial hinpsy revrals diffuse infihation by mononuclear cells, predominantly lymphocytes: i tltcrqtirial <*clrnja; focal ~nvrdil~rr necrosiy Focal lillr-osiq.
Rcst: sptcific an tinlicrohial thcmpy whcrl :tp~ropria tr; control of conxe:i ire cardiac Failure by rliurer ics. tligitaliq, and vasodilators: antiarrhvthmics if indicatrrl: cardiac transplilnt in in t i-;~~ti~l)le cases. A1 lhough lnrlst cases oi' ;ICIILP 111yaral-rli iis rnav resolve ~ p ~ t l t a ~ ~ ~sra~ ~t il ~ e ps m lg vr e , s 5 to cli1;~tcdcardiorr~yup;~ tliy.

Gross Pathology

Micro Pathology

Treatment

Discussion

The elir>logy or mvocar*ditisis ~ ~ s u a lcoxsackic lv I 3 or other viruses: less often implicaterl are hacrcria o r fungi. rickrr tqiae (u.g.. Rocky Mou~rtain spotted fever.). qpir.ndl~ces ( P . K . , L?me rliscase) , Try/)nnor.ornnw t r z E (Chagns' disease), 11)~ersensitivity d i ~ ~ i l(SIX, w dr-lig reaci i o t l ) . rarliation. and sarcoidr~sis, Diphtheria toxin also callscs m ~ o t ; ~ r d i tby i s inhibiting eltkaryotic elotigation factfir 2 (EF-2), th11s inhibiting lllyocytc protein ~yltIlesi<. l r mav aIw hc idiapathic. IJt~ung males arc primarily
affected.

MYOCARDITIS

ID/CC

11 R4vrar-old while ferri;~le corr~pli~iris of sudden-onset severe


pain in her lrrt leg wit11 awociated weakness or the lert Toot. T h e pain intensifies w h e n she mm-es her leg, ant1 clie cannot move her tors at a1I.

HPI

She is a smoker and has a hislory o f limited exercise tolerance rlll~ tn pain in her lower extremities ( I Y ~ E K ~ ~ I ~ E N T
(:LII'~II:,ITIOV).

PE

TS: rlor-rnai. PE: lipid depositior~in ski11 { ? L A N T H E L ~ . ~ \ ~ A % ) ;

~>c,plit~al. dorsalis p c d i ~;111tl , po~~rrim til~ial - pulses lost o n l ~ f t qidc: frmurst p~tlses easilv ~~alpahlc: lcft leg cold and mottled; anesihesta over lnlves Icfi Icg.

Labs

CBC: leu koqtosis.

Imaging

ITS,l h l ~ p l ~ 1 ol>strl~ctinn -: nC left Crmoral artcry at origi11.Anair): con fir-~natory: assess runoff and roltatcrals prior- 10 surgery.
T11n1nlholysi.i: consider rmholectoinv. . h e r i a l emholism mav have various c:ttlscs, s ~cll i ar: atrial fibrillation, myocardial infarction, prosthetic heart valves. etldocal-ditis, cancer, rlila ted car-cliotnvopatl~v., paradoxical embolism from the venou5 swtem, or a dislocIgerl mnral tlzrom hus Irom a n abdominal aortic anellrysm 01- an atheroma tous plaquc. Tlie earlier 1 he iiit~t-venrinn, T ~ I higher P the likc=lihoorl thaz t h e litnh may he saIvage~l. ClinicaIl~ c hararterized Iw the five P's: pain, palIor, paralysis, paresthesia, and pulselessness.

Treatment

Discussion

PERIPHERAL ARTERIAL EMBOLISM

ID/CC

A 25ycar-old male is brought to he ER after having wctained a sia1-1 ~ v o u n c l011 his left thig11 following a drunken hrawE. A iourniq~~er waq tier1 abmre the site, which rhc attc~~clants said was spurting 'blood like "a tap sun open."
VS: hypotension; weak, fast pulse. PE: anxious and confused; cool skin with reduced capillary filfing; w r y low central venous pressure: releasing r n ~ ~ r n i q ~ cnnfirmetl ret femot-al artery puncttll-P.

HPI

PE

Labs

CBC: milrllp decreased hematocrit. BUN and crearinine normal.


1,ytes: norma!.

Imaging

Artcriograrn shows abrupt te~,~ni~bation of dye propagation in i lie cnmmnn Fcmnml arlerv. Arrest of fcmoral artery hemorrhage with vascular repair: it~rensive I F ' fluid t h e r a p using normal saline and cross--matched blontl t ranst iisions: a~pplemen tal o x y r n : cloqe monitoring of pulse rare. hlood prewure, urine output, a n d central venous preqsure.

Treatment

Discussion

The clinical conclitions that cause hypovolrrnic shuck include acute and subacute hemorrhage and dehydration: fluid loss into an rxtravascular-cnmparlrrle~ll can significan~ly recl11ceiutra1-a~rlilnl- unlu~ne and result in non hemorrhagic hypuvolernic shock. .\ct~tr pancre:~tizis. loss of'the enreral integurnen~ (Crorn concliI ions surh a? hurnq ant1 ~lirgical ~vnunds) , or occIusivc or dyr~aniir ilel~s can all inrluce nligemic hvpotcnsion as a rrsult of clxtravasatio~~ of fluids into the rxtraccll ular cornpartrneilt. Othcr furms nf water and P C ~ I I I ~ 1 P 0 s ~ . s z ~ r l ias diarrhea, hyperglvcemia (leading to glucnsuria), diahctcs insipidus. sal t-wast ing nephritis, protrac~cd vorni ring, aclrenocr>rt ical railul-e, acute call a150 lead to deperiteni tis, and u\,crzcalmis iise of ~lii11-etic~. crciised in tr.avasca~lar imlu~ne ancl hyp.po~~olernic shuck. Piuients with prolrmged tissl~e hvpoperfusinn may progres? to metabolic acidosis.

"

SHOCK-HYPOVOLEMIC

TD/Ct

A 29-y-ar-old-malc is rc-frrred to a cardiology clinic Tor rvaluation For a prrmancnt pacc~naker.


T h c patiuri~ is aqmptomatlc and denieq r l i 7 7 i n r ~svncope, . chest pain. or shortnes~ ofbreath. Idle was incidrntallv noted to have sinus bradvcardia. M e is a marathon n m e r ant1 works as a ranger in a. t~atiotlalpark, often at cle\gtir,ns abolrc 8,000 feel.

HPI

z
.-

PE

1 ' s : nc) revet: mild hypotension (BP 90/50) without orthostasis; bradycardia (HR40 ). I T : 111i 11 allrl athlef ic-lnol-ing: normal JCT: S1 and S 2 r~ormally auscul t a t 4 withnut any murmurs, hr;rllr>ps, atlrl/ol- n~hs: n o lower extremity edcma.
CRC/Lytes: normal. ECG: ~nar-Red sinus bradycardia with a ~entiicular rare of40 heat4minute.

<

Labs

Imaging
Treatment

SR. chest: normal.


In u r n e r q n ~ sit t~ations. treaL symptomatic sin115hradycardia wit11 IV access, supplemental oxygen, and cardiac monitoring. I i ' atropine t11ay he 11serlin symptn~natic patients. Correct all underl>-ing eirctroIytcr and acid-hasc disorders or 11vpoxi;i. ~Zddress cause of hradvcardia. T h i q parient has a physiologic sinus bradycardia, and thus n o trcatmen t is indicated.
Sinus bradycardia is defined as a sintls rliytll~n with a resling I I P ~ I - t ]-arc oI' less t ha11 60 beats/minute. Phy~inlogiccauses of sinus hraclvca~-dia include increased oagaI tone seen in athIetes anrl incirlental findings in young or sEeeping patients. Pathologic causrs inclz~de inferior wall myocardial infarction, toxic or environmental exposure (dimethyl sulfoxide, toluene), electrolyte disorders, infection, sleep apnea, drug effects (digitalis gEycosides, beta-blockers, amiodarone, calcium channel blockers), hypoglycemia, hypothyoidism. and increaqcd intracranial pressure. T l ~ c most comrnon cauw nf ynlprornatic sinus bradycardia is sick sinus syndrome.

Discussion

SINUS BRADYCARDIA

ID/CC

,I SSyear-old man cninplairl~ of Fewre, cramping pains in his calves that prevent him fmm walking (ru'r~~hrr-rrs;m

c~~vn~c,mo~).
HPI The pniien~ ctates that t h e pain conies mainly drer plaving ba&erh:tll. MOYP recently i t ha< appearecl, accompanier1 I>! n ~ ~ m h n erollowing ~s, mild csul-tion and at rest (c11~eto progrcsrinn of distrase). Hu admits 10 smoking u p to t11ree pack% nrriprcttes per day. Pai nt'rll, cordlike itidur-atiot~r of veins (scquclac o f migratory superficial thrombophlebitis): pdar: c\ anosis: col(Ines~; diminihherl pcriphc-~-;d artery pulsatic~l~s; Raynaud's phenomenon: dclnyrd rctr~rri of halid color following releasr nT ternpnrarilp occlurIrcl radi~tl artrrv rt41~ilc cserci~ing hanrl.
Angin, peripheral: multiple occlrtrled segment5 of s~r~all and mcdil~m-sizccl artrrics in Iowrr leg.
Arterial s e p r n tnl thrombosis: no atherosclerosis: sc-cuildarv gangrene of leg ii scverc.

PE

Imaging

Gross Pathology

Micro Pathology

Segmcn taI ~ascllliti$ rvi t 1-i round cell infiltration in a11 layers of arie~-i:%I wall; inllammation; thro~nhosis: micrcri~h.;c.ec.; I'wrnation.
Cemation of smoking critical: sympathcctomy: i ~ n l ptat l ~ion.

Treatment

Discussion

Thrornhoangiitis ohlitcrarls t r i i d s r o a k r t merli~~m-size and small arleries o f the dislal exttemitics. I f smoking is not rlisrnt~tinlierl.multiple fingrr ilnd roc ampt~tatinns mav he

bbs T H R O M B O A N G I I T I S OBLITERANS (BUERGER'S DISEASE)


-

IDJCC

.s\ 50-ypa-11-r)lrlIPmale w h o wac arlmirterl ro he hmpital for rrratment nf stapl~yloct~ccal cnclocardit is c u n ~ l ~ l a i or r ~ severe s pain at tlie site of antibiotic infilsion.

HPI

She IKIF rt-ceiling cloxacillin (has proprnsi ty tcr callse 1 f l r c ~ t ~ ~ l ~ ris) ~~ in ~ arlcli l~ie tion ~ ~ ito t pen iciIl in ancl gcntanq-cin. Markerll!. tender, cordlike inflamed area found at sitc of' inCi~sinn.

PE

Gmss Pathology

I tlrmli~niinal renolls thl-omb-t~~ aclhercnt to t h r rcssul wall.

Micro P~athology .\cute in llamin;~loryr.ells with cndolllclial wall darnage and intral~rrninalthrombosis.
Treatment

<:l~ange itlf~~~in site r l freql~entl~~; NSMDF and local hcar; support :1nd brd rest.
Superficial thi+nmhophlcbi tis rrlclst c o ~ ~ ~ n ~clcclit.u or~lv i l l varicose veins or- in veins cannulated for an infusion. Spontauuous ~I~rnmhopl~lrhitis may occur in couditiuns a ~ ticau p i - e ~ ~ a ~ ~ r v . pol\-crthnni;~, pc)l\.rlrteri~is ncldosa, ancl Ruerger'c diwase (~liromhoangiiti~ ohlirel-ans) and as a sign o f viweral cancer (1111-nn~l>ophlel>itis 1 n i g l - a n ~ T r o u s s r ~ 1s~ i1 g' ~1 i. )

Discussion

TH ROMBOPHLEBITIS-SUPERFICIAL

ID/CC

A 15-yeawld boy i s referred to a cardiologist by a primary care physician for an evaluation of recurrent dizzy spells.
During his episodcs he feels intense anxiety with palpitations and breathlessness. He has n o historv of chest pain or sycope and is normal in 'between episodes o f dizzinew.

HPI

PE

reneral and systenlic physical exam nnrmal; cal-cliac exam normal: otologic causes ruled out.
EGG: short PR interval, wide QRS complex, and a slurred upstroke ( "DELT,~IY+VE") of QRS complex: R wave i n Y 1 positive. Elcctrophysiologic studies conlirm presence OF a bypass tract and its potential T o r development of life-thscatening arrhythmia.
Catheter radiofrequency ablation of the accessory tract iq the treatment of choice. Since cligitalis reduces the rersartory period of the accessory tract. it should he avnidecl.

Labs

Treatment

Discussion

Wolff-Parkinson-White (WPUT)%l;nrlr.orne is a term that i s applied to patients with both preexcitarion on ECG ancl paroxysmal tachvcardia; in this caw, the spells of dizziness c o ~ ~ l d have bcen either paroxysmal supraventricular tachycardia or atrial fibrillation. I n MTMrsyndrome, an accessory pathway (Kent's bundIe) exists between the atria and veniricles. An atrial premature contraciion or a veil tricular prema tur-e con traction generally initiates rhe reentrant tachvcarclia, with the accessory tract ~isuallv conducting in a retrograde manner; the rla~igelof atrial fibrillation lie5 in the fact that the accessory pathwav may he capable of conducting very fast atrial rates, leading to a Enst vrr~triruIar response that may degenerate into ventricular arrhythmias.

WOLFF-PARKINSON-WHITE

SYNDROME

ID/CC

-4 60-year-old white male farmer presents with skin lesions on his

forehead, above his upper lip, and on the dorsum of his handq.
HPI
PE

He clops 11~1.1smoke. drink alcoliol. or chew tobacco.

scaly or rouxh surface on

Ro~inrl or irregi~larlyqhaped lesions; tan plaqlies with adhcrent forehead. skin over uppcr lip, forearms, and dorsum of hands; lesions range in size from several rnillimrtcw to 1 cm or more.

Micro Pathology

Kpidermis I tiickened with basal cell hyperplasia: aorpical celIs rend t o invacle most s~~perficial pnrrion or the dcrmis. which
sf~ows thickcnin~ and fibrosis ( E M ~ O S I S ) .

Treatment

1-iquid-ni t maen crvnilierapy; topical trealnient with fluat-nu]-acil;surgical excision; electrodesiccation; minimize ~ u cxposurc. n

Discussion

Alsn known as ~enile or solar keratosis, actinic kel-atosis is the most cornmon precancerous dermatosis and may progress to squarnous cell carcinoma. It occurs most commr~nly in fair-skinned incli\idlzals and in oldcr pcssons. Signs that ar titlic ketalmis har become malignant are elevatinn, lllceration or inflammation, and recent enlargement ( > 1 cm) . Irnmu~~nsupprrsscd patients are at high risk of clcvelopjng actinic keratosis with pmlonged sun exposlue. Look for multiple lesion4 and for newly rlevelop~rl leqinns; biopsy dl suspicious lesions.

ACTINIC KERATOSIS

ED/CC

.2 12-veal-+Id male presents ~ 4 t h SC'VUTP itching arid Iw~-ning at the hick of both knc-es.

HPE

13c11as11;tdsimiIarrpisodrcsincethragrof7.I-lismothers~~ff e n frnm asthma a n d h i s father had ;I similar skin h e n t .


1 ' s : no fcver, PE: prricxal pallor. iiicreaqed palmai- ma t-kings. and extra fold of skin below the tower eyelid (DENNIL'S LINE) : erythematous, vesicular, weeping, rough patchy skin rash in hr>!fipopliteal fc)\sae with zhic'keilitlg. crt~sting, ailcl scaling 011

PE

the peripherie~.
Labs

CRC: eodnophilia. High serum IgE levels. Skin h i q l s ~ reveals Iymphocvtic infiltrate wit11 erlcmatous incel-cell~tlar spaces in the epiclcrmis i111dprominent in~ercellulxrbtidges; splnypd keratinnryre~located primariIy in the rtr-nt~zm spinflsum.
,2voirlance of kin il-riran tq: lo\- or-midpntencv topical glucocorticoids: an ti h ixtamincs: .;ysttmic an tjhi.otic's

Micro Pathology

Treatment

(for rrcondar. y inFec~ion) . Severe exacerhatinns nnr-esponsivc to


ropical ~IPI-O~CIF I F I need ~ ym'temic stel-oids or imnirinosupp~.essi\'e

therapy.
Discussion
Clli nical cr-i~cria f c rh ~ r cliagnnsis o f atopic dcl-matirisinclr~de recurrent cpisodcs of pruritus lasting more tlran 6 weeks with a

pcr~onal 01- Far~tilv historv sf'atopy and 5kin lesions tvpical af

ATOPIC DERMATITIS

ID/CC

PI 6H-vear-cl.1~1 red-haired white male prescil ts wit 11 a S m a n t h history of a progt.~ssively raised. bleeding, ulcerated lesion over his Lipppr lip h a t has not responclcd lo various ointments.

HPI

I-Ie is a fanner and has alwavs worked outdoors: he occaqinnally smokes hut does not drink.

PE

Large, ifl-deherl, telmgiectatic and ulcerated nodule { ' P ~ Y F.~P~TI.E wit11 ~ ) heaped-up hordcrs located over right upper lip;
no re-gional lymphadenopathy.

Gross Pathology

Generalll; local hut s o m e ~ i i n extensivr r~ clestr uction.

Micro Pathstogy

Eiopsv shows basophilic cells with scanr cytoplasm as well as palisadirtg hasal cells with alypia and increased mitotic inrlex. Surgical excision with biopsy: cryosurgeq; elecrrorlesiccarion.
Basal cc.11 carcinoma typically occcli-5 in lisht-skinned people. T h e most common skin cancer, i t is seen mainly on sun-exposed area5 (e-g.,face, nose) anil i q very s l o w - ~ ~ ~ > w Metastatic ing. disease is rare ( < 0. I 7%) ; chronic. prolonged exposure to sun is the mosi impol-tanc risk factor, Other risk lactors incIllde male gender, advanced age, fair complexion, and outdoor occupations. An increased incidence is SPPII in pcopltl wilt1 d e f ~ c t i w DNA repair mechanisms (e-g.,xerndcrma piLgmet~tosum) and
i~nmt~nosupprcssior~.

Treatment
Discussion

Atlas Links

P I-

TLl-'_l MC-33%

MC-335B

ID/CC

:in X-war-cdrl boy preset1ts with intense prrlritus and fluid-filled 'blisters over his arms and legs.

HPI

I-Ec rrccntl\+~*cnt on a camping trip with his classmates, during which he plaved the whole day in the 1 ~ 1 s h earound s the camping site.

PE

typical linear streaked vesicles over both arms and legs; w e ~ p s and encrustert areas: numerolls scratcl~ marks over kin.
Gram wain and cullure to rule out secondary infrctior~: E(OH preparation negative.

Labs

G~OSS Pathology
Micro Pathology

Skin crythema and cde~na, with Iinear streaked veqicles

S~zperficial perivasc~t lar Iymphocytic infilmtion around the hlood vessels aqsociaterl wit11 eclema o f thc dermal papillae and
mast celI depar~ulation.

Treatment

Sysr~rnic ancl oral ~teroids.

Discus~ion M%ilc at the can~p~gr~rt~und the lsov probablv encnu 11 tel-ed poison i1.y. a plant that pl-oduces To~v-molecular-tveigtlt oil5 ( r R U S I ~ I O L S ) that itlrluce contact hyp;p~rsensitirity, wllich i q ;I ceI1-mediated, tppe IV hypersensitivity reaction. The antigrn is presented hv the 1,nngvrhans cellq to the helper Iyrnphocytcs. Both cell type5 travel tn regional Ivmph nodes. wherc ~Tle an~igen presentation is incrcasccl. Up011 antigen challrngr, the sen~itii-erl T-cells infillrait: the dermis and ljegir~the immune response.

CONTACT DERMATITIS

ID/CC

A 95-year-old man present$ with an intenseJy pruritic rash on his elbows, knees, and back. Hc has celiac sprne and obscrvcs prcscrihcd dietary precautions (gluten restriction).

HPI

PE

PE: bilaterally symmetrical polymorphic skill Irqir~ls in the form of small, tense vesicles on erythematous skin (aTtc=n in herpetiForm gt-ot~pq); l~ullae and group of papl~lrs over ~capular and sacral areas, knees ancl dhows, a n d other extensor surfaces.
HLA-RB/DR-w3 haplotypc (partic~il;trlvprone).
Polymorphou~ erytl~ematous lesions, including papules, small vesicles. and larger ihullae. Skin biopsy rervals char;~cteristic subepidermal blisters, tlecrnhis, and dermal papiflat-y rnicroabqceaes; direct immanoflut~res cence rturlies reveal granular depodts of IgA at tips of dermal papillae.
m ,. n 3
r

Labs

Gross Pathology

o 3

Micro Pathology

< k

Treatment

Dapsone therapy after conljrming adequate gluco~e-tipho~p2iate tl~h*y'rlmgenase (G6P1)) levels {dapsoile prclcluces hemolvsis in C;fiPDxIeficie trr inclividuals).
Dcrrnatitis I~erpctiCilr~nis is a vcsicuP;+r ancl esrt~tnely pruritic 5kin diseace associated with glnten sensitivity enternpathy arid IgA inlmunc comp1cxe.i rlepositcrl in clerm;al papillae; itrdi~irl\ials with HLA-BHJDR-w3 I ~ a p l n t t ~ ;%re e pretlispnsed to rlevelop ing 1 1 1 ~Cli~ea~e. Males arc often m o r e c o m ~ n o n l y affcctecl, and prak incirlcnce is in lhc diirtl and limrth decarleq. Paiienrs on long-tern1 clapsone therapy sho~~lrl he mnnitnrrcI for hemolvsis and 1nethernoy;lohinenlia.

Discussion

DERMATITIS HERPETIFORMIS

ID/CC
HPI

A Itiyear-ald female colnplains oC midtiple nevi o n her skin.


She is conce~.r~ecl because an aunt who had a similar illness dcvelclped malignant melanoma and died oi-metastatic complications.

PE

MuItiple nevi measuring 6 to 15 mm noted: nevi are varieptecl qharles of pink. tan, and hrotvn and seen o n hack. chest, buttocks. scalp, ancl breast.^; borders are irregular and poorly deFmed hut lack the scallopitlg of m;llignan melanoma; 1 1 0 regional lymphad~nnpathy noted.

Micro Pathology

Skin biopsy rcvcals rnelarlc~cytes with cytoIogic and architectural atypia. en1;irged and fluccl epidermal nevus rell nests, lentiginous hyperplasia, and pigment incontinence.
Stm protection; regular skin exam to dctect dcveloprnent nf malignant mc1;irlotna anrl l o I~iopsy suspicious lesions. Family 1nt.rnbe1.s should he regularly monitored.

Treatment

Discussion

Dysplast ic nevi are found in inrlivid t~als with an autc3surnalrlo~nirlan t predisposi tioti to rlevelop acquired n ~ v i tl~esc ; may develop into malignant melanoma.

Atras Link

rlV-m7 MC-135

DYSPLASTIC NEVUS SYNDROME

ED/CC

h 24-~enr-olclremale presents with a sridrlen-onset skin rash on


both Forearms.

HPI

She s ~ ~ f f e r from s herpes labialis and llacl a recent recurrence. Cui-runtlv she is not taking any medications.

PE

VS: i ~ o r m a lPE: . papulovesicular, erythematous skin lesions on both forearms, occllrring in concentric rinrinp with a dear center (TAR(;ET Z.T.SIONS); Inllcnus membranes spared (v'i. S t ~ v c n s ~ J o h n s o synclrome). n
Skin hiopw rrvvals cIcrinal edema and l ympl~ocytir in liltrates i ~ ~ t i m a t cassociatrd lv wit11 d e ~ e ~ ~ c r a rkeratinocpte~ ing along the durmal-r.pidcrmal j~~nction; target Ieqions reveal a cciltral nccrosecl area with a rim of perivcrirlla~ inflammation.
Treat underlying cause, ~tipp01-I ive t herapr;; steroids in scverc caws.

Micro PathoIogy

Treatment

Discussion

Ervt tiema rnultiforrne i~ a I i y p ~ ~ . w n s i t i ~ reSponsc ity to curtain d r u (corn ~ tnnrllv ~ i ifnnarnides. l NShIDs, pcrticilli~~. pllenytoin) and infections (~\,lymfilnsmn,HSW. It is clinically ditbided into m;Licsr and minor types. T h e minor type it~vnlves limited cutanerms ~ ~ ~ r f awhile cc~. thc major hq~t (STF\~F,NS--J~~-INSON S ~ W U ~ ~ C is I ~charac ~ E ) trrizccl by toxic fealurw and involvement of

mncosal sto-lnces.
Atlas Link

1 1 . -f 51-'

MC-018

TI

ERYTHEMA MULTIFORME

ID/CC

A 24-year-old male presents with acuteanset, painful swelling i ~ r his left axillae.

HPI

H r aIso reports fever and a h iqtnry of ponrty controlled ,juvenileonset diahcte5 mellitus. VS: fever (59C):t;~hycardiil (HR 1 10). PF,: multiple. mobile, extremely tender, erythematous and fluctuant axillary nvellings; aspiration of nvellings yields Crank pus.
CRC: Teukocytosis. Gram stain of p u r ~c ~ e a l s gram-positive cocci in rlusterq; c i ~ l tit-e i grows caag~~law-positive Sfaplylococcvs
U?lWllS.

PE

Labs

Treatment

Incision and drainage; systemic antibiotics (empiric pcnicillinase-resistant p-1act:rms; the11 accrbrding to reporzcd culture sensiiiviriesl.

Discussion

A boil ( F - ~ ~ R V K C I . is ~.) a deepseated inrection of the hair frdliclc and adjacent subcutaneous tissue. most cornmonlv occurring in rnrrisz hair hearing parts of the body. Diabetes, HW, and rV drug ahuse are preclisposing conrlition~. Recurrent cutnncous infectirlns with S. arrwws may occur due to a chronic carrier state (must co~nmonlv in the anterior nares).

FURUNCLE

ID/CC

A 23-veal--oldHW-positive man prcsencs wit11 nonpniritic


reddish-brown lesions.

HPI

Hc has had a contin~iorir low-padc fiver, significant w i g h t loss tnreli l i e p a ~ fii m ~ ) th t i ~ aild , painles~ lutnps in the cervical, axil l i ~ r y ancl , it~guinal areas.

PE

1 ' s : lever. PE: rn-laciation; pallor: gencralizcr2 lyn~phadenopathy; 110 h e p a t n ~ p l e n n m e or ~ l sternal ~ tenderness; reddish-purple plaques and nodules over trunk ;ind lower- ex1retllities: similar lesion4 nnted in nral murnsa.
ELISA/Westcrn blot positive fur HW. CBC/PBS: lymphocytopenia With d e p e e d C W + cell count (< 100).
m
u m

Labs

Gross Pathology

Reddish-purple, raised plaques and F m nodules wilh no suppiwation.


Skin biopsy of ~iodutar ler;ion dmws maligna~lt spindle cclls with sli like .;pace%corn taining R K s , inflammator?.crlls. ancl Iremosirlerin-laden macrophages. Ratliat i c m ; cheino~ herap? wit 11 e~npoqide nr doxor~~hicin. Ideomvcin. or-interferon, arld vinblastine. IF iatl-ogenic, slop irnrnunosupp~-esxive rneclication.
Iiapo5i.s sarcoma is the most c o m m o n cancer associated with A D S (epidemic lype). The non-,&IDS type affects Ashkenazi Jews {chl-onicor classic type) and Africans (Iyrnphadenopa tltic nr undcnlic ~ v p ~but ' ) . the disease is not as a~gressive. Human herpesvirus 8 is associated with ill1 VpeT: disordered cytokine reg~~latian a l w plays a rolr. Clthrl- than rlle skin, l e s i o ~ ~ arc s most commonly Folrnd in thr lymph nodes, GI tract, ilild lung. It1 contrast to Iyn~phoma, Iymphadenopathv presents earlv and is rlnr sigmificat~t.

Micro Pathology

Treatment

'Discussion

Atlas Link

i 3 Z J T I l IM2-007

KAPOSI'S SARCOMA

ID/CC A +vc.;~r-oldJapitnose male presents with fever and an extensive


skin rash.
MPI

h pri trlarp care physician had pre~iouslvround the patient to havt- cerlical aclenitis: antibiotics wcrc administered but achicvcd n o rrspvnsc.

PE

VS: i'rvet: PI.:: conj~~nctival congestion;dry, red lips; erythematau.; palms and soles: indurative edema nf perip11er;~l cxtrcmiriec: desq~~amatian of fingertips;varir~~is rashes of t r i ~ n k ; cervical Iymphadenopathv > 1.5 crn.
Throat swab and culture sterile. CRC: rclutinc ~ ~ I C I O C CC I ILI~~IS nor-1na1;further difl+rential blood counts r e ~ ~ c increased al Bcell activ;!tion anrl T-hclpclA-cell lymphocylopenia. Paul-Eunnell test for infectious mononz~cleosis nrgative; serologic tests rule out c v t o i n c ~ ~ l ~ ~ vinfrction irus arid loxoplasmr A s . Angio: presence or coronary artery aneurysms.

Labs

Imaging

Gross Pathology
Micro Pathology

.~ne~irvl;mal rlilatittinn o f 'the coronal-v arterie5.


Coronary arturitis is ~lsually durrloristra~ctl atlropsy toget her w i t h aneurvrtn fbl-marion and thromt~osi~. Aspirin and n' gamma gluhulin ;~rr effective in preventing cerolrarv complications if initiated early. k~wnsaki's syridrornr is ~rsuall!self-lirnited, hut in a few instances L~mlccli-orlilrv thz-nmbosis has occllrrecl during the zrcutc stage nf !lie diseasr or inalrv montfls dter apparently complete I-ccovcrv. Casc fatali? rates have bheen ako(11 1%) 102%.

Treatment

Discussion

Atlas Link

P I F I MC-300

rn
:

KAWASAKI'S SYNDROME

ID/CC

A :<O-vear.-olld woman i s seetr with a 1 1 itchy rash over her wrists, forearms, anri trunk.

MPI

She cnrnplain~ thar fi-erh lesions occur aIong scratch mark5 and areas oC trauma (KOEBNEK'S I*H~NC)MI.:NWN).
VS: no Ftvrr-, PE: polvgonal. purple, flat-topped papdes and plaques; tiny white dots and lines over papules (M'IcwM'~ T ~41:) I ; white net1ikr pattcrn n C lesions over oral mucosa.

PE

Gross Pathology Hat-topped. violaceous papules and plaql~es without scales.


Micro Pathology
Dense, handlike (lichcnoid) lymphocytic infdtrate (predominanrly 'I' cell?) along the clermal-epidrrrmA,j~rr~ction: sawtooth pattern of rete ridges; dcxtrl~ction of lx~sal cells.
w
m
33

3
0

Treatment

Steroids. topical (pntent fluari tiated) or systemic: isotretinain is au eCCectivc altel-native. b ~ risk ~ tof teratogenicity must he bol-ne in mind w h e n prescribing.

<
G)

Discussion

Liclieii plan11~ is a self-limited inflammatory skin tlisease, bnt in qome cascs it may be prercnt frlr scvc-ral ?cars. Ferrii~les art. affected more frecll~elirlythan i~ialeq. Postinflammatory hyperpigrnentat ion t1>;1xr 1~ rvirl~n t after tllc Icsio11~ subsiclc. Merlicarintlq m c h ar tetracycline, pcnicillarninc. and I~v~l~ochlorot11ii11,idc can c:~u~stlichen plaruts-li ke skin reactions

Atlas Link

17-77-1 MC-142

L I C H E N PLANUS

h .TO-vear-old white male preretits with an itdiv, rapidly enlar@g, pigmented lesion nil the sole of his left f-not. He states that the spot has recently chan~eged color di.an~a!ically; once ligl~tlp pigmented, i t is now a d ~ e p pltrple hue.

Irregular, q m m e t r i c . deeply pigmented lesion with various shades of red and Idue: cliametel- > 6 mm; lef~-sided nontender inguinal lymphadenopathy.
Gross Pathology

Slightly raised: deeplv pigmentrrl wirh uneven I I L W ~ and irregu131I>ot~Iet,


Excisional l~iopsy sllotvs tx~mor-free horrlers along with large, atypical, vai-iably pigmented cells with irtrglrlar nuclei and ensinnphilic r~ucleoliin epicler.rnis and papillary rlermis; dermal invasion noted in some places: metastases ~11ou.n ntl lymph node biopsy.

MScro Pathology

Treatment

E ~ c i s i o with ~ l I V ~ C ~margin, C reginnal Ivmph node diwectinn, chrmofhrrapy. irnnltinot herap!.


01'aIl skin cancers, ~nelarloma is respnn~ible for the largest n~~rnher or deathq. An increased incidence is sccn i11 fair-skinned pcople nnd in those with dysplastic nevi. immunowippmsion. arid exce~qive sun exposure. M e l a n o t n ; ~ ~~ ind~i.g ; 1o radial (superficial) growth phase followed by an inr-qive. vertical growth phase. Bleeding, ulceration, a n d pain are late manifestations. The chance of metastasis increases with depth of invasion (measured tlsing Clark levels I-V). Metaqratic rn~lano~nas are innuable and signify the need for early detection a i d prevention (r.g..sunblock, clothing).

Discussion

Atlas Link

MALIGNANT MELANOMA

ID/CC

A tX+ear-trl male prpsent5 ~ i t h ~nt~ltiplc Iurnps and a chronic, pruritic, ~rptilen-latolls rash that has sprcad and n o w involves almost his entire body.

HPI

He has seen Inany doctnl-s. hut the rash has not responded to a ~ ~ T i e 01' t y ~nedicatiwt~ s, i r>rlltdin~ topical and systemic steroids.
Frythematous. circinaie rash in plaques with exfoliation (SC:~\LTYI:) : w m e nodules seen nil lace, trunk, lu~t+er al~rlarnen, at~rl h11t1or ks: rln regiclnal l!mphaclenopathy u r hcpatosplcnomcjial~.

PE

Labs Imaging

C:RC/PRS: Iynphocytnsis.
CSR: nn mc.dia<~inal lympl~adcn apathy. Reddislr-brown, kidneyshaped plaques (vs. Hoclgkin's Iymphamn); hcncc name "red man's disease": exl'oliatiu~~, norlzllr ft~rimation. and ~ometimeq ulceratinn. ,.Ztvpical, PAS-positive, large, CD4-antigen-positive (helper T-ceII) lymphocytes with characteristic multiconvoluted. "cerehriform'knclei (S~%~RY-L,~~T%NEK (:ELLS) ; tlcrmal i~~filtratinn with cxocvtosis of atvpical mt~rionuclear cells \\+ithill cpidcrmis fo'oui~d singly or within pl~~iched-ou t epidermal microabscesses ( P A I ~ I F . R ~RS(:I;SST;T). 'S
l'lR'r\; to~al ski11 elcctmn-I~eaintherapy; prednisone and chloramhuci l o r low-rlose methotl-rxate.

Gross Pathology

Micro Pathology

Treatment

Discussion

l4vcosi.i fungoidcs i.j il ~ n a l i ~ w acritilrieous r~r helper T-cell I yinphnma: rli5seminated tliseasr with exlbliative dermatitis and grneralizrrl Iynlphadenopathy is tcrmcd SCzary syndrome.

I h

MYCOSIS FUNGOIDES

IDJCC

A 2:'Fvear-old tnale is admitted to the hospital for art cval~~aficlr~ of recurrent epistaxis.

HPI

The p ; u ien 7 ' s mot her died nf a massive pulmonary hemorrhage


due to an arterio~enous malformation.

PE

Small telangiectatic lesions seen on lip.;, oral i i r j t l ~iasal mucosa, tongue, and tips oI' lingers and toes: anemia ~loted; no pulmonarr bruit llearcl (to drtect a n A\' rnaPfonnatic~11).

Labs

Ch( :: norrnori~ic, r~orr~locl~mmic anemia (due tu occt~ll purl-ointcstinal hlood low). Giiaiac p{>qii ive.

Imaging
Micro Pathology
Treatment

C T AV malf~~rrn:~Lionu in liver and splren.


Irregularly dilated capillxrics and venules.

Nasal packing, cautery, and estrogens rnay he tried to control rrcnrrent epist,nis; sipificnn t \iscc~-al ,4V malfor~nations mav

r ~ qlire i t=tnlw)limtion.
Discussion Hcrcditarv l~emorrhagic telangi~cta~ia. or Osler-l4'~her-Rrndu wnrlmlne, i q inherit er? a? an autosomal-dorninant trait. Telar~giectasim may first be reen dnring artolescc-nce and the11 incrcasc in irlciclence w i ~ h aKe3peaking Iwtween the ages of 45 arid fill years. A\' li5tulas may prescnt with hc~nopysis, indicating high morhiclitt3.
MC-145

Atlas Link

OSLER-WEBER-REN DU SYNDROME
-

ID/CC

A 45-year-old woman visits hcr der-matologist cc~mplairiing of

painful, blistering skin lesions over her back, chest, and a r m s that break down and leave denuded skin areas.
H PI

O\.er the p a s t few \?earsshe has bad large recurrent aphtho~w ulcers in the mouth. Slie waq not laking any drugs before her wmptnms developed.
Large aapbthorw dcers seen over oral and ~ragirlal mucosa; vesiculobullous skin lesions seen in various stages: vertical pressure over I)r~ll;le leads to lateral extensinn ("nl'r.rA SPRFAIJ s ~ c : ~ " ) <kin : over hullae peels Iike that of a "hot tomato"
( Z \ ' l k O l , ~ k 'SI(;K). f~

PE

0 m
; P

2E

Labs

indirect irnmunofloorcrcrncc trst


shows pwseece 01' IgG antibodies.

IU

detrct anrihodirs in serrun

5
Q r

Gross Pathology
Micro Pathology

FI-PSEI vesicle is selected for hiolxw.

I,esion% qhow 1 0 1 9 o f cohesion of epidermal ceIIs (~r.-\h~ror.\.;rs) that prnduces clefts dircctlv above h a d cell laver; Tmnck smear of matcria'l from floor of a l>tilla revc-als acantholytic cells that are round with large I~yperchroma~ic nuclei and h o r n n g e n ~ o l ~ s c~topl;tqm: direct irnrnunofluarescence reveals characteristic I@ interceUular staining and deposits.

Treatment

Steroid5 are mainstay or [herap?; rytotoxic rlr~lgs(qclophosphamide) .


P e m p h i ~ vulgaris, ~r an Intraepiderrt~al blistering disease of the skin and mucous memhranus, usuallv appcars in inclividuals in the third t o fifth rlrcittfe of life. The t>liuter.; resl~lt frotzl 104s o f adhesion 1 1 e ~ e e epidermal n cells car~serlhv the prorlurtion of aumanti1wIies that are direc~ecl against k e r a t i n o c ) ~ cell ~ surface prowins; loss ol' cellcell con tacl h ~ h v ~ e Cn l ? ~ I t l ~ ~ n(which me~ a r e ~ites of attachment for epidermal cellsl has heen demonstrated by c l e c t r o ~ microscopy. ~ Lnrrrated pemphigus m~lgaris is often fatal.
PM-PI-043

Discussion

Atlas Links

E T T I I MC-019

IDJCC
HPI

A 17-v~ar-nld girl presents with a scaly m h on hcr trunk.


'Thl-~e weeks
ii<<),

chv noticc*rla <mall waly m<11on hrr t w r k I h:~i

progressed in :~hout a week 2 0 involve thc trunk and upper


extremitie~. h i d e from the rash, she is asynipton~atir. She i q

sexually aclive.

PE

VS: ~lormal. PE: crop of oval, erythematous, scaly maculopap~~lar lesions on trunk, neck, and proxitna?rxtremitier in a "Clzrishas tree" distribution.

Labs

R T R / \ D R L nun-reactive; EWSA fbi H T C Tn~~ative.

Gross Pathology
Treatment

Biopsy specitnen shows superficial perivascuIar dermatitis.


Treat with moisturizers and antipn~ritic lotions:topical steroids o r oral antihis.tamines rarelv requir.cd: ul~aviolet I3 light used to relieve p n ~ rtu.; i in resistant saws; provide reaqsuranre, since rlisease i q benign and self-limited, and recurrence is uncommon.

Discussion

Pityriasis rosea is i i common skin rash with the Iliglles~ incidtalice i ~ young r ;~dults and teenag-ets.The disease ir hvice as rn~nmon in women as in men. In most cascs. thc initial: lesion i s a I- to In-cm, oval rnaculspaptil;~r Iesioii, called a "herald patch," that is c o m n ~ o n l y I l u n d on rhe t r ~ m k or nc-ck. Pir,.riasil; mwa i~ a clinical rliagnosis; i~ is important ro differenriate the diseasr. from secondary syphilis, tinea versicolor, psoriasis, and drlq reactions.

P I T Y R I A S I S ROSEA

A 40-year-olrl male cnmcs to a clcrrnalolog-y out pat i ~t n clinic wit21 an e x t e n s i i ~mildly , pruritic, and chrnnic qkin rash.
It improves during the summer and markedly worsens in cold weather. The patic~nt was prcviouslv di:tgrlosed by a11orthopedic surgeon r\itI1 distal intenphalangeal joint arthropathy.

Mnl tiple salmoncolored plaques with overlying silvery scales seen O V P ~ I>ark xncl extetlvor xspectc of upper ancl lower limbs; .nn removing scale, underlying pinpoint bleeding capillaries seen
I)HF.NOM FU,~ :)pitting of

[AFSPIT~ SICK) ; lesions seen along ~cratch marks ( K O ~ HER Y nails with nccasianal anvchol~sis scen.
70
3

Imaging

SR,hands: asyrnmrtric clegenera~iv~ rhangerr invol\-ing the distal


in ierphal;lngeC1I -joints. wi tfi "pencil-in-cup''deformity.

3
9 0
4

Micro Pathology

Skin biopsv rev(~:lls mxrkcdly thickened stratum c n r n e ~ i m with lavrrcd L o r l e v o f pi~rakerarosis(relentinn of n u c l ~ i;)m:wkedly hyper plastic epidermis with elnilgation oi'rete projectfnns; cnllection~ o f PM NFwithin the Ftrahlrn cornerlm (MLJNRO'S ~!!(:RO~.~~S(:ES~ES) : marked degree of epidermal hyperplasia with little inflammatory Xitrate (characteristic ~nicrnscopic iinding).
Exposure to sunlight. TIle f o l l o ~ i n g l~avc hccn used either alonr o r .in coml>inatioll:occl~~aivc. clre.isii~gs, tar ointment, rlilht-annl, P'CTLrh,lopica1 c,teroids, and crrtoroxic drug<sl~ch as rnctholl-rxiltt.

G 1

Treatment

Discussion

P~nrixqisis a hrrcrlitarv condition that iq characterized by welldcfincd placlurs covcred by silvcrv scalvs. Lcsions are most commonlv v r r n in an Pxfensni. ~ l i ~ t r i l ~ ~ l l111t tion the , nails, scalp, palms, ailrl soles I I I ~ > alsn he involvecl; arthritis of thc distal in terphalangeal~join t mav hc s c r n in 20% of cases. Parentera1 ct~rtiuostcrnids arc contr;lindicatrd u~vivil~g to t l l r possibility of indt~citlg puc,r~tlar IPF~OI~S.

Atlas Links

7 PSORIASIS

ID/CC

A 40-year-old female presents with on hrr lcft calf.

;tn

cstrrn~ely painfill ulcer

HPI:

Tl~c Icsion appcarccl a month ago as a small boil afrel- the patient hurt I~ersrlf. 1t t l ~ c n beca~rlu progr.rssivelv I>~l-ger. i t rt ~iI it cracked upcn 3 c l a ~ s ago. T h c paticnt rc=pclrts a Iiihtor)' of' ulcerative colitis, which wa.; tliamt~sctl srvc.1-a1 ycarli i ~ g o ant1 ~nanagecl ef'fi-r~i~,el~. wirh \reinid crletnas and oral ~ulfasala7ine.

PE

17s:normal. PE: 10- l ~ y 10-cm dcep ulcer with violaceot~q horder urcrhan~ing ulcer hcd: n o l?.mph;~rlcnr>path~: good distal pulse4 palp;~hlc(TS. ;u-tcrial insrifficicncv ulccrl: ne~~rological exarrl nol-mal.
Nrl growth clcmnnstra'trcl on Gram slain and c~tlt~irt. uf'wr-~unrl s~~:ih skin : biopsy diagnostic. Skin I,iop*iy rrveals h~pel-liel-atosis; rlermal pcrirasr~~lar rotind cell infil tl-ation, and mixcrl infiltrate {ncutrophils, Ivniphr>cytes, macrnplragc~) cxtrnding. to thc subcutaneous plane.

Labs

Micro Pathology

Treatment

n7pulse inet hvlprctlni.inne) a i d immunosuppressive tf~et-apy (cyrlospnrinr or ~arrcrlirnuc): antihio~ics for cecnndar-v inl'rrtion and narcotic analge4c-~ Ibi. pain.
Svstcmic steroids (oral pl-ednisonc or
disease :~nrl fenkemia or prelerlkernic states

Discussion

Cr~mmr)illy i~xsociatpd cc~i~dition i~ic11 + IIIP inflammatory bowel (usi~allv mvclocytic lcukemia o r rnnnnclonzll gnmrnopathics). Most caws occar in thv four111 ur fifth tlc.c:rclel; of lire, wi I h fctnaIe4 aflecterE slightly more orierr than m ~ l e s

PYOQERMA GANGRENOSUM,

ID/CC

/\ 4O-yc.:tr-r1lrl whitr IR;IIC

J I ~ ~ F C I Iwit11 LS

a scaly, mildly pruritic

rash ovcr the face and scalp.


HPI

He wprJrts l l ~ 111r t r+a~Fi is aggravated by humidity scratching, emotional stmss, and seasonal changes. H e tester1 HIV positive laqt yrar and has sincr maintained a goorl Ct)4+ rtsllnt withnut a n r ;In liretro~iral theralx:

PE

2',5: nr~l-ma]. IT,: in!er<perst,rltliirk adherent cmsts and scales owl-lving arm5 nE greasy, yellow-red inflamed skin irivol~ ing the scalp, forehead, nasolabid folds, and chest.
M ; I ~ ;lin ~ I I good hygiene; medicated shampoo: topical hydrocodsone lotion r ~ kt.rc~ct)tla/.olc r ctc;i~n.

Treatment

rn
; D

Discussion

r \

p a p ~ ~ l o s q l ~ a r nskin o t ~ s rarh ir~volving ar.eXq r.ict i in sehaceol~s


Iirjul

o r

itnmzirir rrrpoilrr toward a commnn ?kin commcnsnl. Pityrosponrrn ovule. V:~riousrlr-rigs (halopcridol, l i t l ~ i u i int-tl~vlriopn, ~~, ci~nrtidinc) mav rvorwn 11ic c-c )ilditir>n. t I is also c o m n ~ o n l r rrn~nd in patirnls uliRer.ing FI-OHI parkinsonism :~rlrlin ~ l ~ c > s ~c C. C I I V P 1% ~ . ~Ii.0111 ~ ;III aclltt= MI.
ijbiorlnal

-"

SEBORRHEIC DERMATITIS

lD/CC

A 19-vcar-old black male complains ollmsightly white (dcpigtnerlted) patches on his knees and el how5 (honv pro~nincnc~s).
M e has ncj history of associated pruritus nr discomfort . T h e first patch appeared over the left elbow a few months ago, and the pt.oce~s has hern progexqive since then.

HPI

PE PE: flat. ~ ~ e l l d e m a r c a t arcas c d of depipentation oti face


(per-ioral or periocular)., elbows, knees. a11d neck and in skin fnlrls: si ces of' recent skin trauma arc also scen to have undergone depipenltatian (KOERNER'S PHENOM ENOV) ; most hairs within vitiliginous patch are white.

Micro Pathology

Absent melanin p i p e n t (111 ski 13 hic>psv\tail1 with I'erl-ic fen-icyanide: absence of melanocytes o n ~lectror~ micro~copy.
N a estal~lishprl sat i d a c t o r y treatment exists, a l t h o u ~ h sunscreens prorect and l i n ~ i the t tanning of nnrmallv p i p c n t c d skin. A pminising apprrlarh i s oral psoraletr (a photosensitizing clrug) followed I? cxponirc- to artificial Iot~g-wave ultrmiolet ligi~r (t'VA) ; potent flnorinated topical sreroids mav also be helpful. Generalized vitiligo may bc trcatecl I I ~clc.pi~,rsnc.u~~tion of I I ~ I ~ I I I skin. ~I

Treatment

Discussion

\'i I iIigo usually ilpprars in othcrxvisc-hralthv pcrsons, I J I I ~ sevcral <vlirernicrlisorders occur ~nura often in patients w i t h vitiligo, inrluding rhyroicl disease ( e - g . ,hvperthvroidism, Grwcs' di.iuasu, ;md diyroiditis), Addison'!: disease, perrlirious afre~nia, alopecia areafa, ~tvpir is, a i d cliahaes tnellitlrs. Yt~cipitating f;lrtors S I I C ~XF illness, elnotional stress, or physical trauma arc ofterl acsociated wit11 i ~ s onset. The disease may Ile inherited as a n autosomal-dominant trait ~ i t incornpler h r pener rallce ancl variable espre~sinn. Most sturlics, howcrcr, point to a n autoimmune Imsis (circula~ing complement-binding. an ti-mcl;morytc antihoclies have been deiected) .

Atlas Link

r l I MC-153

ID/CC

An ohrse 44-year-old female cc~rnplain~ of irritability and excewive ut~ight p i n (40 kg) n v r r t h past ~ 3 ?cars and requests ~ncclicnl weight-lops therapy.

HPI

On c a r c f ~ ~ questioning, l she also rcporls easy bruising, tig go men or rhea, weakness, and increased hair growth in various areas of her body. \IS: liypert ension (RP 180/110). PE:fi~cial acnr: tnlncal obesity M I t 11 t l ~ i irxtrernides; l buffalo hump and plethoric moon facies; hirsutism: wide. purple abdominal and lower leg striae.

PE

Labs

U,4: 3 glycosuria. Elevated fasting blood sugar; elevated planna car&qol;high r\CTH. Lytes: hypokalemia. CBC: leukopenia. Demethasone suppression test '~ils to suppress
htpercortisnlism.

Imaging Gross Pathology

SR,plain: generalized osteopomsis


Pit~~itar adetloma: y hilaiel-al adrenocortical ttyperplasia.

Micro Pathology
Treatment

Pi tuitarp: hrnign hasopIiilic a d e ~ ~ o m w-ith a Cl+c>ok's E~!;lli~~i~ation.


A~rgicxlr e m n d of pitl~itarv aclc~lorn;+ (11-anssph~~nidal adenrctomy) or pituitary irradiiition alotig rvirh arljunct mrclical
111 PI-apv.

Discussion

Cashing's syndrome compi-iqeq the manirestations of'I~?purcorti~nlisinr~gm.clless nl' its cause; causes includr cxccss gl~tcocoi.~icoirl administration, pardncopla~ticpr-ocesses (F.CT~PI(:), / L m H lrrod~~cticrn, adrenal lrsions [hat [prorhlre excesq cortisol. and pin~itwy lesions that produce excess ACrrH (Ctrs~rur."~ DIFEASE).
r ' l ' IM1-019b, IM1-019B.m1-ONC, IM1-OlgD

Atlas Links

CUSHING'S DISEASE

.f\ 4 0 - v r a ~ l c woman l

is see11 in the outpaticut clinic )\*itfr rrmlplainr c or ~lrdderl-onset painful neck swelling,

PI.~(H. to ~ l i she i ~ h,~rl a sore throat, malaise, and fever. Pain orJeithe 11iyi.oicl area ~-;~clintps to the ears and is NTorse CHI s~vallowixlg.

C'S: Irrrr: t;irliycarrlia. 13E:fine tremors of tongue and fingers of O LCFIYP ~ trherl hanrlu: firm, cxquisi trlv tender, diffuse, mildly enlarged goiter p ~ l p i ~ l ~ nonfirm tc; nodularilv PC-11: t ~ o cervical Iympl~~rlcnopatl~ no v : opli ~hatrnopitl~v (dis~ ingliislieq rroin Ckaves' ) .
Labs

CBC: elevated leukocytes. Elevated ESR (characteristic); clev~tcd free T . aud T, 1er.c-l~ a n d rcsin uptakc (sre11onlv
daring ,rc;lrlvstagcs due to follicular c l i s r ~ ~ p t i arrrl o~l hormone rclcasc; latcr t ~ u n s i r n z hypotl?y-oi<Iislrl111;1v c-IIW le: rarcly, pcrirlal~r-~l t hvpr )tIiyroirli\rn r e s ~t~ c )f; rlepl-eqserI 'TSTI; mal-kcdlr. reduced radioactive iodine ~tptstke ( kcv cliafiul>.itic l't.:~~ l~i-eanc1 I I C ~ for C ~ diffcl-cntinting i t fruun Gr.avt.c' disei~se).

Imaging

Gross Pathology

Iellr)\tt :ig:iinst

Diffi~scly cnlnr-gcd thvroid gland; i ~ ~ v o l v c arcas d arc firm and ~ I I Pnormal ti11 involvpcl hr.own ~ I i ~ i - o< id ~at>s~ancr.

Micro Pathology

Early lesions incl~lrle disruption of thymid follicIes with a neutrophiric infiltrate and formation of micmabscesses:si 1 1 ~ s ~ C ~ P I I Lmdtinucleated ~V, giant ceUs Inav be seen sut-rolinding colloid Iragmunts. r c s m m l ~ l i ~ g r ~ilriitlorna.;. g

Treatment

Propranolol and analgesic5 Irbr symp toinatir relief nre gencrallv adequate. 'l'he condition is serf-limiting. Iasting 6 ( ( 3 H weeks.
Sever-e ~ : I S P Si i i a ~ S~ P ~ L I pr~dnisone ~ ~ P therapy; pern1:lncnt h v p n t h ~ ~ n i d i s ma\ r n r c ~ u l on t rr?r*c occasions, in ~+*hich c;iw hol-~nonc. r ~ p I a c c m e n is t rcquirccl. Antithymid drt~gs are not indicated.

Discussion

I)e Ql~el-\-in's thyroiditis, o r s u l ~ a c r ~painful tr thvroidilis, i s l h e most romlnoli caLlsc s f scvcrc tInvruirl paill aild t~iirlernesu; il is mos~ c r ) ~ n m o in ~ i women 20 to 50 years o f ;i#e a t ~ r F l ~OWS at1 assoria~inn wit11 HZA-B35. Its exact ctiolop is ~ i n k x ~ o wbut n, viral caltscs l~:lr*c Ilren i r ~ i picaletl. l

P1

DE QUERVAIN'S THYROIDITIS

ID/CC

An obese 55-year-old whi tp male cotnplai t t q nf irlcl-easing thirst ;~nrl excessive appetite.

HPI

He also complain%of increased rtrinary volume, weight loss. ancl weakriess over the past scvcral mon'tli?;togctlir~r tvi th b u r n i ~ i g a n d tingling sensations i11 iI stocking-glove distriht~tion(9ugs:'eqti31gper~pheral neuropathy) . His father was diabetic will1 i i history of leg amputation and kidney biltil-e.
VS: hvpertcnsion (RP l50/9.5). PE: "'dot-blot"hemorrhages, exudates, and microaneurysms on f unduscopic exam: m ~ ~ s c l r arrophv in hipa and thighs: dirnit~ishrrlrlorsalis pedis and tibialis pl~lscs hilatcralIy.

PE

Labs

Elevated glycosylated hernogobin { H E M O G L O B ~,I,,). UA glycosuria. Elevatetl fasling serum glurose ( I ~ ~ J E R ( : I , K ~ ' ~ ~ I ~ ~ ) .
An~yloidosis: h p l i n e atherosclerosis: noclular hy;ilinc masses ( K i h ~ hE:I-S'TIEL-~'~ i LW)N YOLIULES) in glonlerul~~s. Diet aild uxerci.;c: oral htqmgEvct.rnic aget31.s:ins11Iirl as i~eederl. Pre\iously known as non-insulin-dependent. diabetes rnelliius, (Iiabetcs rnellit~tsty,e 11 is a metabolic rlisease it~vnhing carhohvcl rates and lipids c a ~ ~ s e bv d periphmal resistance to insulin. Althon~tl patients with type 11 diabctcs mcllitus are not pronc. to tlevelnping dia tjei ic kcioacidosis, thry ran rl~velop nonketotic hyperosmolar c o m a if their ldontl g l ~ ~ c o isqedt-aqr icallv rlevaterl. Scquelac of type I and type I1 diabetes meltitus include periphera1 vascular- disease, cot-onai-v artery I l i ~ e aSI ~t-eke, ~ , di~hetic nephropathy diahef c neumparIly, nnn healing skin ulcers, and cIelavrd ~vuund healing with incl-cascd risk of infectiorl.

Micro Pathology

Treatment Discussion

DIABETES MELLITUS TYPE I1 (ADULT ONSET)

A 45year+ld woman presents with a swelling in the anterior

portion of her neck.

She also complains o f slowed speeclt. easy fatigability, and cold intolerance. She is known to have rheumatoid arthritis. for tvhich she is taking NSAIDs.
Puffy face; dry skin; coarse ha& swelling o f thyroid gland in anterior portion of neck; swclli~ig is mol>ile with drglutition but not witla ~>r+otrwion of tc>iigi~e: thyroitl has ruhhery ronsistencv: right lnhe more enlarged than left; swan neck drfornlitv of left ring finger: ulnar deviation of finger5 of hot 11 1;tancls.
Labs

T,, T, law; TSH high: antithyoglobulin antibodies and


a n ~ c r o s o m aantibodies l (,WTTI~D I )I~EKOXlU~LSE I NIBO~TFS)

dctectcd by ELISA.

Imaging
Gross 'PatholLogy

Difliise, moderate enlargement of thy-oid ~Iancl: cut surface i s light grav and appears similar tc) a Iymph node.

Micro Pathology

Biopsv sl~o.rtls mawive inti1trat inn by lymphocytes and plasma cells: ~ ~ n r t n a follicles l no[ present: scant colloid: rosinophilic Hiirthle cell degeneration scrn.

Treatment
Discussion

Hashitnoto's thvroiclitis is often associi~~ecl with other autoimm u n e diseases, i tlrluding svstcmic I~tpus er).theniatosiis, pernicious anemia, Sj b g r e n ' ~ spdronie, and chronic hepatitis; it has a genetic association wit11 I-FL4-DM (goizrous f o m ) ancl HM-DRS (atrophic fclrrri ) . Thvrotoxicosiq may he seen early in the course of this autoimmune disease owing tu inflammatory disruption of' tt~yroidfolIicles ( 1 1 ~ ~ 1 1 1 ~ ~ ~ ~ ~ ~ ~ 1 ~

) .

HASHIMOTO'S T H Y R O I D I T I S

ID/CC

1 2 ?+veal--old ~ v l l i l r l i ' t ~ ~ ac lr o~nt eu~llcr Llrr~ilp cloctor hccacise

oCweight loss despile having a good appetite: s h e also complains of incrci~uiilg anxicty.
HPI

Shc admit4 ro 1i;lviug Crerlncn~ IIVLII.; d diarrhea. rccl~lced deep c;xp;>r i ty. hcat intolerance, su.e;l~~ palms. palpitations, tremors. a n d menstrl~al irregularity.
i'S: tachycardia. PE: ~ r ~ m o rof - soutstretched ha~lrl: warm, moist skin: right lobe of tl~yroid palpably enlarged; lelt lobe not palpable: 110 ~ ~ i r l e n of c c rr~roctt.rn;~I goiter; no c ~ ical r ~

PE

Iym pt~;lclenopa~hy.

Labs

Xncreaqed T, ( I o I.U. lJr-\w.*l -nrlaoxrh~) : irlcr-eased resin ~riiorlotl~vroni 11ei ~ akr p ~(l<T :L') : decreased plasma TSR.
XTl. qol't ti5sue: n o calcific;~~ion in area of thyroid. CSR: na merliautinal nl;l?s. N ~ i c hypcrfwnctioning : hot (inrreased uptake) I Ii? r-oirl nodule wit11 decreased lrptake irl sumttnding tissue and right other lobe (dtle m atsophv of I t.r~iait~rler or gland cecondilr?' IO Fthedhxck inhibition (11 T'S1-i).

Imaging

Gross Pathology

Srnoolh. r o t ~ t ~ d ell-circiiinscr-il~rrl ~d, sitlgle mass i r l r i g h t I d e of' ~liyr.c~irl ~1:lnrI: no arras of' heinol-rhagc 01- TIPCI-OS~F:remainr l r s oi'glru~d alrr~pllic.

Micro Pathology

No

r i p s of'ahpia: fcrllict~l;u5rr.oma with al~irnclatlt. normalaplxnt-ing cnlloirl.

Treatment

Trca! tlimo~ I ,uiru~if wilh psopranolol and anti th!roirl ~neriicalion4 (e.g., propylrhio~~i~aril ant1 mcthirrin/cllr): al~lation of :~rlent)ma hv ui ther r.adioactive iodine or. st~rgel-y.
Plutntnrr's n o d u l r is ;j variant or toxic r~odt~lalgaiter in which h~pcrtlzyroidisniis cauwd Ily rwerproduc tion of ~ l i ~ r o i r l hurnlone hv sl single thrr.rli(l arlenoma known as toxic adenama.

I
Discussion

Atlas Link

n - C l I PG-PI-054

HYPERTHYROIDISM (SOLITARY NODULE)

ID/CC

A -1&veal.-old white nlale visits his family doctor ~ ~ r t ~ p l a i n nf iilg impotence and f a t i p e over thc p:lst ycar along with decreased peripheral vision.
I-Ic also cornplain< r>f'decreased appetite anrl cold intoIerance. L a r l week, Ile wmtcherl hnth sicles of 11iq rat. wl~ile driving thl-oltgl~ a n illlcy.
I T S : hypotension

HPI

PE

decl-r;~cerl~4CTI-I). PE: pallcll-; bitemperal hemianopsia on visunl field trsting; op(ic at rophs: loss uf itsillarv anrl plll)ic t1;iit.: increasingly sparse beard; s ~ n o o t lclry ~ , skin; testicular atrophy.
{C~ILP

tc)

Labs

I .vteq: livpnnarrernia. Low FSH. LH,TSH.and ACTH: col-rey>nnclir~gIy Ir lrz- TI,T cortiwl, cstl-oxrn, ai~d Lesloslerone; hypn~lr~ccrnia.

,.

Imaging

CT/MR: pitttitary maw c o t i ~ p . e s s i i irbp~ir ~ ; chiasrn. XR, skull: wid~ili I I 01-s~lla ~ tui-cira.
Cornprcrsien of optic chia~rn and Iiypotl~iilarr~~lr hv pituitary advnr )ma: rnilv undei-go inlkrct ion : xtropl~p oC thy-oid. testes. ai~d adr-enals.

Gross Pathology

Micro Pathology

hcleiioma oTchrntnophohr c ~ l l with s nhunrlant cytc~plas~ri lacking R ~ . ~ I I I I I ~ P S .


Sru-qical resectiorl (u-anss~henoirlala r l ~ n e c t o ~ n y and ) hurmonc repl;lrenren I (g111corr)rticnid~. levorh\-~-?-o.iine IT ,I. trstostcror~e) .

Treatment

Discussion

Thc most common


ct ,m prrwirlfi the

causr of liypnpi ~ ~ ~ i ~ i ~ r i l ; - adenama isrn antel-ior pi tuital-v. C) ther c o m ~ ~ cauwq ~ c ~ n;krr ischemic necrosis of t1.w ;In trrioiA pitui ir 1 p r )sipar?u~n fcrnales (51-IF.EI-r siu's;S \ N ~ R C N I : ) , p i ~ ~ ~ i i n slil-gel-y/snrliation, rv and.

MYPOPITUITARISM

ID/CC

A 6-yr.ar.oldt>oyi s broaigllt in for a pedi;~tric consultazior~ dr~e to a lloar-se voice. growth retardation. and developmen ral delay.

HPI

TIlr hov\ nnlrtther dcscrihes a prolonged gestation and a hirrh ~ueight nf 4.5 kg. The hov has had pr-ohlelns at schnnl owing to a slloi-1 ;~ttcntion span. sleeping in class. ;in$ mental sluggishness.
Dry yellowish skin; wide-based ataxic pait; large tongne ( ~ I ~ ~ C : R O ( ; I . O S S: I nl~lcc~llar ,~) atrophy: short s t a m for age: I3roarl nose: ~~rnbilical hernia: p 1 1 6 tvcs (clue to rnyxede~na) and wide rpicanthal rlistance: slow relaxatioi~ of tcndon reflexes: thin,
britt[e hair; protnberant abdomen: weak, hoarsc voice.

PE

Labs

Elevated TSH; low T , and T,.

Imaging

SK,plain: ah<encenf some ossification rentel-s; coxit vara


( 1 ) 1 ' ( : ~ f : ~ k s t l! .f ) : . ~ o K . i I ..iut:r.l.:) : delaycd rpiphysck~l developrncnt.

Cross Pathology

Enlar.jicd thyroid gland; myxedema; failt11-e of sexual organs t n develop pn~prrly.

Treatment

Levothymxine replacement.

Discussion

Congenital hypothyroidism in an in fan t rbt- child leads tn irrcvcmiblc mental ~etardation: it is c a ~ ~ hv ~e lack d of iodinr, rhrroid ~ l c r ~ c l o p m c ~ clcfects, ~ t a l radioactive i o t l i ~ ~ expowre e during prcgnanq. antoimmune disortlcrs, ancl drugs. Prolean ~n;lnile.;t~tions incl~~d netiromusc~tlar e itnpait-mrnl, .;hot-t stature (dwarfism). cardiovascular s y ~ i p ~ o r nand s . sexual retardation: i t can Ile mistaken for Down's syndrome with grave corlreqllences. A11 states in rht=United Stater r~lrrently rcqlrirc neonatal screening For hi~ochyroidism. g:rlactosernia, ancl

F*"l

HYPOTHYROIDISM-CONGENITAL

ID/CC

r\ 48-vear-nlrl white female cnrnplains of progressive weakness, Ietharg. and cold intolerance.

HPI

She also compl;~i~is, uf weight @ n , constipation, coarsening of her Facial features,Tiair lms, and inrl-easing hoarseness in her \wicc. Sllc adds that h e r pcriod~ have hecome irrcplar a n d Ile;~w. Her huskan tl ~ l o t e s t hill che liac hecome iucrcasinglv fclr:c.l fill an tl depressed.
VS: hratlvcarrlia. PE: ct~drsr facial Scat~ii-rs; perinrhital erlema; vt.llnwisli skin that is r o ~ ~colcl, ~ h . artd drv; brittle. thinning hair; loss of hair of outer third of eyebrows; car-dinmeply; enlarged 1 ~ I l I g F l V ;delaved recovery phase o l ' Achilles tendon reflex.
Low T, and T,; elevatcd TSN; elevatcd suruml cholesterol.

PE

tabs

Imaging
Micro Pathology

Kuc: In~vsarlioactive iodii~o u171akc: ( RtIlU) l>y lie thm-oid.

Myxoirl cl~generatio~l nf conucctivt. tissac, Iiarl>orii~g isolatrcl areas nT atrophic follicle; Ivmpl>oc~lic ii~%ili~'alt* Fern if I-Iashimota's th!~oidiris ir thc utiologv.

Treatment

Lwothyroxinc I T , ) replacemrnz

ilre Hashimoto's Qiscussion The most conlmnn cduse.: of l.~ypo!hr~oidi~mn thyroiditis anrl hylwl-thvroidism that has hecn rrc;~tcrI with surgery or radioactive iodine: it may he primary ( ' E l 1 is high) 01- secondary (low pl-orluctinn n C TSI I h~ pitui tar!,).

"

HYPOTHYROIQISM-PRIMARY

ID/CC

;I 7-year-old male is bt-ought lo a phvsiciai~for an evaluation of precocious puberty.


H e has a history of severe headaches and visual blursing.

HPI PE

Fully developed secondary sexual characteristics (Tai~n er stage W ); paralysis of upward gaze {P.+RINAL L) S Y N D R O ~ I E ; )co~vcrgence 1-etrartian nvstagnlus; hit~ci~~~uscnpy rcvcroals bilatcrd papilledema: wide-ha~ccl gait.
Norrnal
lit13

Labs

pal-arnetel-s.

Imaging

MR. head: obstr~ictive hydrocephalus and brighfly enhancing mass in region of pineal gland. Microscopic pathology revcals tiirnor. to he of germ cell origin
(C.L:R~~I\;OC~.~).

Micro Pathology

Treatment

Neurosurgery. removal nf thr tliuioi- ni- h u n t placement, ancl radiation thempv are the mainstavs or treatment.

Djscussion

Piileal region Lnrnors include pineocytomm and pineob~astomas dcrivcd Froin the pineal parux~chvtllalcells as well as teratomas and germinomas: p r e c r ) c i o ~ pubertv ~s occurs in young males, [~".im:irily as a I-esul4ol'destruction of the pineal gland by a germinorna. Pinralotnas mav prodlice h~pothalamic hormone dcficiencv. leacling to I ~ ~ p ~ l ~ i t ~ ~ ior t ~diabetes t - i s r n Insipidus.

PINEALOMA

TD/CC

.A 33-vcxr-olcl whitr friil;ll~131~ ' s ~ ' i will1 l t ~ menstr~ial cycle irregularity t\'itli Forlji prriorlr, c ~ amenorrhea l anrl miiky nipple discharge (( .\I..\I r ~ )RUT r 11:.\).
Fk11,thrtq11es1ior1i11~ rliurlnscc th:~t~ h c cnnceive. alhc~ been ttnablhl~to

HPI

PE

1 ' s : RP nornm;ll. PE: tlo gvnccr)logicaI rn,lrsvc p:~lpnl,le;prI\-ic


rX2lTl

TIC~T-~I~;~~.

Labs

Ryperpwlactinemia: reduccd LH and estradiol.

Imaging

R f R: enllanciilg pitui t a w m i c ~ - o ; ~ c l c ~ t o(r< n : I0 ~ nlrn) : rlcuiiizint~ r > Tpiu~ir;u-v htalk.


Rrornncripfine ( [lopa ln i ne :ui;llr)g) 111ill11 i hit pt.r>l;ic~in 'cvnlll~\is ~ n r r.ele;lw l (:lnrl tc, 1-ctluce sizr o f lal-KCI I ~ ~ I ~ I rcu - . ; \sitlei. ): rrnnwpIiri~oirl.lIS L I I - ~ C I - \in p;ltic'~i IS, ~ ~ I ~ o c I 11111101.5 c' r e i ~ i i l i ilarge ~ rlespitc h1-miir,tripti~~t. ~ l > c f i \i l~ l ~ 1l111.ir. ~ : !\.\lo ratinn1 t o l ~ r a t c t l o ~ ~ i ~ i ~ i ii g io i ~ ~( ~~ i \;1i1(1 ~ < ,!TIpsvc-l~~atric p;~t~c,~ I<IIO ~ t h rrclilii-i* rlopxminc~ a11 tagoil i ~ t c .

Treatment

E + t tic. m o < t ron1Int)n n.ptbI I C pituirai-J arlur~c~inii. D ~ S C U S S ~tYi.ol;~c~inoina O~ I-l!.pr )I hnlnrnir Gu Kt I i s F I I J ~ P I - C S S C h\ ~ C ' X U C ' S ~ ~ V IJI-C>~;ICI C iti srcrt~tioii IIT 1l1c t t ~ i i i c j i ; xnd 11iur LH ; ~ t i t lc%rtl-:~rlic)l art- r r c l ~ ~ r e c l . In males, i t I>~-PFPIIrc wii 11 headache. impotence. a n d vis~rd distnrbance. I'ashnToqir l~ypct pl-olactinc-lnir~c;In ; ~ l s r ll ~ c;llr c serl In. thf. in t ~t- upt ti on of' Irvpn~hat,in~ic rlc~p:umiiicrgic' c n p ~ ) ! ~ ~ i c ~ i i o f pill~ili~i-? pruli~ct in 1~1eiqqr ((-.q,.\ ~ t p l ~ ; ~ w lIII;I<WT. l:~r pitt~itar~, stalk cccrioi~. a n d drug5 such ac h:*lopcridnl, phcnothiiviilr.u, 31111 tcsrspinv 1. Tht~s, 111iltl li~l,cr~l~i.ch;1r1it1e111ia rloes nnt alwnv~ sig~~if ille t ' pl-esciic~ 01; 1 uc~oplnsin. Elcvxtrd c><tl-ogrn ( s l i ! ~ ~ ! i i nI la r~ ~otrc)p!~ itlid c ) rcn:-11 in fl~f'ficicncy drt. :tdrlitic>r~;~l c ;IIIYC*F 01' I l \ ~ ) ( ' ~ l > ~ i f~ i ~~ ill i ii; l t. ~ l

PROLACTINOMA

rZ 40-vear-old woman complains of an unsightly, progressively


increasing neck swelling and intermittent shorzncss of breatli.

She has ~loticerl unusually engorged neck veins anrl ha7 recently d~veloped difficulty swallowing solids (nmrr1.4c:m) a n d loud snoring ( m m n ~whi ) lc sIeeping. 'I'llere are no symptoms of hypotllpidism or hyperthyroidism. She uscs iodizcd salt
(iorline deficiencv produces endemic goitcl-).

PE: an terinr. irrepllarly surfaced swelfing that moves with deglutition (NUI~TINODULARGOITER) ; percassion over slerl-turn is d ~ ~(rlile l l to 1-ru.o~terna1 extei~sion of goiter); suff~xsion of face wiih marked dyspnea when patient raises both arms overhead ror a ~ P W secotlrls (due to tracl~ealcomprcssion): no tremors or eye s i p ; no cervical adent>pathv.
Labs

T , ,T , normal; TSH elevated;thvroid autoantihoclics absent.


CXR: remsternal extension of the goitcr. producing tracheal comprcssion and deviation. US/CT: diffuse multinodularity.
Resected thyroid grossly enlarged;sr~rfacc covered by nodules of varying sizes.

Imaging

Gross Pathology

Micro Pathology

Follicl~s distended with colloid: Fdlirle lining cells flattened; degenerative changes present in betwren nuduleq.

Treatment

Subtotal thyroidectomy: thyroxine subsequently administered tu suppress TSI-I levels and prcvcnt rrcrirrence.
The cause of enIargement of the thyroid is inost often unknnwn: known uauses include iodine deficiency (in endemic areas). ingestion of gaitrogens (e.g.. cabhaqr, cassava), W ~ ~ Pp~llutants, I or defects in thc svnthesis or transport of her~nonr. In addition to trachcal coii~pression and dvsphagia, subsrel-nal gnitrrq can nlqo callve phrenic 01- reclwrrnt 131-~m.ngeal nerve palsies, and Hnrner's synrlrome. esophageal varire~,

Discussion

Atlas Links

SPORADIC MULTINODULAR GOITER

ID/CC

50-year-alrl female prcsunts with a nodule in the front of her neck that he first nnticed 1 month ago.
r \

HPI

Slip nolrs that tlw norlule has grown. hut shc docs not complain or allv ynptntns strqpestive of a l~!.perthvmid c>rhvpolllvsoid state. S l l r consumes ioclized salt. She works a q an x-ray

technician (rarlia~ion expmure).


PE

Firm, nontender nodule in anterior portion of neck, mobile with deglul ition: ;3n IP~-iar cervical Eyrnphadenopathy; no tremors. sweat ina, pre tihi;lI/perlal r n y ~ d e m aor . csophtl~almos.
Normal thyroid h~nction tests; normid thy-oicl horn~one lel~els.

Labs

Imaging

XR. rirck: stippled calcification. Nuc: cold nodule. LIS: solid nodule.
h'orlzllc can range in sizc from rr~icroscopicto qeveral centimct ~ r nit11 s inmsivc margins; may: he sclerotic or partlv cvstic.
m

Gmss Pathology

u z
0

Micro Pathology

FNA: psamrnoma bodies; Iymphoc~.tes:large pink follicular cells will? emptyappearing nuclei ("Orphan Annie" r~rtclci) atid cosinophific ilicranucIear- irlcIusinns.
Ip.rilat~r-al lohectomv and exploration of regional lymph nodes: follow up Ir\-cls of sermrn thvroglnl~~~liii :levntl~~-oine s~~ppre~~ion.

6 G1

<
Treatment

W scussion

Ionizing radiation i s a 111-eilirposi ng rnctnr Tor the dc~,c.lopmen t of papillary carcinoma n C rhe thmnid. Papillary carcinoma spreads 1 i ; i a the lymphatics and r n ; q prcscnt with only cervical I v r n p ~ ~ ; d e ~ c ~ p ;and l i h ya n orcull pritnary, In cnn h s t , ColEicular
a tlrl sporadir medl~llary carcinomas commonly muta.;tasize zia tl~e hInnclstrearn. Of all histolo@c variants of I hyi.aicl cancer (papillary, follicular, maplastic, medullary), p?lpiHary carcinoma carries the hest progrlosis and auaplastic carcinoma the xvorqt. MrcIullary carcinoma r ~ the f tht~oirl i.< deriver1 frnm parafolltcular cells (C cells) at1 d i s rrlnst c c ~ m monEv uporadic, h u t it mav also orrur in I'amilial Corms with multiple endocrine rieoplasirr (MEN) tv-pes IM and IIB.

Atlas Link

lZlm

PM-PI-061

THYROID CARCINOMA

ID/EC A 3-vcnr+rIcl h i m f r n ~ ; ~drvelopu lc sudden. aoutc- pain arid loss


of vision in llie right eye af'ler-TCIIchinx a ~ e r i r s nf in a dark mom.
HPI

r;l liiily FIICICS

Slip harl heen ro~nplaining or sering "halos" around lights ;11 nig111. Injccrir~nEduu lo r.asudiIation) of ciliary anrl cr~r~j~rnc-tival hIoorl vessels: b a y cornea: lovs of pel-ipl~rr;~l vision: markedly elevated intraocular pressure; shall(>wan tcl-ior ch:~mI~clwith pcriphcsal iridoctwncal cont~ic1 hv slit-lamp cuam; pupil.: rriid-clilated i ~ t l d ~lnrcspnnsivc to light and accomrnoda~iclr~: I~v-perernir. ai~d cdrmnluus nptic ncrvr becl on tl~rrcluecc~pic euani. Pathologically narrow antc7-ior chambcr: cpr hypvpuropic and rock-had in trmsi~trncv; ~yncclliafcorrnaliun: Schlernrn's canal 1113~ he I~loctrecl.

PE

Gross Pathology

Micro Pathology Treatment

L>egrnrr-;l~iot~ ;ind fit>rn<is

t~-:~t>et*ul:~~.

halgosics, J 5 ' ncctazul;-lrnidc: topitill krtit-hlrrckei,~: ~t~t-nid~; pilrtcaiyinc.: Iaser iridotomy.


tlcutu anglc.-closurc glaucoma i.i charactrr-irerl a sticlden inCI.P;+EP in ~ n t r i l ~ r t ~pr.eFsure lar that rnav 1~ precipitated by mydriatics anrl I I ~ O I ZIcavit~g dark environments frjr wrll-li~iit-eas.

Discussion

lCCl ACUTE ANGLE-CLOSURE GLAUCOMA

ID/CC
HPI

A W~-vear-r>ld woman ptweizrr with a ~ ~ ~ d r lserrerc e n , aitack of wrrigo associaterl with narlsca ancl 7.omiting. Her smptclm\ hegin and are aggl-avaieri rvl~en 511c looks toward tl~e I-ight.Tlle allarks lart 3e3s than 30 seconds. Shc lias nn history of hrariilg loss. pal- di~chal-ge, finnitus. trauma. pain. or restr'icterl neck inovcment.
S y ~ l l p t o nrrcuil~ when her hcarl is t ~ l r r ~ c ~owai-d d r ighr; rotatory fatigahlc nytagmus hit11 ;i l i ~ ~ rcnrn[>nilent: ar n o hearing Ioss or nnv othcr neurologic rlrhci~.

RE

Treatment

R~;icl;t~i.anr-P artrl lmsiticlr~ing inanc~ryrrqcIcsignecF to clear- drhris Crnm the 13cnterE01.can;~l.

Discussion

Brnign positional I or~igc~ i s sc)tnr~it~~e< sreu after !lead injuries. em. oprr-aliens, or infections of the ~nirldle car: it is thu~igllt to I>P rl\lt= IC) free-flnatin g o t o c ~ n i : ~ clrhric I in the poqterior semicircular canal. I t tyically abates spor~ti~nrouslv af [PI- a few weeks o r montl~s.

B E N I G N P O S I T I O N A L VERTIGO

ID/CC

:<~i-vparcrFrl mate sepn afrer 3 roadsirlr accicl~nt presents with a persistent bIoody but thin nasal d i s c h q e .
;2

HP I

Di~~ectccl qurutio~iir~g reveals that he I>asalso lost his sense of


smell sinre the ;iccirlrnt.

PE

Watery nasal disrl~ara~. noted; hilateral periorbital hematomas ("hl;~r-k e v ~ " reen; ) anosmia fo11tld o n nt~urologicwarn; senlainrlcr of phvsical exail1 normal: on placing a rlrop of nasat discharge c l r l c l ~ ; l n w l ~1 e i Kalizr, spreading yellow halo noted in addition to central blood stain ("HALO S~GN";due tu presence nf

(:SF).

Imaging
Treatment

CT, hcad: fracture of cribriforrn plate.

1)y 30 rIrgr.~es; pa ti en^ advised not to blow his nose: t~~llr-owrrgical rnnwhr Tor pnsrihle repair of
tInril>ic~rics: Iwad enrl elevated

rnrninges.
Discussion

Frar1tu.e~ nf the h a w of tllc qk11ll involve the anrerinr nr middle cranial fosqx. Tllnw affecting the an~erinr ros~a. as in this case, may cause nasal hlcrding, periol-11ital llcmato~nas. ~uhconjrinctir.al herriorrh;iges, CSF rhir~orrhe;l. a n d r 1-rtriial ~jer-vr injuries ( { : I \ - I-CN V): middle cranial rossa str~~ctures involving the pebous temporaI bone may came bleeding from the ear, CSF otorrhea, bruising of the ear over the mastoid { " B ~ Y I T L .SIGN"), ~ and cranial nerve i n j u r i e s (IX kTr4:n" \;Ill).

ID/CC

A 50-year-ld male cum plains c ~ hearing f loss and a whistling sound in his left ear [~rs\rrrr'%).
Me clai~nq 10 h a v ~ p r o n n ~ ~ i idifficulty c~~l ~mderstancting speech
(0111

HPI

or prnpnrrioii

t n li~al-ing loss). He has also expericnccd

ocraqion:~l vertigo.

PE

Left-sided sensorhema1 deafness: Weher test lateralixed toward right ear; left-siclerl cur-rtc:~lreflex lrlst (CN V rly~l'~itiction). Purc-tone a ~ r d i o m e t reveals r~ sensorinuz~ral hcaiin~ !us\: discrimination of speech markedly redrtced: lo~iclr~rss recru itrnrrit ahwnt; tone decay secn.

Labs

Imaging

CT: lcf't cerebellopondne-angle bunor suggestiw of :~coustic nrllrnnla.


Encrtps~~lated tumor arising o111of p e r i p l ~ r r i(11, CN VIIl ( v t ~ t i l ~ ~cli\'i~iot~) tlii~ a r r r r ~ e h ~ l l o p n n t iangle. t~e

Gross Pathology

Micro Pathology

Spinclle cells wit11 tixhtly intrrlacelf patltrn (XYTOVIA) itnrl Vemcav bodies.
Surgical resection c n n t i w .
m

Treatment
Discussion

The~e benign tumors arise Crnm lie riislal ncusilcmmal portion of rhc cightIi n c r w . usually FI-om thc v ~ s ~ i l ~ divi~ion. ~ ~ l i l r and arc corrrctlv callrd schwannomas: tflcv account for 80% o f cerebellapontine iumon, hmmlriir n w l - o m a s can he s~~ccesrfiillv rr.mo~c.cl, hut cranial ncrvr palsicc s11c1i as CN 171 nur\.chi111t1 deafness arc- cornrr~u~~.

< 0
I --I I
TI

3 z

r Q G1

<

ID/CC
HPI

,4 3~bvcarnlrlmale cn~nplains r~f'sudclcn-onsrt dizziness, nausea, vomiting (nonprojectile), and Ioss of balance.

He a I ~ o complains nf hearlache anrl bl11r.t-ed vision (drie to nystngml~s). H e has chonjc suppurative otitis media (CSOM) of lhc. right ear. T o r ~vhich h e has [ilken treatmen1 it-r~glilat-I?;.
Patirr~t Iring on left ear ancl looking inward right ear; condueti11e deafness: 14Tcl>er l ateralilrd lowarrl right ear) in right ear: hori~ontalspontanenus nystagnius tow:rrcl Ieft: p ~ ~ r l i leat e~i tlis~ cllargc from riglit ear: ilcl neurologic cleticiu.

PE

Imaging
Treatment

SR,mastoid area: obEiteration of mastoid air cells on right side.


Sm-gical c~plnratioll of n~astoid; anti1,iotics and ve.;tih~~lar

sitppressantq.
Discussion

Rwg~nic inflammatinn of the lahbrinth ma? result from acute otitis incdia, operations on the s t ~ ~ p eosr. prcformcd pathways such as fracrurc lines: in CSOM. cholcstcatuma may ciiuse er.osirm of tlic semicircular c;ir~;ilu, cxposir~g the 1at)vtinth in inlections. Menitlgitis is a wrious co~nplicntion of suppl~ratire lahn-in thi tis.

"

LABYRINTHITIS

ID/CC

: I I4ri-yeill--nE~I obese mat1 present5 with excessive daytime sleepiness that ha5 progt-cssircly worst-t~cd uvrr- rhr past

3 vear-s.
HPI

Hi.; wifr corrlplain3 that his snoring ran I>e heard in t h c adjacent room and t h;lt lie in t e ~ . ~ n i ~ t ~ appears ~ i t l y to stop I~rcathiiig rl~lring 111e~ l i g t ~These z. "no-breathing" episodes last 3 1 3 to 90 sect)nds. :und thcn. with ;I loud snort, he I~egins lo breathe agairl. 'She patient a l ~ o I-epol-tsLitigue, forgetfi~lncss, iinxiery, morning headaches,and rliminished sex11a1 ~ n t c r r s t .
'

PE

I ' S : mild hypertension ( 1 10/1)0). I'E: short, thick neck; deviated nasal septum: pharyngeal crowding with enlarged, floppy uvula, high-arched palate and sort palate resting on base of tongue.
Overnight pulse oxirnctry rrvcal.: frequent episodes of arterial 0, desaturation; polysornnography ( i n c l ~ ~ d i n EEC;, g ECG, rve rncjven~eni, cli in Inovvemenr. ail- flow, c h e ~ t aurl al~dc~minal effort, SaO,, snoring, arid leg mnvemenr) diapostic.

Labs

Treatment

Weight loss; avoirlancr oF alcohol arltl eedatives: naqal CPAP or Ri F,\P; phar-macolhrmpv wilh prn~rytilit~e; surgical interventions i ncl~xrEe ur~~lnpalatopharvngoplas~ (UPPP).
m

Discussion

Fathophvsiolugic311v.Il.r~ilsopharv~~gual c r o w d i n g creares a critical 3 0 ~ 1 1 T ~t;~ t ~ i n ~ ~ ~ l i~ e~ i-i Pc S-S ~ I cItu-ing I.P ir~qpitxtinnthat overcomes thc -0 al~iEih~ of thc aii-~cav dilator and a h d ~ ~ c l muscles or LO r ~ ~ ; i i i ~ t ; ~ i ~ i z I 1 nirw:~ patcncy. T11is causcs apnca. Ieadirlg to hypc)semia thar t - v ~ i i t i ~ ; r l l,v~ . o l ~ tire ~es jxltirnt Irotn 4erp. 112 patients with o l ~ 3 str~~ctivc slccp apnea, thcl-e is an increased incidence of coro2 nary events, W h ,anrl right heart failure.

OBSTRUCTIVE SLEEP APNEA

ID/CC

A 44-year-old black male is referred to thc ophthalinoIogist forevall~ation af progressive and paidess diminution o r vision.

HPI

I-Ic I~as 110 known drug allergies and rlpnies use of stel-oirls.
VS: normal. FE: ophthalmology exam reveals tlormal visual acuity with markedly reduced periphelal field of vision; elevated intraodar pressure on tonometry; increased cupto-disk ratio with optic atrophy o n ophtha1rnoscopy:wide opcn a~igle noted on gonioqcop):

PE

Treatment

Relief of intraocular hypertension wi.rtki topical beta-blockers (timolol), ~ t ~ iics o t (pilncarpine), ur prostagla~din inhibitors with or without sm-gical proctdlrres ~ u c h as laser tml~eculoplasty~ trabcoulnlomy goniotamv, and tra~~eculectomy.
Open-ai~gle glaucoma i q the most frequent cause of vision loss in the African-Americanpopulation. Risk factors i ncl~tde diabetes, nearsightedness, ;ind long-term steroid usr. PenpIc with first-degree relatives w i l h glaucnm;~ art. at incl-~ased risk. e'nfortrina trl!~, IIIP disease is usually br advanced wllcn svmptoms arr first noted. Prcvenlion i q through carly delectioll with eve exams once evt.1-y 2 years nr more fsec~lrently for thosc at incrrascd risk.

Discussion

OPEN-ANGLE GLAUCOMA

ID/CC

A 60-).ear-old male cornphi 11 5 o T progressively diminishing hearing amity over the past Sew years.
T l ~ paririrt r '9 Ilral-ing Irlqs is bilateral and is aIrnost ihe same For both ears; he ha? nt, l~istorv of' ear disch;~rge.Ifnrtir~rs, or trauma. Al2ili tv [so diutinguisl~ brlween consonants niarkedlv impaired; air cond~iction exceeds bone conduction (clue to scnsorinc~~ral hc;wi!lq Inss) : a ~ ~ t l i o m e treveals ry bilateral hearing loss in hiqher-frequency range.

HPI

PE

Micro Pathology

T'rc.;l>ycuhisis cl~:oacferizedlw a lo^.; nl hail- cells, atmphp oF ~ h t \piria! g ; ~ l ~ g l i o alrered t~. r~~clolvmph prr?dt~ctiori. mld ~hickrning 01' tllr hasilar rncinhra~lr with sonic n e u ~ ~rlcgeneralion. al
Counseling; some hclp cuulcl bc u h t a i ~ ~ froin r d a hrat-ing aid.
I'r~sh~~cusi% is a t v p ~ oTwnsnri~~c.ural hcarin~ losq tlrat result^ li-a~n the apjng process; dcgcnccativc chaiigrs occ~lrin lhe cells or t fie organ of' Coi-ti and nrsve l j l , v ~ - Deafness ~, is hilatesal and s~minutrical, cominr>nlyaffecting the high itones. Orher h ~ e of s t>~.e~ Ch I\ IS~ i llrl11~1~ ~ strial, which starts i11 rhc fourtli ;i~id vixrh rIrcarlc~, is .ilowly p r t > ~ r c s ~ i and v r , is rhxtxcrerizert 1~ goocl di\rr.imio;~~iota a n r l 12). the prescuce or recruitinrni, a flat or drscending audiogram, and patchrr atl-r~phy of shr middle and ;~picaEturns o f the 5tri;b. Cochlear- cleafnew Ileginq in micldle age a n d i s oL'1he cc)ndurtive varien: 4rmving a clo~mward slope on a~~diop-a and m ;ll>suntpathologic findirrg<.Both tves of sr~rsnrincural loss ~ r Ijo n a~+.oidt.d tl~rougli use nf pmtrctinn in hifill-noice arc;l% ;irlrl moni~oring of ototosic- d r 1 1 ~ s .

Treatment
Discussion

rn

< o
-u 3
--I

% 3

k <

PRESBYCUSIS

ID/CC

A 45-year-old malc is sren with complair~ts of' blurring of vision while reading and performing similar tasks involving near vision.

HPL

H P cnrnpfai~lsthat he has to Ilold the tlewspaper at an increasing distance in orrler to read it clem Iv. T-Ic Ilas hacl n o pret-io~is prohIcrns with liis vision i ~ n d has no historv of di,ibercs or hyperteaision.

PE Arnplihade of accommodation reduced: convex lcrls ruduced


nl.ar:pnint dislancr, nlln.r~~ing patiel11 to rear! co~nfortahlv and erlgagc in tasks requiring near vision.
to

Treatment
Discussion

Convcx lens glasses uswork rrquirir~g near vision.

Presbyopia i q natural loxs of accommodation due to sclerosis of the lens substance, w h i c t ~ fails to adapt itselFto a more spherical shape when the 7onr1le is rclnxed in thc accomnloclatiotl reflex. Preshvopia is Teen in middlc-aged p;ltitnts (niean ;lgc 45 years).

PRESBYOPIA

IDJCC

A Pg-yt-ar-olrl woman visits a clinic rt-itEl cnmplaints of vis~~d blurring.

HPI

Shr xlsa complains of headaches that are worse in the morning. She has hren taking o r a l contraceptives fnr Fnme time.

PE

VS: EP nol-nral. PE: patient is obese: f~unduscopv r~vt.aIs prcsence nf papilledema; nn focal nrurnlugic deficit noted; remaitlder ell Pxam 11orlnal.

Labs
Imaging

LP: clcvated opening pressure; CSF normal.


CT: vcntriclrs ~lorm-nl. increased voll~me of a~harachnnid spacr+. h g i o : rulrs otil duml sinuq tht-omhosis.
S ~ n oral p cont~.aceptives: advise diuretics and ol3e.ii~y-~'ecillcing mcasurc7. If ~ n u d i c a l truatlrterlt becomes inadequate. surgic:*l options s l i t l ~ aq s h u n t plareme~it are used.

Treatment

Discussion

Renign intrarranial hypertension is primarilv a clisr:isc of obese females: its etiuIogv is t i n k n o ~ nalthoug1:h , associations exist with Ihe EIBV o r cerlai n dl-11~9 (oral cr ,ntraceptives, steroids, ni~lidixic arid. t~trarycline)as well as with prcgnrlncp previous head in jury. durn[ sinus thl-nmhosis. and cxccssivr titamir~ , 4 intake.

PSEUDOTUMOR C E R E B R I

IDJCC A EGyeardd male i s referred to an ophlhaltnoloffistfor an


evaIuatiun of a progressively constricting visual fieid.

HPX

The hoy contpiains that hc sees as ~ h o u g h he were luoking through a m o w tube. I)irectecl questioning reveals that he has a long-stariclirlg history of night blindness (clue to loss of rods). His parcnts, although normal. had a consanguineo~w marriage and havc a family history of a visual disorder.

PE

Funcluscopy reveals "bone s p i d e " pigmentation in mid-periphery of frlndt~s, waxy appearance uf optic disk, and marked narrowing and auenuation of vessels: field of vision shows concentric contraction that is especialIy marked iC illumination is reduced.
Heciroretinogram and elecim-oculogram demonstrate reduced
activity.

Labs

Treatment

No salisfacrory treatment; gcnetic counseling Cor prcven~ionor the disease if the pattern of i n h e r i t a r ~ c in ~ a particular farnily can be tracecl.
Retinilis pigmerltosa is a slow degenerative disease of the retina that is always bilateral. begins in childhuod, and results in blindness by middle or adzvnced agc; the degeneration primarily affuuts the rods and the roncs, partic~~larly t h e rods, and romtnences, iri a zone near the equator. spreading both anteriorly and posteriorlv, The condition may be associateil with LaurenceMoon-Biedl syndrome (characterized l>y obesity, hy~ugenitalism, and mental s~~hnnrmality). Rcfsurn'5 clisease (peripheral neuropathv, cerebellar a ~ i adeafness. , and ichthyosis due la a defect in phytanic acid metabolism). and ahemlipoproceinemia. The condition is inherited as an autosomal-recessive trait in 407% of cases, as autosomal-tlominant in 20%. and as X-lirlked in 5 % .

Discussion

R E T I N I T I S PIGMENTOSA

I DJCC

An I % m o n h M boy presents w i ~ h d i s h e d visual acuity and a warrdering right eye that his rnotllcs noticed whiIe waichillg him play with his t o y ~

HPI

On directrd history, the child admits tn havina eye pain at ili~ht.


White ama~~rotic "cat's eye" reflex in r i ~ h eye; t deviation of right eye (STRA~IS~!L:S) ; tenderness in eye or2 gentle co~npression; intraocular mass nn retinal examination.

PE

Imaging

CT/MR. or-hit: lohulatecl. hypvpe~-dr-nrr r~trol~ tal i i (hel~ind lens)


mass: 11o optic nerve compscssion.

Gross Pathology
Micro Pathology

C l r h itish mass behind lens.

S l i e ~ of t ~small, round blue cell% with clusfers of cuhoidnl o r short r o l ~ ~ m n a cells r arranged iirottnd a central It~meu I F I , F . X ~ U - W I N ~ R S TR FO ~I S~ E T~R S).

Discussion

Tlie nonhereditary wriery o f retinohlastoma appears as a single mmor: hereditary fornls occur in carlv childhnod i~rid are often h i l a t c ~ or ~ l rn~~ltiountric. In heredi~iry case.;, patients are at high risk T o r nther- cancers Eater in life (especially osteosrircorna). Gyrogenetic stndies reveal a deletion on chromosome 13 (hand14 o n long arm. Rb gtnc). Rb is a ttlmor. sllpprerwt. gene: the loss of 1 ~ 1 t allelic h copies leads to malignancy ( nun-hit hypothrsis).
PM-PI-073

Atlas Link

ID/CC

A 30-veal--olcl inalc pre5eIlls w i ~ h wdden-onsct pain, redness, and tearing in hi\ Ief~rye. IIc :iIso conlplairls of photophobia and hlmred vision i l l the Icfi
eyr.

HPI

PE

L'S: ~iormal. PE: oph thalrnolngic exam reveals couj~~nctival congestion, diminished visual acuity, normal Gsual field, anrl pupirlarv miocir ~iclch normal reactivih~; aqueous flare with keratic precipitates noted in anterior chaml-rcr on slit-latnp
uxa111.

Labs

CRC: normnl. ESR, tZh7A,RPR, VDRL,, L\mp titer (to rule sy~temic causc-s): 11orrnal.
XJt, chest and sacrt~iliac joints: normi11

out

Imaging
Treatment

Cycloplegics (atropine) to r.clax p~~pillary rphinctes and ciliary musc1c.s;topical corticostemi&: nccasionally immune suppression. Treat ~mderlying systemic illness.

ID/CC

A :'13-vc;r-c)lrl Fr~nalc conlplai~ls ol'increa~ing substernal pain and difficulty swallowing liquids ; ~ n d wlicls ( n w r t ~ c i) ; ~ over the
pax1 ~ e ~ e r months. al

HPI

Shp hxq Imt "0 pounds in t h c past 3 months ancl has occi~siorlallv espel-iencerl acute siibsternal pain ancl regurgitation of rood i n t o h e r rnoutll wherl lying dotvl-r.

Labs

E ~ o hageal p inannmctry rcvcals apcris~lltfc c.sophag(rs; increawd lower esophageal sphincter pressure: nega ti\.e a i l t i nuclear aritiborlic~ (ANAS) (vs, sclerorlcrma) .

Imaging

LTGI: "rat-&ledw lower esophageal segment;dilatation: utiroor-dinatrd prz-iqtixl~ir. I<C;U: aaping cavitv fillcd with dirly fluicl. CXR: air-fl~ucl levcl in enIargccl csophap~s.
( C ~ I I Pto defect in csophagtal perisral~i~ ;~nrl/nlimpaired relaxation of lower esophageal sphincter d11rin~~ swallowing).

Gross Pathology

Mawivr rlilata~ioli of esophapis

Micro Pathology

Los~ of number of ganglion cells in myenteric p l e x ~ ~ (similar % to Hirschspr~~ng'\ disraw ol ~Eie colnti 1 .
I-Ieiler's e~opt~agocardioml~otomv: bafloen dilatation; botuIinum toxin injection.

Treatment

Discussion

Primary idiopathic nc l~alasix i~ x morility clisorclcs uf' tliu cwphag11~ d u c to loss of ganglion cells in Auerbach's plexus. C:L)mplicatiorrs in(-lt~rle esc~phagral squamour; cell carcinoma, catldirlrtl esophagiris. divertic~~l;l, a n d / o r aspiration pnc~trnonia. Secondary achalayia ma\] he cal~serl Iw ChapqWdisease, Iwnphorna, gacil-ic carritrnma, o r .sarcoidosis.

G)

z + ,
o m
rn Z

m w

--I

r0
G 1

<

ACHALASIA

IDJCC

il

34-year-alcl woman presents to the ER with cornplai~~ts af colicky abdomind pain, spiking fever, and vomiting.
She was d i a p t ~ s c d with gallstones oli at1 al>rlomii?dultrasaurld ccvorill weeks ago :111cl is awaiting clectivc .;usgerv.
VS: ftlver (39.4"C):tachycardia (RR 120); zaclavpnea (M 24); mild hypntension (BF 94/60), PE: toxic-lnoki~ig patient: scleral icterus notcd; marked RUQ tenderness w i t h lnild l~epatome~:ly on ahrlominal exari).

HPI

PE

Labs

C:RC:: leukocytosis. Markedly elevated direct bilirubin arid akaline phosphatase with motlcr;itrly clt-vated ASr anrl hI:T: normal afhnmin, PT, a ~ PTT: ~ d blood c ~ t l i ~ w poritive cs Cor Kscherichia coli.

Imaging

US, ahdomen: dilated common bile duct (CRD) with obstructing stone. CiT. abclomcn: gallstones and xtnne in C1ID with dilatcd intrakeparic bile d l ~ c t s .

Treatment

NPO with nasogascric suction; TV antibiotics: emergent cncloscopic (inlcrven lional F:RC:P) or wr-gical ' b i l i q tree decompression f o l l o ~ v ~ hy d lapa~-o~copic 01- open
cholecystectomy.

Discussion

Prolonged cholerlncholithiasis lead5 to


which has a very high mortaliw raw.

st~pp~iri~t cholangitis. ive

ASCENDING CHOLANGITIS

ID/CC

A :ihyear-olrl male presents with a long-standing hi~~or-)r of retrosternal burning, belching, and water brash. especially after

mc.;11 s.
HPI
Hc is ;I chronic smoker and alcoholic and ga$troesophageal reflux dyspepsia.
Phvsiual exam nor~nal
is

under treatment for

PE
Labs

UGl en~loscopv mvcals linear streaks of rcd, vulvcty rnuct>sa at g;lstrocsophagcal jlinc tion.

Imaging

Bariuin swallow: line I-ericularpattern distal to 311 esophageal ~ t r i c n ~ t .gastroesoplzageal e: refl~rx.


Red, velvety mucosa in form of circumferential hand and linear s t r e a k s around gastroesophageal junction.
Mixture or metaplastir:p t r i c and intestinal-t?rpecolumnar epithelial cells {mucin-stru~-ucin~ arid ahsorpi ilre,respectively)

Gross Pathology

Micro Pathology

Treatment

PI-rblonptlmp i~ihibitars, H, an~agnnists. anrl antacids: cessation orsrnmking arlrl alrohol: carer~ll endoscopic follr,wup to detect csoplzngoat u;lIlccr.
Ra1.1-el~'s esopha~l~ issm a ~ k e d bv metapIasia of the distal esophageal squamol~s epithelium to a columnar epitheli~un i n response to prolonged injury; long-standing esopllagr:e;~l reflux lr.;lcls to inflamrnalion and ulcerat ion r l f ?;qlt;lmc>us mucosa. NraIirlfi occt~ra throuyh reepifhe!iali7atiot1 hv pluripotent crTls, \uhirll in ihr selling nflnw pH rlillprel~riares inro rhe more resistant gactric (hot11 cardiac and hindic) wr or the specialized colnrnnar (irrtustir~al)rvpe. Only the cnll~tnnar tvpe is of clinical in~pnrtaiic~. T h e most serious complication is tllc development of adenocarcinorna: Irei-we, patirnts with Barrett'li shu~ild tindergo endoscopic surveillance cvery " t o 3 years, Addi~ional ctjmplicatir~ns of R;lrrett9.; esophag119 may inclucle stricture formation nud ulcerations.

Discussion

3
o A
m
GI

2 -4

o r o

<

Atlas Link

= E l PG-PI-077

BARRETT'S ESOPHAGUS

X 40-vcai+-olcl woman is seen wilh cornp1i1ii-1~~ of s~tclden+nset, profiressiivelv iiicreasirlg abdominal distention and pain and

vnmiting.
TIIP patient also c n n l p l a i ~ nf ~ svisible abdominal and hack veins thar apprar rr,hile s h e is standing and look Like ropes, ln addition, s h r has ohserved increasing swelling of her reet. She has Iwcn laking oral contraceptives f n r a lkw vearq. I(*ter-~ I.;: pitting pedal edema; i~-ial-k~rlIv clisrenrIctl ;~bclo~ncn; rlilatrd, torrnnris wira;s ovrr al~dorntsnand hack: flow is from below upward {clur to hepatic vrin arid IVC r>txrrncrinn): no jl~gtilarvertcrus rliq~en tion ( t h ~ r e l h r nn r right heart failure); hepatqjug~lar reflux absent: fl~~icl thrill hrit shifting dullrlchss II~PSPIII (:1s(-i tes) : il~ildly tender hepatomegaly and splenomegaly.
Labs
Imaging
C X C : : 1e1 tkocytosis.

FAETselcvnt~rl; asci tic fluid transudative.

I T S .abrlnmen: h e p a t o s p l e n o ~ n e ~ aand l y nscitcs. rS,Duppler:


incrcascd p~>rtill vuin !low: hepatic veins obstructed where they empty into the inferior vena cava. IVC pnl-tnv~nograply: confirrr~s ohsl~.l~c~iorz.

Grass Pathology

Thromh(~sis of I l c p ~ t i c vein w l l r s r i r dr;lills into the inrrrinr vt-na cati). Livrr i s .iwollcn and r-edrli~li-pr~rple and has a tensc cnp~tzlr. :Il'fecled areas sllow severe re11t riloh~tlar congralion and necl-o<is along with sinusoidal dila~ation. l]l;illnor~ angioplasty: thrnmhol~~tic therr~py i n t o hrpatit veins: wu-gical scmo\~al or t>>,pass of t h c obstruc tiun: cessation of oral contraceptives; tli~rreticu i11rmg wi I h w r l i ~ u n anrl Illlid r~striction.
Burlcl-Chiari syadromc occurs with corlditions t l ~ pretlispoqe l throi~~hr~sir;, r.g.. polycvlhernia \)era. pregnancy, postpartum slii tes. iiue <IF oral c o t ~ ~ r a c r p t i vpamwvmal ~s, noct~trnal hcmo~ l o l ~ i ria n u { PK l I ) , and in tra-ahdnmitlal cancer?; rne~nbranous rvch i r r t Iir ir~frrior \ - m a c a w niav I I T ~ ~ I ~111e C Pr)f>structitm. U n tl-eater! Budcl-Chiari wncIromc may progress to Iivcr fail~rrc.

Micro Pathology

Treatment

Discussion

- 7

BUDD-CHIAUI SYNDROME

IDJCC

A 4.:-war-olcI malr wllo 1111sl ~ c c n diag~~rno.;ecl with AIDS PI-esenis wit11 pain on .waUowing ( O I I J U O P H . ~ ~ and :~~) 1~1ild difficlllty swallowing ( 1 ) t>~-l.zc ~ an) .
'l'hr~r. wtnptoms arc mal-kecllyaggi-avated hv tlic ir~gcstionol aciclic fluids.

HPI

PE
Labs

Oral thrash.

C:III~III-F o f ~xophagealwasliiny~reveal'; C;u?~(Eirln flll)trrrns


Esnphago~copy:smal I, raiwrl white or yellow pfnques.
Sca ttrrrd yrflowisl~-whi t c plaqlws w i t h r>cci~\ioniil mucnwl 111cet-s.

Imaging
CYQSSPathology

Micro Pathology

L:vroln~ir exarninaiion of I>r~~sli ings reveals prrsencr r,F yca.;1


crlls.

Treatment

Oral llncnnazole.

Discussion

Thr curr-cnl stii-gc in car~didalesnpl tageill in Frctiori~i q due to :lIT)S anrl Irt pn~~-ot~a11-ti.a!i~plant i l n n ~ ~ ~ t ~ o s ~ ~ p ptherapv. ~-csrio~i
1

Atlas Links

I--

PC-PI-079, M-PI-079

C A N D I D A ESOPNAGITIS

ID/CC :2

13-?~e;11--olrl white T e male, the daugl~ter or Norwegian i i n m i ~ r a ts, n complai~ls nf diarrhea and flatulence.

HPI

ITer parents say F'IIC has suffcrcd from weight loss despite the fact that shc cats wcI1. H c r motlier adds that her srool i q foul-smelling ailrl greasy ( m ~ ~ with ~ ~no ~ k~loorl ~ or - mucus. r ~

PE

Pale m i l thin: xrrosir (UKINESS) and l~vperkuratosis of skin (due r o vii amin ,I ~naFahsorption) ; vesic~~lar rash on knr-es, ell~owq, and neck; pruritus with erflematous base ( D Z R M T ~ S IIERPETFORIVITS): C I I C ~ I O S ~ S (SWI.IUG); e r c l ~ y r n o ~ e (due s to vitamin K mi~labsorption).

Labs

CBC: macrocytic, hypochromic ancmia. Lytcs: decreased pc)t~wiurn and calcium. Etevated senun antigIiadin and anti-endomyid antibodies, necreaveti serurn cholesteroI and a l b ~ ~ m iprolnnged n: PT; abnormal d-xplose test: positive Suclan ~lairifor- fecal Tar.

Imaging
Micro Pathology

LlGI/SBm

loss or mucosal

folds ancl dilated jejunum.

I-lallmxrk flattening and atrophy of mucosal villi with kaxophilia ;md loss of nucleal polarity I~qtiphocvt fc and plasma cell infiltration of lamina pf-opria.
Gluten-free diet; glucocort icoids for rerractory rases.
Celiac riiseas~ is a disease of the small intestine that is due to gluten (gliadin) hypersensitivity. It is associa red ui 111 HTA-DR.7 arid Ht.4-DQvP ant1 i s char-arrerizedhv varying clefi-reesof nutrient malabsorption: iron. folatc, fat-soluhlc. vitamins (A, Il. E, R). I t is also knt~ttn as nontropical sprue, celiac sprue. or

Treatment
Discussion

gIt1~t.11-serlsi t ive ~nreropathy.


Atlas Links

TrTI PM-PI-080A, PM-PI-080B

CELIAC DISEASE

ID/CC

\ 1

66-year-Id Scandinavian male cnrncs to the docror's olTice h r an insurance p h ~ ~ i c ctlrnplaininy: al of ir~crra-sing fatipe. occasional indigestion, and diarrhea.
I-Ir has huen takiilg antacids for his du5pepsia.

HPI
PE

Marked pallor; mild splwomegaly. CBC/PBS: mamcytic anemia, macro-ovalocytes. a i i r l ~Ievatecl hypersegmented neutrophils. Low vi~itaminR Ic\~eFs; homo~vqtinc and rneth~linalor~ic acid: ScKiing test conFirms vitamin Blpmalabsorption rnrrpctcd ~ vth i admirlistration of' in trinqic factor: anti-parietal cell antibodies present; reduced ga~@ic acid Formation (~I:I-II.~)KI IVI~I.\) ; biopsy laken di~ring enrlowopv reveals chronic atrophic gastritis wit1 I no rvirIeilce of intestinal rnrti~plasia.Endoscopv: tliirmit~g of mtlcosa ancl Ila~tenina ofrllgal fi~lcl~ e e n nmre in li~ndus ancl I~odv of <tomach rvizli sparing of antrum.
See labs.

Labs

Gross Pathology
Micro Pathology

1.ymphnryes and plasma cell infiltrates in lamina pl-opria; rlerl-eased n ~ ~ m h e or l - gluncfs. Parent era1 arlniinist t-atinn of vitamin R , ?; regular f oIlo\tt-~~l> {c t 11-oniciitrophic giastri t i'i predisposes to gastric carcinoma).

Treatment

Discursion

Pernicioi~s xnrlnia is chararteri7ed hv chronic atrophic ~ a s t ~ i t i s with achlorhyclria arld antibodies tu p;irirlal oclls and intrin~ic Cacl or. TIIP cc~~lrli tion is int,l-e cnnimon alllong o1clr.r ts (nipan age of 60) : these patients are predisposed trJ uther autoimtnu~~e clisol-den, FUC tl as vi t i l i p , I~tp~par;tlFl>~~idi~~~~, atlrcnal ins~ifficiri~iy and thyroirl disease.

Atlas Link

l T I E l PM-PI-081

CHRONIC ATROPHIC G A S T R I T I S

ID/CC

A 35-veatwld alcoholic male presents w i ~ h recurrent epigastric pain that sometimes r:zdiates to l i i h I ~ i l ~ k . Ht: ;~lso complains of bulky, greasy, foul-smelling stool ( r r ~ - ~ o n r ~M t~ e~ has h )lost . 10 pounds over the past 3 m o n t h s ,

HPI

PE

Epiwtric pain on deep palpation.


Qti1111ilntivre~tim:~tfon of far i l l S F ~ Cr-evrals ) ~ steatorrhea; elevated serum amylase and lipase levels. K l l , J)domrri: pancreatic calcification. tlT/lrS: panmatic atrophy and calcil-ication. KlICP: small stricture or l3ancrcatic d ~ ~ in c thead: di4lal panrreatir duti shciwr ~ a c c ~ ~ l a twiih ton
in~cbrvcni~i): short strictures "C,FLZIN 0 1 : LAKES").

Labs

Ernaging

Gross Pathology

Scarreddown, fibrotic panmas with wlii [is11 a r r a r of ratty n e c r o r i s anrl areas of cystic cavitation.
Pancreatic hiopsr rrveals prcsencc of clilalc-ci tll~cts, Iihi.otic .ilt-ulna, ancl x~r-opl~y of csocrine ~ l a t l d s arid i~lels ( d m to

Micro Pathology

en/l;inar ic fhl necrosis).


Treatment
h n c r c a t i c enzyme replacement: low-fat diet: rt1i.ger.y for relief ol i111r;ictahlepain.
Chronic pancrcaritis is a pcrsistcnt inflarnmator?. tliseaw of rhe pancreas that is il-rcvcrsiblu a11tl c;zuses p h i tl anct permnnenr impairme11t of enrlc)csine ancl exocrine C~inction. Alcohol abuse i s tlte rilost conlrriori c ause in aduIts, cystic fibrosis in children.
[=r

Discussion

Atlas Links

T T I PG-PI-082, PM-PI-082

CHRONIC PANCREATITIS

lD/CC

1 1 (51-war-oldwhite maIe in apparent good Ilealth has a routine


annrlal physical e x a m that reveals a small rcctal mass.

HPI

Fle has no niajr~r ronlplaints c x c c p ~ foi- intcrmitteril, rrlild


cliarrhca.

AE

Mabile, nonpainful rectal mass ntl tligi~alrectal rxarri with no e\;idenceof Ideeding: examination othurwisc r111r-crrlar-knkle.

Labs
Imaghg

CRC;: anemia (Hb 9.2/I-Ict 26.9). Bernoccr~It-positive stool.


Sigmr )iclnwopy/i3E: n l i ~i ple l ~ pedt~nculated masses in sigmoid and trarlwerse colon.

Gross Pathology

Discretc illars lesions Tram colonic cpitl~clil~m protrutlirlg irlto intestinal lumcn; ~ a s majority t rnc:!sllrc < 2 cin, a l l h o ~ ~ g may h reacl~ tip t o 5 cln; rtlay hatre qalk (~rnrivt:~'r,~~en) or have a 111-oat2 I~aw (~;r;,ssrr.r,): mav be h~b~llar. villous, o r tubulovillolls. Frequcnt mitosis, cc.llulal-atypin, ; u ~ t luvs l of'nor-mal polarity in illtestinal ~ l ~ i ~ h e l i of ~ uglands. rn

Micro Pathology

Treatment

Ccrlunoscopic biopsv and ren1ot:t7: repeat cnlnt~~)qcopy [>I. l>aliuin enema f n r periodic sul-veillanr-e.
Tllc oncogene auuociatcrl wit11 adei~r)matr>u\ pchypnsis coli is t h e tumor suppressor gene Iocaterl on chromosome 5. Thc most rnminnn variety i~ adenomato~~s; the risk of malignant transformation increases with size, v i l l o ~ ~ in sorphrdo~ i ~ ~ itlic c l familial form of ~ h disease c (wljich Iias a 1 0091 pi.olml>ilir)af l>rcomirl~ rnalig~~nnt).

Discussion

C,

i%

--I

7 u 0 m

Z
I

m 70 0

r-

0 0

<

COLONIC 'POLYPS

A 21-year-old Female complai~ls of intermittent abdominal pain, mild. nonbloody diarrhea, a11cl anorexia of 2 wars' durilti~n.
She claiins that during these episodes. s h e also tias a fcvel-:she arlds that tlw pain is alnlost always coniincd to the right lower abdomen anrl is cramping in n:t tu1.e.

PE

Pallnr: wcigl~t low abdomina1 mass in right iliac Eossa (thickericd br)wel loop) ; perianal fistulas.

Labs

CBC: megalohlastic anemia; l c u k u ~ ~ ~ t c ~ Guaiac s i s . positive: rtool cxarn rrvcals no parasi teq.

Imaging

RE: g-ranulon~atous colitis and regional enteritis involving rnz~ltiple areas, most common1;v ileum and ascending cnlon, with intervening segments of normal rnucosa.
Terminal ileum (lesions !nos1 cnmm.only seen in iFeocecaI arra Imt can af'fecr anv part of the GI tract) sliows lesions that have a "cobblestone" appearance: discontinuous are- of inffarnrnation, edema, and fibrosis ("SKIP r,~s~nus") : toxic megacolon (thick intestinal w;jll, narmweil h~rnen). Cbmnic inflammatory involvement of submucosal layers o f howel wall (-~-KLVSVUKAL INFLAMMITIOU). marliiesterl mninly by Iynphocytic infi ltl-atiori with associared lymphoid Izyprperplasia and fitrmatiorr of noncaseating granulomas.

Gross Pathology

Micro Pathology

Treatment

Antidiarrheal drup and rystemic glucocorticoids;. ~ a m i n o ~ l i c y l i c acid agents (c.g., n~lfawl;v,ine): wathioprine or mcrcaptopurir~e in par iencs with fi-equent exacerbations; surgery iF paticnts devclop scvrre malahsorptiort, syrnp~otnit~ic flqtl~la\, nr suhacute intes~ it~al nhsrl-iictinn.
Co~nplicationsof Crohn's disease include adh~qions, ulcers, suic-t t~res, ~ ~ F S U I - ~ and F, fistuias. Extrain trstinal manifestat ions may includc arthritis. anfivlosing-spo~~dvfitis, srlernsing ckr~langii is, arld ~ ~ ~ e i Crohn's tis. disease patients also Ilavc. ;I f i v e to six-lid i~lcreaserlrisk of clevcloping color1 cancer; hnwever, thiq risk is much Iowcr than that associated wirh ulccrati\-c colitis.

Discussion

nl CROHNrS D I S E A S E

ID/((

A 54-vrar-old white f ~ ~ l comp1;iins a k of colicky pain in the left lower abdomen and Fever.
She hm had frequent attacks o f moderate pain in the same area 01wveral tnonths a n d one episorlp r)r bloody stools without
~ X C P S S ~ V~nllc-115. P

HPI

PE VS: lotv%~adc. fever. PE: pallor; trrlderness; r e h n ~ l n d and guarding of left lower q~iaclnmt h ~ i normal t stools: sigmoid colon palpable, thickened, and tender.
Labs

CBC: normoqdc, norrnochromic anemia; neutrophilic leukocytosis nit11 associated left shift ( R A W ~ E M I A )S .f no1 culture rcvcals no p n t h o ~ c n s .
C;T, ahdnmrn: di~~ei.ticular diwase with pericolonic inflarnmatory stranding. BE (after acute phasc) : "saw-toothed"

Imaging

appearance.
Gross Pathology
Resoctcd s e p e n c rcvcals oxtcrnal outpouching lip to 1 cm in cllam~ter. alrlng colon herwren tenia roli from lumen; small mucosal opening lead into pouches.

Micro Pathology

A clivertic~ll~im sectiorled alrlng i t s axis shows mucosa and submucosa herniating through a defect in the internal circular layer of muscularis. Biopsy obtained during sigmoidosropy reveals no rnaligttancv. High-fiber diet; antibiotics rvr ~lirertic~ili tis; stlrgical srverelv ir~vol~,ed segments.
resectinn of

Treatment

Discussion

Diverticulitis is a condition of the colon in which the inuceqa a n d sul>mucosaIlrrniate through Qletnuscular layers of the colon FO form o ~ l t p o i ~ r h i n that g s tnav become ohsh-uctecl with Feces. T h e pathogencsis involves increased intraluminal pressure and FacaI weakness of the wall of' the r d o n (near areas of nerve and vessel penelratinn alongside d ~ taeniar e culi). Ozitpouchir~gs may become repeatedly inflanlcd. resulting in abscess Forina~ion. develnpmen I (11-fistulas to acl jniniilg organs, colonic ohs~ruction , perforation. and sepsis. It is InUSK con~motllv seen in the sigmoid colon.

% -4 ; n F? Z
+ rn
n

~ 1 -

G)

fi

DIVERTICULITIS

IDlCC

.3k 60-year-old funialr pre\ei>tswith lower abdominal discomfort, chronic constipation, and passage of bright red hload per rectum. She is a heal? crnokcr, a ~ i d her diet contains a significant a m o n n t nf Feasy fc~odant1 lide natural fiber.
VS: r ~)Ir mal. PIC: pnllor; mild left lower ahclcxninal lenderness 14111 palpable descending coIon; guaiac-positive 5100 l nn rcctal
CKilm.

HRI

PE

Labs

C;BC: no]-~~locvtic. uurrnochr-ornic aiipmia. Frank blood in stool; no leukocytes or epithelial cells secn.
Si y 11 oidoscopv: mdtiple small outponchings in wal Is or descending ar ld s i p o i d colon withorit inflanirna~ion.

Imaging

Gross PathoEogy

Mu1 tiplr .itkcen ~ i m e t tc flaskl tke o u t p o u c l ~ ingq alongside tacniae coli in wall5 ol'clewending ancl sigmclid colnn. Thin-walled hcrniatiunu of awophic mllrosa and comp~essed suhmucosa: t1yp~rtrop41ied circular layer of nll~sctrldl-is propria wi111pr.r~tnirier~i taeniae coli.

Micro Pathology

Treatment

High dietary fiber: s ~ ~ p p l u r r ~ diet e ~ l~ t vith soluhlc Fiber ant1 bulkforrning l a x a ~ i w s awl L xs psyllium.
Diverticuli arc. o u t p u ~ r c h i ~ l in g s he coIonir waEls in which the ;ii2tcricsI~ellelratcihe rt~usculai.iqlaver to reach Lhe rnt~cosal wnH, CI exling an it~herently weak arra. M o s ~ cnmmonly found TI thc dcscc.ncling a114sigtnnid ccdon, divcrticutocis is a div~ase of Wcstern industrialized society.wit11 a low-fibcr/high-fat diet I~eing a significant contribtiltory lactot.. C;omplications include ir~flarnmation of the diver-iiculi (nn~~mc-rrr.1~1.1~) . sielificinl t lower GI bleetlir~g, pel-Inratinn, al~scecs for ma ti or^. and colovesiral fistilln.

Discussion

Atlas Links

K T I T I I X-11-086, PM-PI-086

DIVERTICULOSIS

ID/CC
HPI

A 3.i-vearrtIrf Cen~ale presents with diffimlty swallowing.

She :~Esncc)mplains ni'retrosternal chest pain and heartburn. She 1~ not rurt.entlv taking anv m ~ d i c a t i n n s is . il ric~usrric~ker; and rlrirtks alcrlhol 0131: occasior~allv. VS: nnrn~al. PE: ~lo~-rrhal. CDC: ~ ~ o t t n aESR, l . ,4N,4, )tZ '~ctol-: 110t.mal.ECG: 1101-~nal.

PE

Labs

Imaging

Esophagog~lnl: irrrglllar uncuordinated ~ s ~ p h i ~ corllI-arrg~al t i c l l ~ v(likc a giant corkscrew). Esopl~irgeal manoinetr-v: lowrr pit~'t OF e s e ~ p h ; t g ~ tl~nluristr;~trr ~s prulongrd, nonsequential, lararpampIitude, repetitive contractions of simultaneous onset.
Sublinp~alnitrogly~erinFnracuteatr~~cks:;~nxiulyticsar~d calcit~mchnnncl hlockrrs: H, I~lockerror PI-oionp11mp inl~ihiturs for fissociatrrl GERD.

Treatment

Discussfon

Esophageal spasm is charactrri~ecI 11): uncoordinated esophageal rntlsde contractions thal do iicll ~ ~ r o p Irmd e l i 1310 1 1 1 st~~mach. ~ E~ophi~geal spasn~ ic: Fern i r i acsociniian with advanced age, emotional stress. collagcn V ~ I S C U discasr, ~ ~ ~ I-rf-1ux t.<ophi~fi t ir;, ~ q n l ~ h a g e oh~tr~ictiotr, al at~rl ir-radia~ioti ~sophagiti5. Pain c l w to esophageal spaqm mimics angina,which ~htrulriIlc inclurlccl in [ h e rliCfcr~ntii11 diagno~is.

ESOPHAGEAL SPASM

X 47-ycar-oId ~~11ite m d e is hl-nught bv ambulance to 111e crncrgrrlcy rclorn hecause or massive, painless vomiting of bright red blood (IIFMATEMEC~S) m a d shock.

He is a known homeless dcoholic whu lives in the streets sur rounding the ho9pitar. I-Iis friend slales that he hrts bccn drinking heal-il!. for the past 2 monlhc.
VS: tachycardia; hypotension. PE: skin cold and claintny; hard nodular hepatornegaly: mild splenomegdy; spicier nevi; caput meclusae; rllihbing; ascitcs. mild ~yneconiartia: bilaterally enlarged parotid glands.
Labs

CBC/PBS: ~~ormocytic, normochromic anemia. LOWserum albumin; eIevated alkaline phosphatase; increased hilinabin, A H , AST. ECTD: :tctively hleeding varices.
Tortuauq ancl dilated submucasal esophqcal veins sccoridar-y to

Imaging

Gross Pathology

hunting from portal hypertenvion; superficial ti1cer;lt inn, inflammation. aiid r-uptnre.
Treatment

Restore 171n0d vol~trne,iasoconstrictor.~( e . ~ .rxsoprcssin, , so~natostatin ) , hallonn tamponade of \xrices follrbwrd by endorropic sclerotSlerapr; xplenorenal or other shunt if sclert~thcrap!fails.

Discussion

E~ophageal varices are oftcn silent unlil ~hev rupture and arcassociated with a significant ~ O I - r a l i t rate. y Cirrhosis is L ~ I P ntnqt
other causcs of portal hypetiension [nay alsr~ Ire invnlvrrl, inclitding Bucld-Chiari synclrome, tumor inviision nf the portal vein, ancl n ~ e ~ n l > o l diseases ic that alter liver si11lis~)id~ (e.g.,amyloid).
CO~ITIOII C ~ L I S C .t m t

ph

ESOPHAGEAL VARICEAL BLEEDING

ID/CC

A :1.3-yrar-oId obese inalr chronic smoker presents witla heartburn.

HPI

Hi\ hv;o-il,ui-n worsens w h r n hending aiid lying dawn at night. prcvcnting him Crom ~ l ~ c p i nit g; i h prornp~ly relieved witkt antacids. C;ontinl~o~ csophagcaI ~s pH monitoring c o l - r ~ l a t e s Fmptoms with postui-e.meals. ancl ref111x.

Labs

Imaging

UGI: 5mall biatal hernia; spontaneous reflux t o tnirl-ewipha~~s. EGD: ervthenta, friability. and crosic)ns over esopllageal mucosa.
E~icloncu of inflar~~n~ittion all h i o ~ ~ \n vo : malignant char~gr
llOt~11.

Micro Pathology

Treatment

Cessation of smoking; e l c r . r ~ t i o of ~~ head of bed: weight rerluctio~~; woiclance or f a t Li)odli, ~ coffrr, rlir>cc~lat and ~, alcohol; N,receptor antagonists, prnton primp inhibitors. and antacids provide ~vr>~ptomatic relicac mctoclopramidr i r ~ c r e a s r ~ lower esophageal sphincter prrssurr and ~peed5 gastric emptyirlg7 prrvcrlting rt-flus:sui-gcr?.(e.2..N i w n filntloplication).

Discussion

The pathophv~iolo~w 01gastro~sophageal rtflu?;d i s e a ~ {GE:RD) it~vnlvts ;I ~lrstaincrl decreaqe in LES tone (r-xlrsetl Ilp rnurclc r\+e;rkrless, sclemrlet-nlalike cliseascs. prpgnancv, s m u k i n ~ , aIcnliol, or surgerv), which a l l o ~ v rcflux ~ t o occur. The extent or ciarn;~gcrlrpcnds orr rhr iimotmt nf'refltixed tnalerial per el'isotle, I he 1 ' 1 - e q u e n~Cepisorlcs. ~ tllc clcarancc r ; ~ l c kv gravity and peristalsis. ancl the zatu ot'nc~~traJifi~tinn ni'acirlu ly calim1-y st-crction. CF~rorlic u r ~ ~ r e a tG r dE W can lead to s i l - i c t u ~ .Col-ma~ ti011or culurrlr~arn~etaplasianf dir;~;llesr ) ~ I > ; I ~ I I S epi tllelium (RARRI:TT'SESOPTI.Z(:I~S), wl~ich prrdispoces to e<t)phagcal adenocarcinoma.

t )

k-

i n
X Y

rn
-4
fO

r:
rn

z
0

<

CASTROESOPHAGEAL REFLUX DISEASE (GERD)

:2 .50-year+lcl Caucasian malc presenw with progressively inrr~asing yelInming of the eyes (J~CTNDICE), a peculiar skin rash, a11t I palpitations.

O n rli t - ~ c i e d questioning. t ~ admits c to Fiaving decreased libido. diagnosetl with diabetes and is on oral Three years agn he Ii~~cl>o~lvremics. He qmokes and drinks alcohol nnly occasionaIl~, has nevrr rcccivcd a hlur>d ~ r a n ~ f i ~ s iand n n , haq no prior history of jaundicu.

PE

Generat i7ed hronze discoloration of skin; irr-egularlv irregular p ihse; ictel-us; loss of pubic and xiTlarv hair: iesticular atrophy; firm, nnntendrr, nonpulsatilc hopa~onlegaly.

Labs

Incrrawcl blood glucose: elrvaterl LFTs; dccrcasrd qerlum t ~ s t o s kr-ntir anrl gnnariotropius; increased serum iron: decreased total iron-binding capacity; transferrin saturation > 80%; serum rerriik > 1000 pg/L (hest screening method); rlesfei3rrioxaminrchclatahle urinary Fe excretion > 7.5 g Fe: hepatic Ee quantitation > 100 pmol/g dry weight (cliapostic) . ECC:: atrial fibrillation. CT. i~bdorxien : diff~~sely itlrreasecI liver d c n s i ~ Echo: cadiomyo-

Imaging

pathyGross Pathology

Liver sllows p i q ~ e r l t a r y cirrhosis.


Cirrhosis wit11 ahundanr I?emt>sirIerin rlepositioiz in livcr crlIs, Kupffvr cells. and hile ducts. Rcpeated phlcl~otornitts: monitor fnm- rlewlopment of hepatoma (due to increased r i ~ k: )screen first-degree relatives.

Micro Pathology

Treatment

Discussion

In idiopathic h~tnochromatosia, iron accu~nulates 1111ril the total l ~ o d !iron con tent reaches 51) g. The cansc. is a br-eakdown in the 1101-rtlalcontrol or i ~ o n absorption F I on1 1 h e GI tract: normally the amuunt of iron accl~niula~ed inversely affects the GI mucosal absorption of hot11 hemc and nullheme irnn. rk? iron or7crlnarl prog-ewes, iron that is 01-clinarill;stored in the cells uf the reticnIoendothclia1 sysiern i s depclsited in the livcr. joints, gonads, par~cr-ras, heart. a n d skin.

AtFas Links

HEMOCHROMATOSIS

h S(k.car-old ni;~lrv it11 a hisrrlrr o r alcoholism is UIII csporlsivr


t o rtini~lli ~ 1 1 ~)- ~nr
- r ~ ~1i 3 ~ ;I ~I n i ~ ~ i g h h o lin -

~ h eiiicrgurlc\m r rrtorn,

Hi4 11'igllI>o1. F ~ ~ I P LFI I X ~11~-liad vomited blood ( I ~ F ~ ~ I T T \ I ~ S I S ) t'tilre rnnnttis agn 13111 l ~ E.~c~ETc'cI ~ d no ~ T C ~ ~ I I ~ UTh' I I I , neighhnlalso w \ that ~ the patient got cirr~nk t11rt.r timrs I : week filr lour wars until app~*c~ximatc.lv oiic \.e;tl- itgo. h.ll~srlc rvasring: icteric sclera;spider angiomata Irlur t n incl-rascrl lcvcls rrf' c.; tr'clgc-n ) ; n r ) t l 1li11: ~ hard hepatnmegaly; caput medrrsae; loss o l Imir c ~ 1 1c l l e \ ~ a r ~ t genit;~l l ia; ascites;Lqncco1llac ti;,: testict1l;li ;I 11 c jphv; p;ai,cr~ id r n 1a1-grmc.11~: flapping tremor of hanclq ( ~ s - r ~ ~ :~ palmar u i s ) erythema; digltt pitling etlrma in Irmmcr- extr-t-rnitiru.

Labs

C:RC:/ PRS: $1 ixl~t111r-r>rrlt>c2c\-t npeli in; ~nacrocvtic :~ncrlli;~. I1lrreacer I Iril ii.iilni11; t,lc~:~ted scrtuii 1ri1~1'i;iiii i II;IU- aricl alkaline pltr~sl>hat:~w; low senrm albumin wi I 11 inrr easvrl glr~hulins: prolonged PT; high blood ammonia.
LTGI: esophageal varices. EGIE: ~ s n p l ~ a g\-aric.r~ ~ a l ct,iifirmetI. CIT/I IS. i~lrrlomeu:c n l a r g r d a n d h t t v livr,i.: tortuoLlu, cIi1;tled varicc;~lvcsscls.
EarFv: cnEi~t-gcmcr~t and fi1~1\ inliltrativn of Ihrrr: late: brownish cliscolor~tio~ Ji;~rcler~itlq, l, xntl a ~ r . o ~ oI' ~ lliver ~ y p;u-enchyma.

Imaging

Gross Pathology

Micro Pathology

Ner r o ~ or i ~ ~ r ) ~ . i i iI aIIC ~ ; L ~ O C T ~~ lC i fFf; ~~ i .~ rp r l a c r l n ~ ~vi! - ~ lh l Lhrour co1111cctivu 'tisquu ;~ilcZl v n ~ p h r r ~ viiilil~r lc ;(LC; regenerating nod~lles or liver lacking norma1 organization: rt ~\irlrq>llilicMir l l r jry hr>rlir.i: hilc-t-rurqr\~t.tl r l ~ t c t ~ ~il~ en sd 11rnliferatic)rlor fiht*nhlauts. Discontinue alcohol;supportive IT-ra tn1c.n L o f ilrt i ~ e \ . cncc-phnlop;~tIlt. ~.aiice;tI b l r ~ c l i n g;11irl , anelilia. Mrp:i~icciri-l1osi.i i.; rnrls~co~rirnoir lv ~ ~ I I IN S P : ~ I~. I J I I ~ I : l r s ~ romlnonlv i t i~ r a i ~ s r r l by hiliarv cliscascs, hepatitis R a11d C , It'ilst)n's discasc. il11~1llrr~3ochsorna1osi~. End-stage liver rliwase Icmnrls to liver fitilitr-c. niltritional rlr-ficicrlcirr. C;1 I>leetling,a n d ~oxic ; ~ r r i m o ~ ~ e r rT i il;~ ~. e i i-4 ~ ;11fo ;In incr rased risk nl' hcpatocclILllal' car~i~lotllil.

I 3

V,

Treatment

4 73 0

rn

Discussion

4 rn 70
0 r

I 3

<

Atlas Links

H'EPATIC C I R R H O S I S

ID/CC

A 36-yearold male with vellowing or thp eye5 and skin (due to severe jaundice) i s brolight ro t h e k:R in a n agitated state.
I-Ic has bccn passing black. tarr-y, foirI-smelling s r n o l ~ (MELI:N,I) ancl has exhibited a reversal of deep pattern with &@e sleepiness. A few months ago h e vomited blood (H~XWTEMESIS) and was adrnitted to the hospital, at which time he was d i a g ~ ~ n with ~ e d alcoholic liver cirrhosis.
VS: tachvcardia; tachypnea; hypotcr~sion . PE: lethargic and somnolent: marked ictens; feculent, fruity breath (FETOK HEPATICL~S) : signs of chronic liver disease for~nd; astcrixis, dyqarrhria, and primitive reflexes (suck and snout) rleinnnstrated; exaggerated dccp tendon reflexes: ascites: liver span redrrced; splenomegdy.

HPI

PE

Labs

LFTs markedly elevated (i~~creasrd ser~lrn hiliruhin, MT, and A I d r decreased serum albumin, revcrsed albumin-to-glol>ulin ratio) ; alkaIinc phosphntasr modcratelv elevalcrl; prnlnnged PT. EEG mmrnetric slowing; triphasic waves.
Endoscopy (during previous admission) : bleeding esophageal varices.

Imaging

Treatment

Eliminate precipitating Factors {such as GI bIceding, electrolyt~ imbalance, a i d infection); institllte Iligh-calorie and very low protein diet; bowel cleansing with enemas: neomycin and l a d o s e (induccs rliarrl~ea and clears the g ~ ~ alters t ; bnwel flora; and converts NHs to N H 4 + ,whicli i5 less ahsorhahlc) . The cause of hepatic encepl-ialopathvis multifactr~rialand includcs elevated concenttations of blood ammonia, short-chain fatty acids, false neuro~anm&ters, decreased branchud-chain amino acids, and a circulating substance thal has properties similar to that of bunzodiacpine agnnistq that potentiate the action of GABA.

Discussion

HEPATIC ENCEPHALOPATHY

of increasing ID/CC A 40-ye,71.rhd male pr-eqrntqwii I1 rom~>laints

yellowness of'his eyes and skin, daddy colored urine, and loss of ;y~pelite.

HPI

I-Ie has no history of blood tfilli~ft~sic~r~s, contacl with other jaundicrd prrsons, or espr~surr to all epidemic of hepatitis in the neighbor-l~ood: thc patient is a knowm alcoholic.
F ~ v e l - icterus; : pwnticl enlarge men^: Dupirytren's contracture in left index ancl little fingcr: palmar erythema: rnilrtly tender hepatornegaly; no splenomegalv or asci tes.

PE

Labs

CRC: 1riic1-ocytic mlernia: leu kocytosis. F,lpvaterE serum l~ilirullin; elevated AST and AL.T (MT > ALT, usu;illv Iw il kit lor of hm) (qpically wen in alcoholic livc=rclisrasc; irl oil~ei. pareticlivmal livcr cliscascs, ALT is more t.lev;~tedl: increased alkalin~ pho+ phatase and y-glutrzrnyl-tran~frraw(GGT): s ~ r n l r q i c nlarkers llegative for hepatiti5 A, B. and C:: PT prolongccl.
CJS, abrlomcn: 11cpatornc.g;tlywith coi~rwnecl ~clw texture sug-~cctiveof 3ic-p3titis a n d faltv iiifiltri~iin~~. EnIargcd liver wit11 ycllow anrl v c a s v s ~ i r f : ~ c(sht-unk~n e ~i7e with rnicr.onr~dular 5urf;rr.e is FPPTI in cit.rtintic stage).
Hepatocellular necrosis,neu~ophilic infiltration, and alcaholic hyaline bodies ( M u.I ,ORY RORIFS) : sclrii~ hel~atocrrterdistenclecl wirh fit, displncjng nuclcris to siclc: pcrivunalar and sirll~soidal fibrosis also seen. Abstinence i\ ewential: nutt-itiotlal stlpport: colcIiicine tried with varia hle sl~cccss.
I,ivrr rliseawq p r n r l ~ ~ r eby r l excrssive consumption of clharlol

Imaging

Gross Pathology

Micro Pathology

Treatment

Discussion

inrl~lde fattv liver. alcoholic Ilepatiti~, ancl cirrl~osis; fa';lltv change and, to a n tstonl, ;llcoI~olic hepatiliu nlav rrverse fitllv~vith ah~tinencp. Ten to fiftccn perccnt of chronic i~lcohol ics d~velop cirrhosis. Thc se\'cri? of alcot~nlic hepatilis can I-ang~ froin nrild illness to fiiln~iriant hepatic f;lilut.e.
Atlas Link

r7PM-Pi-093

HEPATITIS-ALCOHOLIC

I DJCC

A ti5-I ear.-old alcoholic male pl-esenr q ~cifh right llppcr quaclr:rr~t pain. jaundice, anorexia, anrl. pmgresqive abdominal distention 2 mr)n~hr' dt~rilrinn: the d i \ l c ~ ~ r i chi+% ~ n r.iipirlly worsened over the pa'' 1U d3y5. He also cornpl;-lin~ nrweight loss. H e has a history of chronic hepatitis (dtlc to hepatitis l3) bur has n o histot-Yolr l~cillatrmusis, mclcn;i. Ilern,~tocl~ezia. or ;~l~cl-ed sellsori~~m.
Jnur~rlicc; palmar erythema; spider mgiomaia over upper ahrintnen; loss of auillary and ptlbic hair; nodular hepatomegdy free fluid in peritoneal cavity lizscr rw) : milrllv cnlargcrl splcru. Incwinect ~lirecl h i l i r t ~ l i l i decreased : serum albumin: inrreased ~et-~u ~sanwrn n i naqe; milrlly elevaterl a l kxline pho<phn~asc: pralonged PT; markedly elevated serum a-fetoprotein (AFP); pnsi tive scrtum hupati tis B \i~-us (HBsAg) surfi~ce antigen.
LTS/CT: i ~ , r ~ g ~ enhancing ~la!hepatic mass: enlatgerl qpleen:

HPI

PE

Labs

Imaging

enlarged portal vein; ascites. Angin: hypervascularity of hepatic n1aw. Gross Pathotogy

Hc.pxtoinrg;-1Iy wirh s i n g l ~ hile-staitied Inh~llated hepatic mass impinging in to portal triacl.


Xscit ic fluid m~ology is he~nnl-rhagic and revcals PI-esencr of malign:ln t ct.115: Iivrr h i c ~ p m reveals in:~crnnoclula~. cirrhosis and hc-j~jtoc\,tcs 111i1tgrow in columns will1 ~ l i s e e possible pat lrrns:
I r~l>err~fal; acii~ill: a

Micro Pathology

~id pse~~rlnglatlrlular.

Treatment

Trimor ernkofiri~iinn: chcrnotli~rapv; r o n ~ i t ls ~~ r~ r g i cresection al rleppnrlitjg o t ~ stage.


Hep;~tor~11111;11rarrinoma i q a maligtiarli primal-v rleol~lnsm or t h live-r. ~ In tEic IYc'ster11wol-lcl,it usually arises from a cimhatic liver and, rvhcrcwr prcvalcnt, is fi-eql~cr~tlv associ:t~ ~ t ~ i t f i hepatitis R ilntl h r p a t i ti^ C infuctinrr. or tie^- prrdiuposii-r~ conrlirir~ns i t ~ c l u r hernoclirotnatosis. l~ Wilson'~ discasc. n,-:tntitrvpsi~ic l c f cicrrcv, alcohoTic cii-rhocis. nnrl atl;ilo?ti~~ 151.

Discussion

TI

iq

oltrn

cpr ead I>v hematogenous and lvn~phatic di<seniination, tn the I I I I I ~ ? .

-?

HEPATOCELLULAR CARCINOMA

ID/CC

;2 50-veai.wld rrlaIe ~ v i t h ad~anced alcoholic cirrhosis develnps

oIi+a HPI

and abdominal distention.

His renal and electrolyte status 11a.i heell steadily dctcrierathg.

PE VS: ~acl~\~c;lr-tliil. 1'E: j ; ~ ~ ~ n d iaqritrc: ce: asiprixis.


Labs

BUN increases rlisproportio~~atelv to setttm creatinine (indicating p e l - e n a l azntcnlia): ml-kcrl increase in urinary osmolality. LrA: no active sediment: r:~nclo~ii ~ i r i r ~sodi t : otri Inu:

Gross Pathology

Renal hir~ps\~ show? tin ;~hizol-n~alit~.


Kevrrs;ll n l rcnal Iailurc call occur uilh ~ ~ ~ c c t ' s s livrr f i ~ l11-anqpl:intation. In contl-xst. survi\,;~l( 1 1 ~r l i a l ~ \ i s ir grn~rallv limiter1 by thc scvcrim of' thc hrpatic fi~itul-r. No proven druq therapy.

Treatment

Discussion

Hrpa torrnal svt1dr.clrnc (HRS) refers to the rlc\-elr)p~nenr of acutc i-r.r~al Llil~lrc i n n patient wit11 :adv,tr~ccdh e ~ ~ it-i disease. t [t i s ofterz c i ~ l ~ s by r d ftrlminant hepatic faihur, cirrhosis, and. less

Fr.ec1ut.n~ l v A , metastatic tumor or severe alcoholic hepatitis. IHRS 11s11ally represenls t h e end stage of n reduction i n rend perfmion induced IIV inci-ci~sii lgly scvt-rr. I~cpai ic i n j ~ ~ tIv t. cT;rr-rim a high mortality.

rri
; D

"

HEPATORENAL SYNDROME

ID/CC

X ti.5-war-t,ld male wit?th uwesectable carcinoma of the s i p o i d colon is cx~:~lunted for hepatomegdy thal was drtcctrcl on iI

Follort-11pexam.
HPI

The patient is 11nrlergctingcl~cmothn-;tpv arzd 11nclet.w~nt palliatirc st1rySrr-y 1 month r q u .

PE

Marked pallor; large nodular liver palpable; ascilcs (dtlc to prritoncnl sccding); coloszamy bag norerl.
Markedly elevated carcinoernbryonic antigen (CEA) levels; markedly raked alkaline phosphata-e:orher L F T s normal. CT/US. :~bdorr~rn: rnnl~iple etihanring- hepatic nodnlcs. Sig~~~oirlnqcop.: infiltrating "napkin-ring" growtt) in sigmoirl col011.
On ;iutopsv, multiplr. n o d ~ ~notetl, l c ~ some with c ~ tral n necrvsis (d~rr t c l invificirnt vascular s ~ ~ p p l y ) . Suppc~rtivc managcmrnt; consider RF ablatioil rnr solitary meca.;tasis.
l,ivri rnetastasr.; XP nlore common than primary tumor< r j f ' the liver. The most common ~tirnol-s that rnetaslaqi7~ tc) the liver arc colon, gaqtr-ic,pancreati~. breast, atld lung carcinomas.
PG-PI-096

Labs

Imaging

Gross Pathology

Treatment

Discussion

Atlas Link

METASTATIC CARCINOMA-LIVER
-

A 34-year-old alcoholic malr complain5 of s ~ ~ d d c n u r l s e unret, Icnting midepigastric pain radiating to the lower thoracic spine.

H e idso complains of assnciatecl anorexia, nausea. and vomiting. T h e pai 11 I~ecolnes worse when he is u ~ p i n e .
VS: hypotension; tachycardia. PE: palc. nvcatv; in scvcru dixtrtss: perklmhilical ecchynoses (Ct ,I,I.~:N's SIC:^') ; left flank ecchymosis (GWL\.TP'RNEK'S S I ( : K ) ; m;n.ked epigastric tenderness ant1 dif'f'iise rehound tenderness bitt minimal I igidily; a3xlor11eii dicrel~dcrt with mal-kerlIv rl~cr-eaceri htnvpl sn~inrEs.

Labs

C:BC:: Ir-ukocv~osis. Markedly elevated serum amylase nrrrl lipase: elevated glrrcose: clrratcd STDT anrl LDT-I; hypocalcemia.

A R G : hypoxunia.
Imaging
I:NR/F;L"I<: nr, fret=air unrler rliapl~ragm: ahrnpt trrmination of gnwoel5 transverse colon ;ir ~plcnic ff cxrlrc (COL(.)K TLlTnFF q1c:T'if : rlistc.nclcd loop of I~owel in proxi~nal jejtlnllln (S~.NTINIII.I O ( W ) . I : ? ;~l)dnmen: , enlargement and inhomogeneity of pancreas; streaky peripancreatic inflammation.
,111lr qwy: pancrcts i.e\,eals pasly W I I ~ I P lbci c)f fat necrosis, hemorrhage. and cystic ca\ritatior~.

Grass Pathology

Micro Pathology

Edt-ma o f cclniiert i t r I~SCIIP, ~ p ~ > I y t t a ~ r p I i n ~ ~ infiltl-atictn, ~cIear hprnorrhage a n d n ~ c m s i s af parzcreatic acini; fat nccr-osis apprars as pale bluc :imorphous foci rvhcre adipocyt~ i i i e i ~ ~ h t n arc n e ~rli5sol\;pd.

Treatment

"Rest" the pancreas (analgesics, n;ao~astric SIIC~~O~~).

I V fluids, n o on1 intake,

'Discussion

Gallqtones and alcohol abuse are etiologic f;~ctorsin 90% of' patieniq wilh x u t r pancl-eariti5. Gallctones are t h o ~ l g h to t cause panrrrmitis hv transient obstruction at the ampulla of Vater, which I c a d ~ to i ~ l c r c a ~ c parlurearic d ciuulid pr.c\sxu.e. Other C~IISPF i t ~ c ~trle l irifectionq (e.g., mumps) . hel-edi tar-v pancrc:lti ti?. qhock, acllte ischemia. hyperc;llcemia, h ~ ~ ~ r t r i g l v c ~ r i c l m a nid ii~, clruqs (e.g., cfliazides. ~utfonamiclcs). Complicaliorls i r ~ c l ~ ~ r l e DTC. shock.AWS. h ~ - p ~ c a l c e n ~ ac.tIle i a . ren;ll hiI111.e.a114 pancreatic psez~clocvst.

PAN CREATITIS-ACUTE

: Z P+yeai.-nld ~ v h i r e frrnalc conlplai~ls of crampy abdominal pain, irlabili? to pass flatus. a b d o n ~ i ~ distention, ~al nalusea, and
vo111iling.

After lack of impl-cwrmer~ t will1 H 41out-sof nasogastric-tube suction 'i~id rCTfluid, she underwent laparotor~l?~ to relievr bowel obstfirction ( d ~ lo ~a r large ha~nartoma) .

Hyperp@ented m a d e s on lips and buccal mucosa and o n p:llms, lingers, and tocs; multiple harnartomato~~s growths palpatud throughout GI bact.
Labs

CRC/PRS: microcytic, hypochramic anemia (duu to rnorlel-ate GI bleeding). Positive stool guaiac les!. UGI/SRFT/RE: acute small. I,o1+-e1obstruction: multiple polypoid p w t h s ofjqjunutn. ileum. ancl colun.
Pc~luncul~~zrcl nodules up to 2 rm in size in stomach. d u o r l ~ nu~li j ,t j ~ i ~ i i i ileum, ~ n . anrl colon.
Increased melanin deposition in buccal mucosa and lips; 11nmartom:ttou.jIt.sioris r,ar.ely undergo malignant t r.al~sfnrn~ation; smooth ~nusclu a11rlcoilnective tissue cxtund into t l ~ c pcdunculaled polyps anrl rol-m an arborizing nenvot-k.

Imaging

Gross Pathology

Micro Pathology

Treatment

Periodic \lit-veiElance; consirler endoscopic rums1 al of all polyps: treat cn~nplications such as obstruction, intuss~lsception, md bleeding.
P ~ u r ~ ] e g l ~~yt~clrome ers is autosemal dominant and i s one typc of hcrcditary familial. 1~01ynsis cyndrolne. Polyps arc lianlar-tc~lnns with low malignant potential, so resection is performed only if polvps are symptomatic. The condirioi~ is associa~rd with an increased risk of extraintestinal cancer (e.g., pa~cwaas. Ix-east. lung, ovary, :,ittcrus).

Discwssion

ID/CC

,I 36-year+ld while female ctmplains nf rlizzineqs. ratiglre. weight Iosr. a n d difficulty swallowing solid ht)rl ( ~ \ ~ I * F I ~ \ G L I ) .
She Fias hccn conccl-ned ahout a reccnt craving for ice and clay (r'rc:t). Shr reports n o nausca/\wmitillg 01- hen~;~iemer;is/ mclr-na.

HPI

PE

VS: tacllycnrdia. PE: pale skill and mitcouu i n e m l ~ ~ x ~ ~ e s ; spoon-shaped nails ( ~ o [ ~ o n l r : ~smooth, r , i ) ; shiny red tongue
((;I.O~S~TI?) : <t[>nlillitis.

Labs

CBC/PBS: rnicrocytic, hpochmmic anemia. Low serum iron. UGI: ~ l i i r ln~enil>rarles OF ~q11amo119 InLlcoqa typically in mid- or IIIIPPI. e ~ o p l ~ a g l (ESOPI ~ q ~ n t : j~r .. ~ ' ~ < R H I v ~ ; ) .
Postrl-icoirl e~ophagrral concentric rrcb.

Imaging

Gross Pathology Treatment


Discussion

Esophagu;ll rlilatalion; s t i p l ) l c m ~ r ~ iron. ~al


Xlro calIccl Paterso11-Rclly s y d r o ~ n ePlummrr-Vincnn .
SF-

tlrorrrc is assoui:tte~lrt-i;1111ail increased risk or esophageal cancer.

PLUMMER-VINSON SYNDROME

A 55-year-olcl woman presents with incr-easing yellouing of thc pvrs ( * ~ . ~ . I L I ~ N ~ I' I a( : t igme, F.) anrl chronic itching.

She also has rheumatoid arthritis and chronic thymiditis. She lras no prim history ofjaundice, dnes not drink alcohol, a n d ha^ never- received a blood transfirsion. Icterus; numernus ~crarch markc: and ~xcnriations o n h e skin; santhamas and xanhlasrna; hepatomegalv, splunomegaly, and no ascitcs; rheumatoid joint deCormities and goiter ( d u e to chrwnic i h ~ o i dlis). i

Labs

Raiscd ESR, markedly elevated allraline phosphatase; mildly elevated direct bilirubin ancl a ~ ~ ~ i ~ i o t r a ~ ~ s elevated fera~es; serum cholesterol (> 300 ~ n g / d L j ; high titers of scrlrn) antimitochondrial antibody.
US, abdomen: hepatomegaly and sple~io~~iegaf y.

Imaging

Gross Pathology

Dark gt-een, ertlarged liver- (in late stages, liver s h o ~ cirrhosis ~s that rannot he diainguishrrl frcstn other causes).

Micra Pathology

Bilr d ~ ~desi c t ruction with tymphocytic-p1astnac:~ic infiltration sf portal areas; periportal epithelioid granuloma formation and
iicllc-6 (in

portal scarring with linking o C portal tracts: periportal l~ile stasis advanced cases. cirrhosis may be Found).

Treatment

Q~oles?rarnint. tu control pruritus; immunosnppres~ion; liver ti-ar~spianmtionis the r ~ t l l y defiriitive treatment.

Discussion

Primary hiliary cirrhosis is a chronic livcr disrasc ofpsohabIe autoimm~me eunlclgy t h a t clcc~u-s primarily in middle-aged women and is characterized hy nonsuppurative obliterative cholangitis that progresses to cirrhosis: it is associatrd with other a~~loimnlime direitses in 8 5 % of cases. Complicatior~sinclude cirrhosis and portal hypertension, malabsorption due to steatorrhea, and osteoporosis due to rnalahsorption of vitamin D and calciurn.

Atlas Link

P R I M A R Y B I L I A R Y CIRRHOSIS

IDJCC

ill 6C)-ve;lr.old male present5 rr'ith compIaint5 nf fever, generalixed abdominal pain, persistent bloody diarrhea, ancl increaqing rectal pain.
Hc !%asdi;zgnoscd ui th ulceralive cnlitic srvel-al vears ago and hnd hrtrl ~ r r a t e d wi11i sulfas;~l;uiiie a n d predtii~oneC n r intermitlent

HPI

exarel-haiions.

PE

VS: fever (39.9"C); tachycarclia {FIR 120): miIcI h y o l e t ~ s i o n (RP XX/5O). I'E: toxic-lookingpatient: pallor uotcd: al~do~ninal exam rr.ve~ils generalized tenderness and reduced howel sounds.

Labs

I:13C:: normocy~ic, n o r m o c h r o ~ ~ ianemia. ic RBCs, puu. and e p ithelial cells secn on stool csam.

Imaging

XR. ahdomcn (inclrlding cross-table lateral ) : moderate diatation nf' descending colon; thickmed colonic wall (cluc to w:tll edernab: 110 ~ v i c l ~ n or c e intesti~lalperforation. CT. alxlomen: no pvirlence or diverticular abscesses. Bal-ium enema is cc)t~rraindirated d ~ i c to risk or pcrfoi-ation.
%lorlet-afe rlilatation of cnEat~: rcctal muctlsa frialjlc artd swnllen; cornpl.ctc. loss of mucrls;rl folds. Uifllire mono11uclral- in fill rate in lamina propria with neutr-~~phils, mast cells, and cosinophils.

Gross Pathology

Micro PathoZogy

Treatment

Bowel decompression with uasog-a~tric suction; fluid and t=lertrnl\t e replacempen t; parenterd antibiotics l o prevent sepsis; corticostcroids may hu inclicatud tcl suppress iriflanima~orv react ion in gut: surg-ical colectomy if immediate sl~edical mra+ ures fail.
Toxir n i e ~ c o l o n results from ct~rnpIcte st~utdowr~ ofcolnnic nez~ro~nuscularf~ n c t i o n dz~r to inflan~niazinn o f r he myenteric neural plexus. 11 may he seen ;IS a complication or Crol~n's rli~ease. ischemic colitis, pserrdomembranorrs colitis, and ulcerative colitis.

>

Discussion

m
i3

--I

0 r

0
tl

<

TOXIC MEGACOLON

A 31-vex-old male complains nT having more than five howeE movements a day together with cramping abdominal pain and
tenesmus.

The patien I adds that his stools consist of watery or pasty material with rnllnu m ~ gross d quantities of blood. He also cnrnpIains of intermi ttenl fatigue, Fever, and an increased need for sleep.

PE
Labs

VS: tnild fever. PE: Iocalized lendernew over distal colon.

CRC;: ancmia; leukocvrasis: hypoalbuminemia. Elevated ESR;


stool exam reveals no parasites; no bacterial pathogen isolated in culturt..

Imaging

BE: early mucosal g-ranularity;later, rigidity and loss of hauqtrations ( " L E ~ D PIPE"). with ragged ulcerated mucosa and ulcerations. Cnlonoscopy: mucosal erythema and granl~larity wit11 hemorrl~ agin g and inflammatory pseudopolyps.

Gross Pathology

Scarring a n r l coarse, granular tnucnsat surracc indicating prescncc of micl-oulcerations; mucasal surface is friable; lesions are continuous from anal to oral direction.
Increased numhcrs of Iy~nphocyter, plnsma cells, and PMYs; atrophy of mucosal glancls and presence of PMNs in crypts or 1,ieherkiihn {often caFIecl crypt absces5es) : inflammatory changes confined to mucosa and s~thmucosa.

Micro Pathology

Treatment

Antidiarrheal drugs; sulfasalazine, 5ASA preparations; glucocorricoids; cpclosporine for severe colitis; surgcry if intlicared (total proctocolectnmy is rui-ative). Patienu wii h ulc~rativecolitis are at inmawd risk Tor colon cancer. Factoi-s favoring the developmertt of colon cmcer in o r longer, ulcerative colitis are Lhe duration of clisease for 8 year% involverner~ to f the e n tire colon, continuous clinical acti~lty, and, possibly, a severe irii tial attack. 1t is routinely advised that patients undergo regular surveillance that includes colono~copy and an examination of multiple biopsies for dysplastic changes or frank cancer. Mnjor complications include toxic megacolon and mas~ i v intestinal e hetnorrhage with shock and sepsis. Extraintestinal tnanifestations may include arthritis, erythema nodosr~m. arlkylosing sponclvlitis, and sclerosing chnlangitis.

Dircussion

AtEas Links

ULCERATIVE COLITIS

ID/CC

A 43-v~ar-nlcI white male complains of severe burning epigas~c pain and diarrhea o f '2 yrars' dura ti011 t l ~ t a has been re1 rartory l o mr-dical managrment.
The pain awakens hit11 early in rtie morning, is accompanied by nausea and ~ornitirtg, increases with coffer c o ~ l s ~ t n ~ p t iand on, also appcars 2 10 3 ho~rrs after meals. Three d a y ago, he also noticrrl black stools.
Sligh~ discomfort on cpigasliic palpa~ic~n hnr n o signs of prritnll~alirritation: pale skin aild mc1coir.i ~ i ~ e n ~ l ~ i occl~lt an~s: blood 0 1 1 digital rertal rxam. Fisting serum gastrin markedly increased; increaqed gastric add
cix~tpl~ (M t WEIZI:$-ILC)KH~"IIM (~ I) ~IIC to e1rv;;ttedrasll-in).

HPI

PE

Labs

Imaging

t:TJMR/rZt~gio: rtnall l e ~ i o n in parlcrcas. difficult lo locali7e. LTC;T/NRFT: atvpical ulcers: gastric folcl tl~ickening.
Wcers in uncommon places in esophagus, duodenuni, and jqj1111lum(due to excessive gastl-in secretion) : gastrinoma (commonly in pancrcas or dnotlunurn).
IT5ui1ll~ originate fmrn delta cells of pancrclas: original lesion may he adcnoma. h?peipl;t.;ia,or tin-cii~orna: hyprrplaqia of

Gross Pathology

Micro Pathology

an tl-al ax41rin-prnduci~r g cells.


Treatment

High-rlace nmeprazole: surgical resection if nrll tclcalijled and no mct;istasrs; gaqtrrclr*ln?2: vagol nmv. Znllinger-Ellison wndrome causes paillfill chronic diarrhea (vs. intestinal pa~.asitelj,cal-cinoid svndl-oine. ulcerative colitis); I - K > I I ~ Ihalf' I ~ ~ may be malignant. It is associated with rnllltiple endocrine neoplasia (MEN) type I (WERX~ER'S SITU[)KWME).

Discussion

Atlas Links

1 1 1 . PM-PI-103

f7TYX MC-112

ZOLLTNGER-ELLISON SYNDROME

You might also like